Ευκλειδης Β 75

Page 1


Επιμέλεια:

Για την Α' τάξη. AS. Έστω

ότι:

Αντώνης Κυριακόπουλος Γιώργος Τασσόπουλος

α2+ β2+ γ2+ δ2+α β+ γδ=1.

Να αποδείξετε ότι:

Π ροτεινόμενες Ασκήσεις

αδ-βγ -:1- 1.

Να λυθεί η εξίσωση: 9σuv2x +9σuv'x = 4. Για την Γ τάξη. Γ3. Μια συνάρτηση f είναι ορισμένη και τρεις φορές παραγωγίσιμη στο διάστημα Δ= (α, β) . Υποθέτουμε ότιf(χ) �Ο, για κάθε χ ε Δ και ότι η fέχει δύο (διαφορετικές) ρίζες στο Δ. Να αποδείξετε ότι η εξίσωση: f'"(x) =Ο έχει μία τουλάχιστον ρίζα στο Δ. Για την Β'τάξη. Β4.

Αν

τότε

Λύσεις Προηγούμενου τεύχους 74.

Α4.

ΚΔ+ΔΓ+ ΓΛ�ΚΑ+ΑΒ+ΒΛ,

π. ΚΔ+ΔΓ+ΓΛ�-

Άρα: ΟΜ<

Π

4

.

ΟΜ<-. 4

ΜΚ+ΜΛ

2

ΚΔ+ΔΓ+ΓΛ Π �<

2

4

Χρ. Παλυβός 4° Λύκ. Πετρούπολης

ΔΒ

2

J2

.J3-1

--

::::?

(�J'

2

4 -2.J3 --4

1

+--

2

.

.J3-1 ΜΕ= -- , 2

.J3-1 . J2 . J2 2

.J3-1 2-.J3 --2 2

4-2.J3

(.J3-1)2

2

2

=

(ΕΒ ) =

2

1

2

=

.J3-1

2

.J3-1 J2

(α-β)(α-γ)

γ2 +---'--(α-β)(β-γ) (γ-α)(γ-β)

J6-J2 2

=(β-γ)(α2-αβ-αγ+βγ)=...=(β-γ)(α-β)(α-γ).

Άρα Α=1

.

Ερ. Χατζηφωτιάδης, Λύκειο Μελισσίων

Άσκηση Γl. Θεωρούμε δύο πραγματικούς αριθμούς α και β. Αρκεί να δείξουμε ότι: f(α)=f(β). Εξετάζουμε αν υπάρχουν πραγματικοί αριθμοί χ και y για τους οποίους ισχύουν: 3χ-2y=α και 2χ-y=β. Έχουμε:

{2χ-y=β

2

=

β2

α2

3χ-2y=α

+----=

Έχουμε: (EZ)=(ME)+(MZ)=.J3=ρ.J3=λ3 οπότε ΕΟΖ= &3 =120°

.

_

-2ΜΕ·ΜΒσυν45σ =

-2

α2=-α β-αγ

Ο αριθμητής του τελευταίου κλάσματος είναι ίσος με: α2(β-γ) β2α+ β2γ+γ2α -γ2β= α2(β-γ)-α(β-γ)(β+γ)+βγ(β-γ)=

=>

Λ

( J2

::::?

Έτσι έχουμε: 2α2+ βγ=α2-αβ-αγ + βγ = γ

(α-β)(α-γ)(β-γ)

.J3+1 .J3-1 ΜΖ=--+1=--. Όμως Μ1 =45°, οπότε: 2 2 .J3-1 2 J2 + 2 : 2

α=

α2(β -γ)-β2(α-γ)+γ2(α-β)

2 '

οπότε: ME·MZ�MA·MB=>x (x+l ) �

(

α +β+ γ=Ο

::::?

ii). Ονομάζομαι Α το πρώτο μέλος της (1). Έχουμε:

=? ΜΑ=ΜΒ=

(ΕΒ)2=( ΜΕ)2+(ΜΒ)2

i) Έχουμε:, -β1-

1

Γ. Κλάδης Σχολή Αυγουλέα- Λιναρδάτου

Άσκηση Α2.

Α=

2χ 2 + 2χ-1=Ο::::? χ =

- 13s+

=

και2γ2+αβ=(γ-α(γ-β)-:�-0.

z

=ρ=1,

ΑΒ=λ 4 =ρJ2 =J2

=

Όμοια: 2β2+ αγ =(β-α)(β-γ) -:1- Ο

μεσοκάθετος των ΜΝ, ΕΖ=?ΜΕ=ΝΖ=χ. ΜΝ =

-

-

ο

]

=α( α -β)-γ (α-β)=(α -β)(α -γ) -:1- Ο.

Β3. ΜΝ//ΔΒ=?ΑΓ

Εξάλλου:

( J2]

π

ΟΜ�

[ Ξ ( � � ±J �

1 2 1 20 1 2 1 J2 = πρ ρ ημ120 -2 2 2 360 -2

Αρκεί να δείξουμε ότι:

,

ΑΜΝ )

Έστω Μ σημείο της περιμέτρου του ΚΔΓΛ. 4

πραγματι:

[

� (Ο· m)-(ΟΕΖ)-(

Άρα: ( ΜΑΕΜ)=

.

<=>

3χ -2y=α

χ =2β- α -4χ +2y=-2β <=> {y=3β-2α

{

Άρα, θέτοντας στη δοσμένη ισότητα χ=2β-α και y= 3β -2α βρίσκουμε ότι f(α)=f(β). Συνεπώς η συνάρτηση f είναι σταθερή. Αν. Σχοινά, 1° Λύκειο Γέρακα


ΕΛΛΗΝΙΚΗ ΜΑΘΗΜΑΤΙΚΗ ETAIPEIA

Τ ε ύ χ ο c; 75

Ι α ν ο υ ά ρ ι ο c;

Φ ε β ρ ο υ ά ρ ι ο c;

ΜΑΘΗΜΑΤΙΚΟ

Μ ά ρ τ ι ο c; 2010· Έτ ο c; λ θ' - Ευρώ: 3,50

e-mail: info@hms,gr www,hms,gr •

nΕΡΙΟΔΙΚΟ

ΓΙΑ ΤΟ

ΛΥΚΕΙΟ

ΠΕΡΙΕΧΟΜΕΝΑ

.f Αρχαίοι Ρόδιοι Μαθηματικοί και Αστρονόμοι

.f Τα περίφημα "άλυτα" προβλήματα της αρχαιότητας

4

.f Μαθηματικοί Διαγωνισμοί- Μαθηματικές Ολυμπιάδες

7

.f Homo Mathematicus

14

Μαθηματικά Α' Τάξης

Μαθηματικά Β' Τάξης

.f Άλγεβρα: Εκθετική-Λογαριθμική συνάρτηση

.f rεωμετρία: Κανονικά Πολύγωνα - Μέτρηση Κύκλου .f Κατεύθυνση: Κωνικές Τομές- Θεωρία Αριθμών

ναβάθμισή του με την καλοπροαίρετη κριτική

18 26

σας και τις ενδιαφέρουσες εργασίες σας • Η συνεργασία σας μας είναι πάντα απαραί­

29 35 42

Μαθηματικά Γ' Τάξης

.f Μαθηματικά fενικής Παιδείας: JYXAIO-ΠΙΘΑΝΟ -ΒΕΒΑΙΟ 52

.f Μαθηματικά Κατεύθυνσης: Παραγώγιση- Ολοκλήρωση

59

.f Το Βήμα του Ευκλείδη .f Χάρης Καλλιγάς

72

.f Τα Μαθηματικά μας Διασκεδάζουν

76 80 82

•••••••••••• • •••••••••

τητη και μας ενδυναμώνει στη συνέχεια της προσπάθειάς μας Σε μικρό χρονικό διάστημα θα έχετε στα χέρια σας και το 4ο τεύχος για μια ολοκληρ­ ωμένη επανάληψη Με ευχές για υγεία και πρόοδο Ο Πρόεδρος της Συντακτικής Επιτροπής Γ.Σ. Τασσόπουλος

74

.f Ο Ευκλείδης προτείνει...

.f Στήλη του Μαθητή

ΜαΘηματικός Διαγωνισμός 27

Σας ευχαριστούμε που περιβάλλετε με εκ­ τίμηση το περιοδικό μας, και βοηθάτε στην α­

.f Άλγεβρα: Σύστημα δύο γραμμικών εξισώσεων

με δύο αγνώστους .f fεωμετρία: Εγγράψιμα Τετράπλευρα

Γράμμα της Σύνταξης Αγαπητοί μαθητές και συνάδελφοι,

;\? /UJ\J-JΔJ-JZ:

Φεβρουαρίου

201 Ο

Ο Αντιπρόεδρος της Συντακτικής Επιτροπής

β. Ευσταθίου

Σύνθεση εξωφύλλου: Από το έντυπο "L'edυcαtion Aυjoυrd'hυi"

της UNESCO του 2005 σε ελεύθερη σχεδίαση

••••••••••••••••••••••••••••••••••••••••••••••••••••••••••••••••••••••••••••••

Συντακτική επιτροπή

ΕΚΔΟΣΗ ΤΗΣ ΜΑΘΗΜΑ1ΙΚΗΣ ΗΑΙΡΕΙΑΣ

ΠΑΝΕΠΙΣΤΗΜΙΟΥ 34

106 79 ΑΘΗΝΑ

Trt-.: 210 3617784-3616532 Fox: 2 1 03641025

Εκτελεστική Γραμματεία

Αθανασόπουλο<; Γεώρyιοc;

Κόντζιαc; Νίκοc;

Ιdί'rη Εύα

Αναστασίου Γιάννηc;

Κοτσιφάκηc; Γιώρyοc;

Ιταϊκοc; Κώσταc;

Πρόεδρος:

Ανδρουλακάκηc; Νίκοc;

ΚουτρουμπεΛαc; Κώσταc;

Ιτάϊκοc; Παναyιώτηc;

Τασσόπουλος Γιώργος

Αντωνόπουλο<; Νίκοc;

Κυριαζήc; ιωάννηc;

Ιτρατήc; Γιάννηc;

Αρyυράκηc; Δημήτριοc;

Κυριακόπουλο<; Αντώνηc;

Ταπεινός Νικόλαοc;

Βακαλόπουλοc; Κώσταc;

Κυριακοπούλου Κων/να

Αντιπρόεδρος:

Εκδότης:

Καλογερόποuλος Γρηγ6ριος Ευσταθίου Βαγγέλης Διευθυντής: Τuρλής Ιωάννης

Γραμματέας:

Χριατ6ποuλος Πανaγιώτης

Επιμέλεια Έκδοσης: Μέλη: Τασσόποuλος Γιώργος

Ευσταθίου Βαγγέλης Ζώτος Βαγγέλης

Κωδικός ΕΛ.ΤΑ.: 2055

ISSN: 1105- 7998

Αργυράκης Δ. Λουρίδας Σ. Ταπεινός Ν.

Ευσταθίου Βαyyέληc; zαχαρόπουλοc; Κων/νοc; Ζώτοc; ΒαyyεΛηc; Κακκαβάc; Απόστολοc; Καλίκαc; Σταμάτηc; ΚανεΛλοc; Χρήστοc; Καραyκούνηc; Δημήτρη<;

Κυβερνήτου Χρυστ. Λαζαρίδη<; Χρήστοc; Λάππαc; Λευτέρη<; Λουρίδα<; Ιωτήρηc; Μαλαφέκαc; Θανάσηc; Μανωλάκου Στοματική Μαυροyιαννάκηc; Λεωνίδαc; Μενδρινόc; Γιάννηc;

Τασσόπουλοc; Γιώρyοc; Τζιώτζιοc; Θανάσηc; Τριάντοc; Γεώρyιοc; Τσαyκάρηc; Ανδρέαc; Τσατούραc; Ευάyyελοc; Τσικαλουδάκηc; Γιώρyοc; Τσιούμαc; Θανάσηc; Τυρλήc; Ιωάννηc;

Μεταξάc; Νικόλαοc;

ΦανεΛη Άννυ

Κατσούληc; Γιώρyοc;

Μπρίνοc; Παναyιώτηc;

Χαραλαμποπούλου Λίνα

Κερασαρίδηc; Γιάννηc;

Μυλωνάc; Δημήτρη<;

Χαραλάμπουc; Θάνοc;

Καρδαμίτσηc; Σπύροc;

Μώκοc; Χρήστοc;

Χριστιάc; Ιπύροc;

Κηπουρό<; Χρήστοc;

Πανουσάκηc; Νίκοc;

Χριστόπουλο<; Παναyιώτηc;

Κλάδη Κατερίνα

Ρέyκλης Δημήτρη<;

Ψύχαc; ΒαyyεΛηc;

Καρκάνηc; Βασίληc;

•••••••••••••••••••••••••••••••••••••••••••••••••••••••••••••••••••••••••••••• • •

Τα διαφημιζόμενα βιβλία δε σημαίνει ότι προτείνονται από την Ε.Μ.Ε. Οι συνερyάτεc;, τα άρθρα, οι προτεινόμενες ασκήσεις, οι λύσεις ασκήσεων κτλ. πρέπει να σrέλνονται έγκαιρα, σrα yραφεία τηc; Ε.Μ.Ε. με την lνδειξη "fια τον Ευκλείδη β'". Τα χειρόyραφα δεν εmσrρέφοvται.

Τιμή Τεύχους ευρώ 3,50 Ετήσια συνδρομή (12,00 + 2,00 Ταχυδρομικά= ευρώ 14,00) Ετήσια συνδρομή για Σχολεία ευρώ 12,00

Το αντίτιμο yια τα τεύχη που παραyyέλνοvται σrε'λνεται με απλή εmταyή σε διαταyή Ε.Μ.Ε. Ταχ. fραφείο Αθήνα 54 Τ.Θ. 30044 ή πληρώνεται σrα yραφεία τηc; Ε.Μ.Ε.

Εκτunωση: ΙΝ7ΕΡΠΡΕΣ Α.Ε. τηλ.: 210 8/60330 Υπεύθυνος τυnοypαφείου: Β. Σωτηριάδης


Αρχαίοι Ρόδιοι Μαθηματικοί και Αστρονόμοι

Σ

Καραγιάννης Γ ιάννης- Τσομαρ έλη Τ ριανταφυ λλιά,

την εργασία αυτή θα προσπαθήσουμε να αναδείξουμε αρχαίους μαθηματικούς και αστρονόμους που γεννήθηκαν, έζησαν ή εργάστηκαν στη Ρόδο ανάμεσα στον 4°v - 1 ον αι π.χ που ,δυστυχώς, οι περισσότεροι δεν είναι ευρύτερα γνωστοί. Σχεδόν σε όλα τα μαθηματικά sites ανάμεσα στα ονόματα σπουδαίων Αρχαίων Μαθηματικών και Αστρονόμων αναφέρονται και τα ονόματα Ρο­ δίων που ξεχώρισαν για το έργο τους. Ανάμεσα τους αυτά των Άτταλου του Ροδίου, Γέμινου του

Ροδίου, Εύδημου του Ροδίου, Ίππαρχου του Ρο­ δίου, Παναίτιου του Ροδίου και Ποσειδωνίου του Αππαμεύς ή του Ροδίου. Είναι ιστορικά απο­

δεδειγμένο ότι η Αρχαία Ρόδος ήταν επιστημονικά και πολιτιστικά πολύ αναπτυγμένη και ότι κατά καιρούς είχαν περάσει από τη Ρόδο πολλοί αρχαίοι φιλόσοφοι και επιστήμονες για να δουν και να μά­ θουν από τις σχολές που υπήρχαν σε αυτήν. Ας δούμε όμως πιο αναλυτικά μερικά συνο­ πτικά στοιχεία από τη βιογραφία και το έργο τους (

Άτταλος ο Ρόδιος (4°ς αι π.Χ): Ο Άτταλος ο Ρόδιος ήταν αρχαίος γραμματικός, μαθηματικός και αστρονόμος, καταγόμενος από την Ρόδο. τί­ I)

ποτε δεν έχει διασωθεί από τυχόν έργα του. Μνη­ μονεύεται όμως το όνομά του σε υπόμνημα του Ιππάρχου περί των "Φαινομένων του Αράτου και του Ευδόξου". Στο σύγγραμμά του αυτό ο Ίππαρ­ χος αναφέρει διάφορα παραδείγματα επί των πα-

ραπάνω φαινομένων, αποσπάματα του οποίου έ­ χουν διασωθεί. 2) Γέμινος ή Γεμίνος ο Ρόδιος(llΟ-40 π.Χ). : Ο Γεμίνος έζησε και εργάστηκε στη Ρόδο στη σχο­ λή του Ποσειδωνίου, του οποίου υπήρξε μαθητής. Αναφέρεται ως Μαθηματικός, Αστρονόμος, Γε­ ωγράφος και Μετεωρολόγος. Το μοναδικό σωζό­ μενο έργο του Γεμίνου είναι η «Εiσαγωγή εiς τα Φαινόμενα», βασισμένο σε έργα παλαιότερων α­ στρονόμων που είχε σκοπό να διδάξει την επιστή­ μη αυτή σε αρχάριους όπου περιγράφει τη φαινό­ μενη κίνηση του ηλίου, τις σεληνοηλιακές περιο­ δικότητες, τις εκλείψεις, ότι η Γη είναι σφαιρική, η Σελήνη είναι ετερόφωτο σώμα, οι διαστάσεις της Γης είναι σημειακές (αμελητέες) σε σχέση με το μέγεθος του Σύμπαντος κ.α. Ο Γεμίνος συνέ­ γραψε και ένα σχόλιο επί του έργου του Ποσειδω­ νίου «Μετεωρολογικά». Σπαράγματα αυτού του σχολίου διασώθηκαν από τον Σιμπλίκιο στο σχό­ λιό του επί των «Φυσικών» του Αριστοτέλη. Το κυριότερο μαθηματικό έργο του Γεμίνου είναι το «Μαθηματικών Δόγμα>> (δηλ. θεωρία των Μαθηματικών). Παρότι αυτό το έργο δεν έχει σω­ θεί, αρκετά αποσπάσματά του υπάρχουν σε έργα των Πρόκλου, Ευτοκίου και άλλων. Ο Γεμίνος δι­ αχώρισε τα Μαθηματικά σε δύο μέρη : τα «νοητά» και τα «αισθητά», ή, όπως θα λέγαμε σήμερα, σε Καθαρά και Εφαρμοσμένα Μαθηματικά. Μακρά αποσπάσματα του έργου του διασώθηκαν και στο Σχόλιο των «Στοιχείων» του Ευκλείδη από τον Al­ Nay riz. Επίσης ο Γεμίνος έγραψε ιστορία των μα­ θηματικών συνεχίζοντας το έργο του Ευδήμου του Ροδίου. Πολλοί ερευνητές, ανάμεσα τους και ο Άγγλος φυσικός Derek de Solla Price, ισχυρίστη­ καν ότι ο Γεμίνος ήταν ο κατασκευαστής του Μη­ χανισμού των Αντικηθύρων, ίσως της σπουδαιό­ τερης τεχνολογικής κατασκευής του αρχαίου κό­ σμου, του πρώτου υπολογιστή που τον χρησιμο­ ποιούσαν για να υπολογίζουν τις κινήσεις των πλανητών και των αστέρων. Ονομάστηκε προς τι­ μή του ο κρατήρας Γέμινος (Geminus) στο Βόρειο ημισφαίριο της Σελήνης και υπάρχει αφιέρωμα του στο Τεχνολογικό Μουσείο Θεσσαλονίκης. 3) Εύδημος ο Ρόδιος (350-290 π.Χ): Ο Εύδημος ο Ρόδιος, ο πρώτος ιστορικός των μαθηματικών,

ΕΥΚΛΕΙΔΗΣ Β' 75 τ.3/1


------- Αρχαίοι Ρόδιοι Μαθηματικοί και Αστρονόμοι

υπήρξε ένας διακεκριμένος μαθητής του Αριστο­ τέλη, ο οποίος τον περιέβαλλε με μεγάλη εκτίμη­ ση. Λέγεται ότι ο μεγάλος φιλόσοφος βρέθηκε για αρκετό καιρό σε δίλημμα, σχετικά με το πρόβλημα της διαδοχής του στο Λύκειο είχε δυσκολία να ε­ πιλέξει μεταξύ του Θεοφράστου και του Ευδήμου, λόγω των πολλαπλών προσόντων τους. Τελικά ε­ πέλεξε τον Θεόφραστο, χωρίς φυσικά αυτή του η επιλογή να υποτιμά το έργο του Ευδήμου(«ο μεν Ρόδιος οίνος και αδρός, αλλ ' ο Λέσβιος γλυκύτε­ ρος»). Ο Εύδημος υπήρξε πολυγραφότατος. Έ­ γραψε έργα στην ιστορία των επιστημών, καθώς και στις θετικές και θεωρητικές επιστήμες. Γνωστά έργα του είναι τρία : Η ιστορία της αριθμητικής αναφερόμενη από τον Πορφύριο, η ιστορία της Γεωμετρίας αναφερόμενη από τον Πρόκλο, και η Αστρονομική ιστορία που μνημονεύεται από τον Διογένη το Λαέρτιο. Πολλοί σύγχρονοι ιστορικοί των επιστημών αποδίδουν στον Εύδημο διασκευές διαφόρων aριστοτελικών έργων. Είναι πιθανόν το έργο "Ευδήμια ηθικά" που αποδίδεται στον Αρι­ στοτέλη, να είναι έργο του Ευδήμου και να αποτε­ λεί περίληψη της διδασκαλίας περί ηθικής του Σταγειρίτη φιλοσόφου. Από το μεγάλο συγγραφικό έργο του Ευδήμου δεν έχει σωθεί σχεδόν τίποτε. Ευτυχώς ορισμένοι αρχαίοι συγγραφείς , Πρόκλος, Πάππος, Θέων ο Σμυρναίος, Σιμπλίκιος, Αιλιανός, Βοήθιος και Πορφύριος, διέσωσαν ορισμένα απο­ σπάσματα. Υπάρχει αφιέρωμα του στο Τεχνολογι­ κό Μουσείο Θεσσαλονίκης. 4) Ίππαρχος ο Ρόδιος (190 - 120 π.Χ.): Ο Ίππαρ­ χος ο Ρόδιος ή Ίππαρχος ο Νικαεύς ήταν Έλληνας aστρονόμος, γεωγράφος, χαρτογράφος και μαθη­ ματικός, θεωρούμενος από αρκετούς ως ο «πατέ­

ρας της Αστρονομίας».

-------

εντυπωσιάζουν. Ο Ίππαρχος γεννήθηκε στη Νίκαια της Βιθυνίας στη Μικρά Ασία γύρω στο 1 90 π.Χ� Το μεγαλύτερο διάστημα της ζωής του έζησε και εργάστηκε στη Ρόδο και την Αλεξάν­ δρεια Ανέπτυξε μαθηματικά μοντέλα για την κί­ νηση του Ηλίου και της Σελήνης, από παρατηρή­ σεις αιώνων αρχίζοντας από τους Χαλδαίους της Μεσοποταμίας. Υπήρξε επίσης ο πρώτος που συ­ νέταξε τριγωνομετρικό πίνακα, πράγμα που του επέτρεπε να επιλύει οποιοδήποτε τυχαίο τρίγωνο. Τα έξι κορυφαία πάντως επιτεύγματά του ήταν: Από τον 2ο αιώνα π.Χ. υπολόγισε πως το ηλιακό ή τροπικό έτος είναι 365,242 ημέρες, όταν σήμερα τα σύγχρονα ατομικά ρολόγια τον επιβεβαιώνουν υπολογίζοντάς το σε 365,242199 ημέρες! .. Η ανακάλυψη της μεταπτώσεως των ισημε­ ριών. Υπολόγισε τη διάμετρο της Σελήνης και τη κυμαινόμενη απόστασή της από τη Γη. " Η δημιουργία του πρώτου καταλόγου αστέρων, τουλάχιστον στο δυτικό κόσμο. Η επινόηση της κλίμακας των μεγεθών των αστέρων από τη μέτρηση της φωτεινότητάς των, που χρησιμοποιείται ακόμα και σήμερα από όλους τους aστρονόμους του κόσμου. (I

Το 1 34 π.Χ. ο Ίππαρχος ανακάλυψε ένα αστέ­ ρα που δεν υπήρχε πριν (πιθανόν κομήτη) στον αστερισμό του Σκορπιού, τότε διατύπωσε την αρχή της αστρονομίας ότι «ο αστέρες δεν είναι αιώνιοι στον ουρανό». Ο Ίππαρχος ο Ρόδιος έγραψε 17 γνωστούς τίτλους βιβλίων εκ των οποίων μόνο το «Περί των Αράτου και Ευδόξου φαινομένων - βι­ βλία τρία» διεσώθη. Υπάρχει αφιέρωμα του στο Τεχνολογικό Μουσείο Θεσσαλονίκης. ι

Ίππαρχος ο Ρόδιος(190 - 120 π.Χ) υπομονή του, η οξυδέρκειά του αλλά και το βεβαιούμενο ιστορικά πάθος του με ότι καταπια­ νόταν τον οδήγησαν σε δρόμους που σήμερα, ανα­ λογικά με τα δεδομένα της εποχής του, σίγουρα Η

Ίππαρχος

Λιθογραφία που αναπαριστά τον Ίππαρχο (από βιβλίο του 1 88 1 ).

ΕΥΚΛΕΙΔΗΣ Β' 75 τ.3/2


Αρχαίοι Ρόδιοι Μαθηματικοί και Αστρονόμοι

Παναίτιος ο Ρόδιος (185-109 π.Χ): Αρχαίος Έλληνας Μαθηματικός και Φιλόσοφος των στω­ ικών δογμάτων και μελετητής της πλατωνικής και aριστοτελικής φιλοσοφίας. Ασχολήθηκε περισσό­ τερο με την ερμηνεία των μαθηματικών εννοιών σε φιλοσοφική διάσταση. Γεννήθηκε στη Ρόδο και πέθανε στην Αθήνα. Υπήρξε μαθητής του Διο­ γένη του Βαβυλώνιου και του Αντίπατρου του Ταρσέα. αργότερα, όταν εγκαταστάθηκε στη Ρώ­ μη, μαθήτευσε κοντά στο Σκιπίωνα τον Αιμιλιανό και το Λέλιο. Θεωρείται ο πραγματικός ιδρυτής της "Μέσης Στοάς" και ως φιλόσοφος προσπάθησε να συγχωνεύσει τη φιλοσοφία των Ρωμαίων με τη στωική ηθική . Υποστήριξε την κυριαρχία της Ρώ­ μης ως έργο της θείας πρόνοιας, γεγονός που τον κατέταξε μεταξύ των ιδεολόγων του κύκλου του Σκιπίωνα και συντέλεσε ώστε να γίνει ο στωικι­ σμός η τυπική φιλοσοφία των δημοκρατικών και aριστοκρατικών φιλοσοφικών κέντρων της Ρώμης. Δάσκαλος του Ποσειδωνίου του Απαμεύς. Πο σειδών ιος ο Ρόδιος (135 .- 51 π.Χ.): Ο Πο­ σειδώνιος ο Ρόδιος ή ο Απαμεύς ήταν Έλληνας πολυμαθής Στωικός φιλόσοφος, aστρονόμος,

-------

Ο Ποσειδώνιος έκανε επίσης ένα υπολογισμό της διαμέτρου και της αποστάσεως της Σελήνης. Γνω­ ρίζουμε επίσης ότι είχε κατασκευαστεί στη σχολή του ένας « φορητός υπολογιστής» των κινήσεων των ουράνιων σωμάτων, παρόμοιο με τον Μηχανισμό των Αντικυθήρων όπως αυτός αποκα­ λύφθηκε μετά την πρόσφατη ανάλυση (20062008), και ίσως τον ίδιο τον μηχανισμό αυτό, που χρονολογείται στην ίδια περίπου περίοδο. Σύμφω­ να με τον Κικέρωνα (De Natura Deorum, Π 34), το φορητό «πλανητάριο» της σχολής του Ποσειδωνί­ ου έδειχνε τις ημερήσιες κινήσεις του Ηλίου, της Σελήνης και των 5 γνωστών τότε πλανητών. Εκτός από τα όσα έγραψε για τη Γεωμετρία, ο Ποσειδώ­ νιος πιστώνεται με τη διατύπωση-δημιουργία μα­ θηματικών ορισμών, ή τουλάχιστον για την ξεκά­ θαρη διατύπωση απόψεων για όρους όπως π.χ. «θεώρημα» και «πρόβλημα». Ο κρατήρας Ποσειδώνιος στο βόρειο ημισφαίριο της Σελήνης πήρε το όνομά του από τον Ποσειδώνιο τον Ρόδιο Υπάρχει αφιέρωμα του στο Τεχνολογικό Μουσείο Θεσσαλονίκης.

μαθηματικός γεωγράφος, πολιτικός, ιστορικός

και δάσκαλος που γεννήθηκε στην Απάμεια της Συρίας. Τον θεωρούσαν τον πολυμαθέστερο άν­ θρωπο του κόσμου για την εποχή του. τίποτα από το τεράστιο έργο του δεν έχει σωθεί ως ολότητα σήμερα, αλλά μόνο αποσπάσματα. Το μεγαλύτερο μέρος της ζωής του το έζησε στη Ρόδο και πέθανε είτε στη Ρόδο είτε στη Ρώμη . Και όχι μόνο έγινε πολίτης, αλλά συμμετείχε ενεργά στην πολιτική ζωή της Ρόδου, φθάνοντας στα ανώτερα αξιώματα ως ένας από τους πρυτάνεις της και υπηρέτησε εξάλλου ως πρεσβευτής της Ρόδου στη Ρώμη το 87 - 86 π.Χ. Τα πολλά έργα και οι διαλέξεις του Ποσειδωνίου του χάρισαν μια αυθεντία και φήμη παντού στον ελληνορωμαϊκό κόσμο, ενώ στη Ρόδο αναπτύχθηκε γύρω του μία σχολή . Παρότι λίγα πράγματα είναι γνωστά για την οργάνωση της σχολής του, είναι βέβαιο ότι ο Ποσειδώνιος είχε ένα σταθερό αριθμό Ελλήνων και Ρωμαίων φοιτητών. Αποσπάσματα από το αστρονομικό έργο του Ποσειδωνίου σώζονται μέσα από την πραγματεία του Κλεομήδη «Κυκλική θεωρία μετεώρων», όπου το πρώτο κεφάλαιο του δεύτερου βιβλίου φαίνεται ότι έχει στο μεγαλύτερο μέρος του αντιγραφεί από τον Ποσειδώνιο. Για τη διάμετρο του Ηλίου βρήκε μία τιμή μεγαλύτερη και ακριβέστερη από αυτές που πρότειναν άλλοι αρχαίοι Έλληνες αστρονόμοι.

Προτομή του Ποσειδωνίου (135 .- 51 π.Χ.) Μουσείο Νάπολης. 1. 2. 3. 4. 5. 6.

Βι!:iλωγι�υφ�κi:ς πηγf::

Θέματα από την ιστορία των Μαθηματικών, Γιάννη Χριστιανάκη, Πανεπιστημιακές Εκ­ δόσεις Κρήτης. Εισαγωγή εις τα φαινόμενα του Γεμίνου του Ροδίου, Ε. Σπανδάγου, Εκδόσεις Αίθρα. Ο Εύδημος ο Ρόδιος. Ε. Σπανδάγου, Εκδό­ σεις Αίθρα. Ιστοσελίδα του Τεχνολογικού Μουσείου Θεσσαλονίκης( διαθέσιμο στο www.tmth.edu.gr). Ηλεκτρονική Εγκυκλοπαίδεια βικιπαίδεια. Εγκυκλοπαίδεια Παύλου Δρανδάκη.

ΕΥΚΛΕΙΔΗΣ Β' 75 τ.3/3


Τα περίφημα "άλυτα" προβλήματα της αρχαιοτητας ,

Σωτή ρης Χ . Γκουντουβ άς

Τα τρία περίφημα "άλυτα" γεωμετρικά προβλήματα της αρχαιότητας εμφανίστηκαν περίπου στα μέσα του 5°υ π.Χ αιώνα στην αρχαία Ελλάδα. Είναι προβλήματα γεωμετρικών κατασκευών που πρέπει να πραγματοποιηθούν με αποκλειστική χρήση κανόνα και διαβήτη. Αυτά είναι ο δ ιπλασιασμός του κύβου ή Δή λιο πρόβ λημα, ο τετραγωνισμός του κύκλου, και η τριχοτό­ μηση γωνίας.

Το πρόβλημα του διπ λασιασμού του κύβου είναι το εξής: Να κατασκευαστεί κύβος με όγκο διπλάσιο από τον όγκο δοθέντος κύβου ακμής α.

Ζητείται λοιπόν, να κατασκευαστεί η ακμή χ ενός κύβου που θα έχει διπλάσιο όγκο από κύβο δοθείσης ακμής α. Δηλαδή να κατασκευαστεί το τμήμα x=αifi. Το πρόβλημα είναι αδύνατο να επιλυθεί με κανόνα και διαβήτη όπως αποδείχθηκε το 1 83 7 με τη βοήθεια της θεωρίας Galois. Παρόλα αυτά οι αρχαίοι Έλληνες Γεωμέτρες έδωσαν πολλές λύσεις στο πρόβλημα με άλλα μέσα κατασκευής. Ο Ιπποκρ άτης ο Χίος (�470-400 π.Χ) πρώτος ανήγαγε το πρόβλημα στην παρεμβολή δύο μέσων αναλόγων χ, y μεταξύ των τμημάτων α και 2α. Το πρόβλημα τότε ικανοποιεί τις συνεχείς α αναλογίες � _)'_ , που οδηγούν στη ζητούμενη σχέση χ=α if2 . χ y 2α Ο Ε ρατοσ θένης ο Κυρηναίος (276-1 94 π.Χ) σε επιστολή που έστειλε στον βασιλιά του Ελ­ ληνιστικού κράτους της Αιγύπτου Πτολεμαίο Γ' τον Ευεργέτη (284-222 π.Χ) αναφέρει την ι­ στορία του προβλήματος, διάφορες λύσεις που έχουν δοθεί και τη δική του λύση της κατα­ σκευής δύο μέσων αναλόγων με τη βοήθεια του μεσολάβου, ενός οργάνου που επενόησε γι' αυ­ τό το σκοπό. Ο Ευτόκιος (6°ς μ.Χ αι.) στα σχόλια του για την πραγματεία του Αρχιμήδη "Περί σφαίρας και κυλίνδρου" αναφέρει 1 2 λύσεις του προβλήματος. Λύσεις στο πρόβλημα εκτός του Ιπποκρά­ τη έδωσαν οι : Αρχύτας ο Ταραντίνος (428-365 π.Χ) με ημικυλίνδρους Πλάτων (427-347 π.Χ) με παρεμβολή δύο μέσων αναλόγων Εύδοξος ο Κνίδιος (407-354 π.Χ) με την "καμπύλη του Ευδόξου" Μέναιχμος (� 350 π.Χ) με δύο παραβολές Αρχιμήδης (287-21 2 π.Χ) με δύο κωνικές τομές, παραβολή και υπερβολή Ερατοσθένης με το μεσολάβο Απολλώνιος (�270- 1 90 π.Χ) με κύκλο και ισοσκελή υπερβολή Νικομήδης (�200 π.Χ) με την κογχοειδή καμπύλη Ήρων (�200 π.Χ) με νεύση Κ R Διοκλής (�1 °ς π.Χ αι.) με την κισσοειδή καμπύλη Πάππος (�3 °ς μ.Χ αι.) με νεύση. Θα δώσουμε τώρα τη λύση που αποδίδεται στον Πλάτωνα. Έστω δύο κάθετοι άξονες που τέμνονται στο Ο. Στον ένα άξονα παίρνουμε τμήμα ΟΑ=α και Λ στον άλλο τμήμα ΟΒ=2α. Φέρουμε τώρα τις ΑΚ//ΒΛ έτσι ώστε ΑΚJ..ΚΛ και ΒΛ_lΚΛ.Το ζητούμενο τμήμα χ είναι το ΟΚ. ΟΑ ΟΚ , Στο ορθογωνιο ΑΚΛ: ΟΚ2 = ΟΑ·ΟΛ<::::>ΟΚ ΟΛ

= =

=-

ΕΥΚΛΕΙΔΗΣ Β' 75 τ.3/4


------- Τα περίφημα «άλυτα» προβλήματα της αρχαιότητας -------

ΟΚ ΟΛ , ΒΚΛ: ΟΛ2 = ΟΚ . ΟΒ <=>- = Στο ορθογωνιο ΟΛ ΟΒ ΟΑ ΟΚ ΟΛ α χ , το ζητουμενο , τμημα χ - ΟΚ- α νr;::;2L . Άρα -- = -- -<=>- = = Υ . Οποτε ΟΚ ΟΑ ΟΒ χ y 2α Η τεθλασμένη γραμμή ΑΚΛΒ με ορθές γωνίες στα Κ και Λ μπορεί να επιτευχθεί με ειδικό όρ­ γανο, όπου σε ένα μεταβλητό ορθογώνιο πλαίσιο ολισθαίνει μια πλευρά κάθετα σε δύο άλλες που είναι σταθερές. Το πρόβλημα του τετραγωνισμού του κύκλου είναι το εξής: -.

=

-

-

,

_

3

Να κατασκευαστεί τετράγωνο με εμβαδόν ίσο με δοθέντα κύκλο ακτίνας α.

Ζητείται λοιπόν, να κατασκευαστεί η πλευρά χ ενός τετραγώνου που θα έχει ίσο εμβαδόν με κύκλο ακτίνας α. Δηλαδή να κατασκευαστεί το τμήμα χ=α ,J;.. Ο πρώτος που αναφέρεται ότι α­ σχολήθηκε με το πρόβλημα είναι ο Αναξαγόρας ο Κλαζομένιος (500-428 π.Χ). Το αδύνατο της κατασκευής με κανόνα και διαβήτη αποδείχθηκε το 1 882 από τον C.F.Von Lindemann.

Ο Ιάμβλιχος (3 °ς μ.Χ αι.) αναφέρει ότι το πρόβλημα έλυσαν οι: Αρχιμήδης με την έλικα Νικομήδης με την τετραγωνίζουσα του Ιππία Απολλώνιος με την κοχλιοειδή Κάρπος με την καμπύλη "εκ διπλής κινήσεως προερχόμενη". Επίσης ο Πάππος στη Συναγωγή του δίνει μια λύση με την Δr-� r_ι τετραγωνίζουσα του Ιππία και την αποδίδει στο Δεινόστρατο ( 350 π.Χ) αδελφό του Μέναιχμου. Στη συνέχεια θα δούμε τη λύση αυτή. Η τετραγωνίζουσα είναι καμπύλη που παράγεται από δύο ομαλές κινήσεις, μια ευθύγραμμη και μια κυκλική κίνηση. Στο α παρακάτω τετράγωνο ΟΑΓΔ θεωρούμε την πλευρά ΔΓ να κινείται ομαλά προς τα κάτω παράλληλα προς τον εαυτό της και την ΟΔ να στρέφεται ομαλά γύρω από το Ο, μέχρι να συμπέσει με την ΟΑ σε ίσο χρόνο με την ΔΓ. Η τροχιά του σημείου τομής των ΔΓ και ΟΔ, ο Ε κάθε στιγμή της κίνησής τους παράγει την τετραγωνίζουσα. Στο σχήμα το τόξο ΔΕ είναι η τετραγωνίζουσα. Στο σχήμα λοιπόν έχουμε την τετραγωνίζουσα ΔΕ και ένα τεταρτοκύκλιο ΑΔ. Έστω •

__

q το μήκος του τόξου ΑΔ. Ο Πάππος αποδεικνύει τη σχέση :

�r

__

Α

_g_ = � .

α ΟΕ Από τη σχέση αυτή προκύπτει ότι το μήκος q του τεταρτοκυκλίου ΑΔ προκύπτει σαν τέταρ­ τη ανάλογος των τμημάτων α, α και ΟΕ. Μπορούμε τώρα να κατασκευάσουμε ένα ευθύγραμμο τμήμα με μήκος 4 q, δηλαδή ίσο με το μήκος του κύκλου. Χρησιμοποιούμε τώρα την πρόταση που απέδειξε ο Αρχιμήδης στο έργο του ''Κύκλου μέ­ τρη σις" ότι το εμβαδόν ενός κύκλου είναι ί σο με το εμβαδόν ορθογωνίου τριγώνου με δύο κάθετες πλευρές ί σες με το μήκος του κύκλου και την ακτίνα αντίστοιχα.

Για να τετραγωνίσουμε τον κύκλο κάνουμε τα παρακάτω βήματα: Κατασκευάζουμε ένα ορθογώνιο τρίγωνο με κάθετες πλευρές ΑΓ=rκαι AB=4 q. Παίρνουμε το μέσο Μ της ΑΒ και σχηματίζουμε το ορθογώνιο ΑΜΔΓ του οποίου το εμβαδόν είναι ίσο με του τριγώνου ΑΒΓ. Προεκτείνουμε την ΓΔ και παίρνουμε τμήμα ΔΕ=ΔΜ Κατασκευάζουμε το ημικύκλιο με διάμετρο την ΓΕ Στο Δ φέρουμε κάθετη στην ΓΕ που τέμνει το ημικύκλιο στο Ζ. Το τμήμα ΔΖ είναι η ζη­ τούμενη πλευρά χ του τετραγώνου. Άρα το εμβαδόν του τετραγώνου ΔΖΗΘ ισούται με το εμβαδόν του δοθέντος κύκλου. •

• •

ΕΥΚΛΕΙΔΗΣ Β' 75 τ.3/5


------- Τα περίφημα «άλυτα» προβλήματα της αρχαιότητας -------

2πr ΑΒ 2 = ΓΔ·ΔΕ = ΓΔ·ΔΜ =-ΑΓ =-r=πr. 2 2 Η τετραγωνίζουσα από μερικούς συγγραφείς αναφέρεται ως τετραγω­ νίζουσα του Δεινόστρατου. Όμως ο Ιππίας ο Ηλείος είναι αρχαιότερος του Δεινόστρατου και με τη βοήθεια της τετραγωνίζουσας είχε τριχοτο­ μήσει την γωνία. Για το λόγο αυτό οι περισσότεροι συγγραφείς την αποδί­ δουν στον Ιππία. Επίσης με τη βοή­ Μ Α 2πr= 4q θεια της τετραγωνίζουσας μπορούμε να διαιρέσουμε μια γωνία σε όσα ίσα Έμβαδόν κύκλου = Έμβαδον ορθογωνίου τριγώνου μέρη θέλουμε. Η εξίσωση της τετραγωνίζουσας σε Πράγματι:

χ

2

καρτεσιανή μορφή είναι :

Β ΑΒΓ

χ

Στην τριχοτόμηση γωνίας ζητείται να κατασκευαστεί γωνία ίση με το 1 /3 δοθείσης τυχαίας γωνίας. Το αδύνατο της κατασκευής (με κανόνα και διαβήτη) αποδείχθηκε το 1 83 7 . Πρέπει εδώ να σημειώσουμε ότι για ορισμένες γωνίες η τριχοτόμηση με κανόνα και διαβήτη είναι εφικτή. Για παράδειγμα η ορθή γωνία τριχοτομείται. Για να τριχοτομηθεί η ορθή γωνία πρέπει να κατα­ σκευαστεί μια γωνία 3 0°. Αυτό μπορεί να γίνει με την εγγραφή σε κύκλο ενός κανονικού δωδε­ καγώνου. Τότε η κεντρική του γωνία είναι 3 0°. Γενικά μπορεί να αποδειχθεί ότι οι μόνες ακέ­ ραιες γωνίες που κατασκευάζονται με κανόνα και διαβήτη είναι πολλαπλάσια του 3 . Λύσεις του προβλήματος με άλλα μέσα κατασκευής έδωσαν οι: ° Ψ Ιππίας ο Ηλείος (5 ς π.Χ αι.) με την τετραγωνίζουσα : Αρχιμήδης δύο λύσεις με νεύση και την έλικα του : Νικομήδης με την κογχοειδή, : Πάππος με νεύση. Επίσης λύσεις έδωσαν μετά την αρχαιότητα και οι Pascal ( 1 650), Ceva (1699), Mac Laurin ( 1 742), Delenges ( 1 7 8 3 ), Plateau ( 1 826), Longchamps (1888) και Pergassi ( 1 893 ) . Θα δώσουμε τώρα τη λύση με νεύση που αποδίδεται στον Αρχιμήδη. ••

••

••

Έστω η προς τριχοτόμηση γωνία ΑΟΒ. Α Γράφουμε κύκλο με κέντρο Ο και τυχαία ακτίνα. Προεκτείνουμε τη I ΒΟ και φέρνουμε την ευθεία ΑΔ έτσι ώστε ΟΑ=ΓΔ. Τότε η γωνία Δ ο Β\ ΟΔΓ θα είναι το 1/3 της ΑΟΒ. Η απόδειξη στηρίζεται στα δύο ισοσκελή τρίγωνα ΑΟΓ και ΟΓΔ και στις εξωτερικές γωνίες τους. Η κατασκευή (νεύση) της ευθείας ΑΓΔ με την ιδιότητα ΟΑ=ΓΔ μπορεί να γίνει με ένα ειδι­ κό όργανο που επινοήθηκε από τον Αρχιμήδη. Κλείνοντας αυτό το άρθρο μπορούμε να θυμηθούμε τη ρήση του David Hilbert από την πε­ ρίφημη ομιλία του στο 2° Διεθνές συνέδριο των μαθηματικών στο Παρίσι το 1 900. /

I

«Αλλά πόσο σφρίγος, ζωτικής σημασίας γι α τη μαθηματική επι στήμη, θα χάνονταν με το ξερί­ ζωμα της γεωμετρίας και της μαθηματικής φυσι κής.»

ΕΥΚΛΕΙΔΗΣ Β' 75 τ.3/6


,αθηματtκοi Δtαyωvt σμοi αθηματtκές Ολυμπιάδες

Ε πιτροπή Διαγ ωνισμ ών της Ε . Μ. Ε .

70ος ΠΑΝΕΛΛΗΝΙΟΣ ΜΑΘΗΤΙΚΟΣ ΔΙΑΓΩΝΙΣΜΟΣ ΣΤΑ ΜΑΘΗΜΑΤΙΚΆ 'Ό ΕΥΚΛΕΙΔΗΣ" ΣΑΒΒΑΤΟ, 23 ΙΑΝΟΥΑΡΙΟΥ 2010 ΕΝΔΕΙΚΤΙΚΕΣ ΛΥΣΕΙΣ Α' Λυκείου Π ρ όβλη μα I

Να βρείτε τις τιμές του ρητού αριθμού α, για τις οποίες ο αριθμός Α=αJ3 είναι ρητός. (ii) Να αποδείξετε ότι ο αριθμός Β= ( 1 + J3 Y είναι άρρητος. (i)

Λίι ση

(i) Για α = Ο είναι Α = Ο, ρητός . 'Εστω α ::F Ο . Αν ήταν ο Α = αJ3 ρητός, τότε ο αριθμός

Α = J3, θα

α ήταν επίσης ρητός, ως πηλίκο δύο ρητών αριθμών, που είναι άτοπο. Επομένως, ο αριθμός Α είναι ρητός μόνο για α= Ο . 2 (ii) ' Ε χουμε Β = ( Ι + J3) = 4 + 2 J3 . Αν ο αριθμός Β ήταν ρητός, τότε ο αριθμός Β - 4 = 2 J3 θα ήταν επίσης ρητός, ως διαφορά δύο ρητών, το οποίο είναι άτοπο, σύμφωνα με το (ί) . Π ρ όβλη μα

2

Να αποδείξετε ότι η εξίσωση χ+ 1 - 2lxl =αχ, έχει, για κάθε τιμή της παραμέτρου αΕ JR, μία του­ λάχιστον πραγματική λύση. Για ποιες τιμές του α η εξίσωση έχει δύο διαφορετικές μεταξύ τους πραγματικές λύσεις; Λίι σ η

Επειδή στην εξίσωση εμφανίζεται η απόλυτη τιμή του αγνώστου χ διακρίνουμε δύο περιπτώσεις: (i) Έστω χ �Ο . Τότε ισχύει lxl = χ και η δεδομένη εξίσωση είναι ισοδύναμη με το σύστημα:

{ {

Ι χ = - , αν α> - 1 χ + Ι- 2χ = αχ, χ �Ο <::::> ( α+ Ι) χ = Ι, χ �Ο <::::> α+ Ι αδύνατο, αν α � - 1 . (ii) Έστω χ < Ο . Τότε ισχύει lxl = -χ και η δεδομένη εξίσωση είναι ισοδύναμη με το σύστημα:

-

χ = Ι , αν α< 3 χ + 1 + 2χ = αχ, χ < Ο <::::> ( α- 3 ) χ = Ι, χ < Ο <::> α-3 αδύνατο, αν α�3 . Επομένως, για κάθε τιμή της παραμέτρου αΕ JR, η εξίσωση έχει μία τουλάχιστον πραγματική λύση . Η εξίσωση έχει 2 πραγματικές λύσεις διαφορετικές μεταξύ τους, αν ισχύει: - 1 < α < 3 . 1 1 Πράγματι, για -1 < α< 3 η εξίσωση έχει τις λύσεις χ = - < Ο και χ = -- > Ο που είναι διαφορε2 α- 3 α+ l τικές μεταξύ τους. 1

-

ΕΥΚΛΕΙΔΗΣ Β' 75 τ.3/7

--


-----11 ρ611λημυ

Μαθηματικοί Διαγωνισμοί- Μαθηματικές Ολυμπιάδες

J

-------

Δίνεται τρίγωνο ABC εγγεγραμμένο σε κύκλο C(O, R) και έστω A1 ,B1 ,C1 τα aντιδιαμετρικά σημεία των κορυφών του A,B,C . Στις ευθείες που ορίζουν οι πλευρές BC, AC, AB θεωρούμε τα σημεία A 2 ,B 2 ,C 2 αντίστοιχα και έστω (ε1 ) η ευθεία που ορίζουν τα σημεία Α1 , Α 2 , (ε 2 ) η ευθεία που ορίζουν τα σημεία Β1 , Β 2 και (ε 3 ) η ευθεία που ορίζουν τα σημεία C1 ,C 2 • Έστω ακόμη (δ 1 ) η παράλληλη ευθεία που φέρουμε από το σημείο Α προς την (ε 1 ) , (δ 2 ) η πα­ ράλληλη ευθεία που φέρουμε από το σημείο Β προς την (ε 2 ) και (δ 3 ) η παράλληλη ευθεία που φέ­ ρουμε από το σημείο C προς την (ε 3 ) • Ν α αποδείξετε ότι οι ευθείες (ε1 ) , (ε 2 ) και (ε 3 ) συντρέχουν (περνάνε από το ίδιο σημείο), αν, και μόνο αν, οι ευθείες (δ 1 ) , (δ 2 ) και (δ 3 ) συντρέχουν ΛίJση

Α Οι ευθείες (ε ι ) και (δ ι ) είναι συμμετρικές ως προς το κέντρο Ο του περιγεγραμμένου κύ­ κλου του τριγώνου ABC , αφού το Ο είναι μέ­ σο της ΑΑ ι . Οι ευθείες ( ε 2 ) και (δ 2 ) είναι συμμετρικές ως προς το κέντρο Ο του περιγεγραμμένου κύ­ κλου του τριγώνου ABC , αφού το Ο είναι μέ­ σο της ΒΒ ι . Οι ευθείες ( ε 3 ) και (δ 3 ) είναι συμμετρικές ως προς το κέντρο Ο του περιγεγραμμένου κύ­ Β\ κλου του τριγώνου ABC , αφού το Ο είναι μέ­ σο της cc ι . Σύμφωνα με τη θεωρία, αν περιστρέψουμε μία ευθεία κατά 1 80° γύρω από το κέντρο συμ­ μετρίας, τότε αυτή θα συμπέσει με τη συμμετρι­ κή της ευθεία , ως προς κέντρο το σημείο Ο . Σχήμα 3 Επομένως, οι ευθείες (ει ) , (ε 2 ) και (ε 3 ) συντρέχουν, έστω στο σημείο Κ , αν, και μόνο αν, οι ευθείες (δι ), (δ 2 ) και (δ 3 ) συντρέχουν στο σημείο Κ' , που είναι το συμμετρικό του σημείου Κ ως προς το σημείο Ο .

nuραη1ρηση

Το σημείο Κ ταυτίζεται με το ορθόκεντρο του τριγώνου ABC , αν, και μόνο αν, τα σημεία A 2 , B 2 , C 2 είναι τα μέσα των πλευρών BC, AC, AB αντίστοιχα. Στη περίπτωση αυτή μπορούμε να χρησιμοποιήσουμε τη γνωστή πρόταση : "Τα συμμετρικά του ορθοκέντρου ως προς τα μέσα των πλευρών τριγώνου, βρίσκονται επάνω στο περι­ γεγραμμένο του κύκλο και είναι aντιδιαμετρικά των κορυφών του" Π ρ6/1λημ Η 4

Οι πραγματικοί αριθμοί χ, y και z ικανοποιούν τις ισότητες: χ 3 - y 3 26z3 x 2 y - xy 2 6z3 • (u) Να εκφράσετε τους x,y συναρτήσει του z . (β) Αν επιπλέον ισχύει ότι χ + 2y + 3z 8 , να βρείτε τους πραγματικούς αριθμούς x,y και z . =

=

=

Λί1ση

Πολλαπλασιάζουμε την δεύτερη ισότητα επί 3 και την αφαιρούμε από την πρώτη, οπότε λαμβάνουμε 3 ( I) ( χ - y ) 8z 3 <::> χ - y 2z . (2) 2zxy 6z 3 , Τότε η δεύτερη ισότητα γίνεται: οπότε διακρίνουμε δύο περιπτώσεις: =

=

ΕΥΚΛΕΙΔΗΣ Β' 75 τ.3/8

=


------

Μαθηματικοί Διαγωνισμοί- Μαθηματικές Ολυμπιάδες

-------

(3) z ::�; Ο. Τότε η (2) είναι ισοδύναμη με την σχέση 2 xy = 3z2 , Από τις ( I ) και (3) , προκύπτει η σχέση χ ( χ-2z ) = 3z2 <=:> χ -2zx-3z 2 = Ο <=:> χ = 3z ή χ = -z , οπότε θα είναι χ = 3z, y = ή χ = -z,y = -3z . } {( χ -yxy)( χ( x-y+ xy) +=0y2 ) = ο <=> χ-y = Ο ή χ = y = Ο, Για z = Ο οι δύο πρώτες εξισώσεις γίνονται: οπότε προκύπτει ότι: χ = y, ανεξάρτητα από το z . (β) Για χ = 3z,y = z η εξίσωση χ+ + 3z =8 γίνεται 8z =8 z = 1 , οπότε έχουμε ότι ( x , y, z ) = ( 3 , 1 , 1 ) , ενώ για χ =-z, y=-3z , η εξίσωση γίνεται -4z =8 <=> z = -2, οπότε έχουμε ότι ( x , y , z ) = ( 2, 6 , -2 ) . Για z = Ο, είναι χ = y , οπότε από την εξίσωση χ+ 2y + 3z =8 προκύπτει ότι ( x , y , z ) = ( � , � , 0 ) . (i) Έστω

z

(ii)

2

<=:>

2y

Πρ6βλημα

1

Β' Λυκείου

Να προσδ ιορ ίσετε όλες τ ις τριάδες ( x,y, z) πραγματικών αριθμών που είναι λύσεις του συστή­ ματος: χ 3 + y 3 = 65z3 x 2 y + xy 2 = 20z3 x - y + 2z = 10 . .\\)ση

3 3 <=> ( 1) ( χ + y = 125z χ + y = 5z . 5zxy = 0z , (3) xy = 4z2 • z ::�; Ο . (1) (3), χ ( 5z-χ ) = 4z2 <=:> χ 2 -5zx + 4z2 = Ο <=:> χ = 4z χ = z , χ = 4z, y = z χ = z, y = 4z . χ = 4z, y = z 5z = 1 Ο <=:> z = , χ = z, y = 4z -z = 10 <=:> z = -10, ( χ , y, z ) = (8, ( x , y, z ) = ( -10,-40,-10 ) . y2 ) = Ο} <=>χ +y =0 ή χ= y=O <=>χ =-y , {( χ+ yxy)( χ( χ2 -xy+ Για z = Ο οι δύο πρώτες εξισώσεις γίνονται: +y) =0 οπότε από την τρίτη εξίσωση προκύπτει ότι ( χ , y , z ) = ( 5 , -5 , Ο) . Πολλαπλασιάζουμε την δεύτερη εξίσωση επί και την προσθέτουμε στην πρώτη, οπότε λαμβάνουμε την γ εξίσωση (2) 2 3 Τότε η δεύτερη εξίσωση γίνεται οπότε διακρίνουμε δύο περιπτώσεις: (ί) Έστω Τότε από την εξίσωση (2) λαμβάνουμε: Από τις και προκύπτει η εξίσωση ή οπότε θα είναι ή Για η τρίτη εξίσωση του συστήματος γίνεται 2 οπότε το σύστημα έχει τη λύση 2, 2 ) , ενώ για η τρίτη εξίσωση γίνεται οπότε το σύστημα έχει τη λύση (ii)

Πρόβλημο 2

Δίνεται οξυγώνιο και σκαληνό τρίγωνο ABC , Κ τυχόν σημείο στο εσωτερικό του και τα ύψη του AH 1 ,BH 2 ,C H3 • Ο περιγεγραμμένος κύκλος του τριγώνου ΑΗ 2 Η 3 τέμνει την ημιευθεία ΑΚ στο σημείο Κ 1 , ο περιγεγραμμένος κύκλος του τριγώνου ΒΗ 1 Η 3 τέμνει την ημιευθεία ΒΚ στο σημείο Κ 2 και ο περιγεγραμμένος κύκλος του τριγώνου C H 1 Η 2 τέμνει τη ημιευθεία C K στο σημείο Κ 3 • Να aποδείξτε ότι τα σημεία Κ 1 ,Κ 2 ,Κ 3 , Η και Κ είναι ομοκυκλικά ( δηλαδή ανήκουν στον ίδιο κύ­ κλο), όπου Η είναι το ορθόκεντρο του τριγώνου ABC . . \\)ση

)

2 , (c 2 ) Η • 2

Έστω ( c ο περιγεγραμμένος κύκλος του τριγώνου ΑΗ Η 3 ο περιγεγραμμένος κύκλος του τριγώ­ νου ΒΗ 1 Η 3 και (c 3 ) ο περιγεγραμμένος κύκλος του τριγώνου CH1 1

ΕΥΚΛΕΙΔΗΣ Β' 75 τ.3/9


------ Μαθηματικοί Διαγωνισμοί- Μαθηματικές Ολυμπιάδες -------

Το τετράπλευρο ΑΗ2 ΗΗ3 εί­ ναι εγγράψιμο, οπότε ο κύ­ κλος ( c 1 ) περνάει από το ση­ μείο Η . Το τετράπλευρο ΒΗ1 ΗΗ3 εί­ ναι εγγράψιμο, οπότε ο κύ­ κλος ( c 2 ) περνάει από το ση­ μείο Η . Το τετράπλευρο CH1 HH2 εί­ ναι εγγράψιμο, οπότε ο κύ­ κλος ( c 3 ) περνάει από το ση­ μείο Η . Τελικά, οι τρεις κύκλοι (C1 ) , (c 2 ) και (c 3 ) περνάνε από το ορθόκεντρο Η του τριγώνου

Σχήμα 4

ABC . Ο κύκλος (c 1 ) έχει διάμετρο την ΑΗ , οπότε ΗΚ 1 .l ΑΚ1 , δηλαδή το σημείο Κ1 ανήκει στο κύκλο δια­ μέτρου ΗΚ Όμοια αποδεικνύουμε ότι και τα σημεία Κ 2 , Κ 3 , ανήκουν στον ίδιο κύκλο. .

Να αποδείξετε ότι η εξίσωση χ 2 + χ + 1 - 2lxl = αχ, α Ε JR, έχει για κάθε α Ε JR δύο διαφορετικές μεταξύ τους λύσεις στο σύνολο JR . Για ποιες τιμές του α οι δύο ρίζες είναι ετερόσημες;

lxl ,

Λόγω της ύπαρξης του διακρίνουμε δύο περιπτώσεις: (ί) Έστω χ � Ο . (1) χ 2 + χ + 1 - 2χ = αχ, α Ε JR <=> χ 2 ( α + 1 ) χ + 1 = Ο, α Ε JR, Τότε η εξίσωση γίνεται: η οποία έχει διακρίνουσα Δ = ( α + Ι )2 4 = ( α - 1 )( α + 3) . Άρα η εξίσωση ( Ι ) έχει πραγματικές ρίζες, όταν είναι α::::; -3 ή α � 1 . Επειδή το γινόμενο των ριζών είναι Ρ = 1 > Ο οι ρίζες είναι ομόσημες, οπότε για να είναι και οι δύο θετικές πρέπει και αρκεί S = α + 1 > Ο <=> α > - 1 . Επομένως έχουμε: Για α > 1 , η εξίσωση ( 1 ) έχει δύο ακριβώς διαφορετικές θετικές ρίζες στο JR Για α = 1 , η εξίσωση ( 1 ) έχει τη διπλή θετική ρίζα χ = 1 στο JR . Για α < 1 , η εξίσωση ( 1 ) δεν έχει μη αρνητικές ρίζες στο JR . (ii) Έστω χ < Ο . ( 2) Τότε η εξίσωση γίνεται: χ 2 + χ + 1 + 2χ = αχ , α Ε JR <=> χ 2 - ( α - 3) χ + 1 = Ο, α Ε JR , -

-

.

η οποία έχει διακρίνουσα Δ = ( α - 3 )2 -4 = ( α - 5) ( α - 1 ) . Άρα η εξίσωση (2 ) έχει πραγματικές ρίζες όταν είναι α::::; 1 ή α � 5. Επειδή το γινόμενο των ριζών είναι Ρ = 1 > Ο οι ρίζες είναι ομόσημες, οπότε για να είναι και οι δύο αρνητικές πρέπει και αρκεί S = α - 3 < Ο <=> α < 3 . Επομένως έχουμε: Για α < 1 , η εξίσωση ( 2 ) έχει δύο ακριβώς διαφορετικές αρνητικές ρίζες στο JR Για α = 1 , η εξίσωση (2) έχει τη διπλή αρνητική ρίζα χ = - 1 στο JR . Για α > 1 , η εξίσωση ( 2 ) δεν έχει αρνητικές ρίζες στο JR . Από τις περιπτώσεις ( 1 ) και ( 2 ) προκύπτει ότι η δεδομένη εξίσωση έχει, για κάθε α Ε JR , δύο πραγματι­ κές ρίζες διαφορετικές μεταξύ τους, οι οποίες είναι ετερόσημες για α = 1 . ο

μυ -'

Να λύσετε στους πραγματικούς αριθμούς την εξίσωση .J2x 2 + 3χ + 2 - z.Jx 2 + χ + 1 = χ + 1 . ΕΥΚΛΕΙΔΗΣ Β' 75 τ.3/10


------

Μαθηματικοί Διαγωνισμοί- Μαθηματικές Ολυμπιάδες

-------

. 1'!j�)'η

2χ 2 + 3χ + 2 > Ο και χ 2 + χ + 1 > Ο, για κάθε χ ε IR . Αν θέσουμε a = �2χ 2 + 3χ + 2, b = �χ 2 + χ + 1 , χ ε IR , τότε λαμβάνουμε: a 2 - b 2 = ( 2χ 2 + 3χ + 2 ) - ( χ 2 + χ + 1 ) = χ 2 + 2χ + I ( χ + I ) 2 , οπότε από τη δεδομένη εξίσωση προκύπτει η εξίσωση με αγνώστους a, b , a 2 - b 2 = ( a - 2b ) 2 <=> 4ab - 5b 2 = Ο <=> b ( 4a - 5 b ) = Ο <=> 4a 5b, αφού είναι b 1= Ο . ' Ετσι έχουμε την εξίσωση 4 �2χ 2 + 3χ + 2 5 �χ 2 + χ + 1 , της οποίας τα δύο μέλη είναι θετικά, για κάθε χ ε IR , οπότε είναι ισοδύναμη με την εξίσωση 1 6 · ( 2χ 2 + 3χ + 2 ) = 25 · ( χ -? + χ + 1 ) <::> 7χ 2 + 23χ + 7 = 0 <=> χ = -23±3 J37 . 14 J37 Μ ' η μοναδ ικη' ρι-' ' την εξ ισωση, ' ' ' η τιμη' χ = -23 + 3 δ εν επαλη θ ευει ε ε' λεγχο δ ιαπιστωνουμε οτι οποτε 14 ' η χ = -23 - 3 J37 . Α υτο' θ α μπορουσε ' a - 2b < Ο η οποια' ' και απο' τη σχεση ' να προκυψει ζα της ειναι 14 αληθεύει για κάθε χ ε IR , οπότε πρέπει να είναι χ < -1 . Κατ' αρχή παρατηρούμε ότι ισχύει:

=

=

=

ακολουθία a., η ε Ν*, ορίζεται αναδρομικά από τις σχέσεις a.+ 1 a. + kn, η ε Ν*, όπου k θετικός ακέραιος και a 1 1 . Να βρείτε για ποια τιμή του k ο αριθμός 2011 είναι όρος της ακολουθίας a. ,η ε Ν*. Η

=

=

Από τη δεδομένη αναδρομική σχέση έχουμε

a1 = 1 a2 = a 1 + k a 3 = a 2 + 2k

a n-1 = a n-2 + ( η - 2 ) k a. = a n-1 + ( η - 1 ) k

από τις οποίες μ ε πρόσθεση κατά μέλη λαμβάνουμε

an =l+k ( l+ 2 + . + η - 1 ) = 1 + k ( 1 + 2 + ... + η - 1 ) = 1 + k ( η - 1 ) η . 2 Επομένως, αρκεί να προσδιορίσουμε τις τιμές των k και η για τις οποίες ισχύει η ισότητα: a. 1 + k ( η - 1 ) η 20 1 1 <=> k ( η - l ) η = 4020 <=> k ( η - l ) η = 1 · 2 · 2 · 3 · 5 · 67 2 <=> ( η, k ) = ( 2, 20 1 0 ) ή ( η, k ) = ( 3 , 670 ) ή ( η, k ) = ( 4 , 335 ) ή ( η,k ) = ( 5 , 20 1 ) ή ( η, k ) = ( 6, 1 34 ) Επομένως, για k = 201 Ο είναι a 2 = 201 1 , για k = 670 είναι a 3 = 201 1 , για k = 335 είναι a 4 = 201 1 , για k = 201 είναι a5 = 201 1 και για k = 1 34 είναι a 6 = 201 1 . . .

=

=

Δίνεται οξυγώνιο και σκαληνό τρίγωνο ABC και έστω Μ 1 , Μ 2 , Μ 3 τυχόντα σημεία των πλευρών του ΒC, ΑC, ΑΒ , αντίστοιχα. Έστω ακόμη τα ύψη του AH1 ,BH2 ,CH3 • Να αποδείξετε ότι οι περιγε­ γραμμένοι κύκλοι των τριγώνων ΑΗ 2 Η 3 , ΒΜ1 Η 3 , CM1H2 περνάνε από το ίδιο σημείο (έστω Κ1 ), οι ΕΥΚΛΕΙΔΗΣ Β' 75 τ.3/11


------

Μαθηματικοί Διαγωνισμοί- Μαθηματικές Ολυμπιάδες

--------­

περιγεγραμμένοι κύκλοι των τριγώνων ΒΗ 1 Η 3 , ΑΜ 2 Η3 , CM 2 H 1 περνάνε από το ίδιο σημείο (έστω Κ 2 ) και οι περιγεγραμμένοι κύκλοι των τριγώνων CH 1 Η 2 , ΑΜ 3 Η2 , ΒΜ 3 Η 1 περνάνε από το ίδιο σημείο (έστω Κ 3 ). Στη συνέχεια να αποδείξετε ότι οι ευθείες ΑΚ 1 , ΒΚ 2 , CK3 συντρέχουν, δηλαδή περνάνε από το ίδιο σημείο, αν, και μόνο αν, οι ευθείες ΑΜ 1 , ΒΜ 2 , CM 3 συντρέχουν. Λύ σ η

Α

Έστω ( c ι ) ο περιγεγραμμένος κύκλος του τριγώνου Β Μι Η3 , (c 2 ) ο περιγε­ γραμμένος κύκλος του τριγώνου CΜ ι Η 2 , (c 3 ) ο περιγεγραμμένος κύκλος του τριγώνου Α Η2 Η3 και (c) ο περιγεγραμμένος κύκλος του εγγράψι­ μου τετραπλεύρου BH3 H2 C . Θεωρώντας τις τέμνουσες ΑΒ και AC του κύκλου ( c) , συμπεραίνουμε: ΑΒ · ΑΗ 3 = AC · A H2 . Το γινόμενο όμως ΑΒ · Α Η3 εκφράζει τη δύναμη του σημείου Α ως προς το κύκλο (c ι ) ενώ το γινόμενο AC · ΑΗ 2 εκφράζει τη δύναμη του σημείου Α ως προς το κύκλο (c 2 ) . Άρα το σημείο Α ,

Β

Σχήμα 5

ανήκει στον ριζικό άξονα των κύκλων ( c ι ) και ( c 2 ) . Έστω τώρα ότι οι κύκλοι (c ι ) και (c 2 ) τέμνονται στο σημείο Κι (εκτός βέβαια από το σημείο Μ ι ). Τό­ τε η ευθεία που ορίζουν τα σημεία αυτά (δηλαδή τα Κ ι και Μ ι ) είναι ο ριζικός άξονας των κύκλων (c ι ) και (c 2 ) . Από τους παραπάνω συλλογισμούς προκύπτει ότι τα σημεία Α, Κ ι και Μι είναι σ υνευθειακά. Θα αποδείξουμε ότι και ο κύκλος ( c 3 ) περνάει από το σημείο Κ ι , δηλαδή ότι το τετράπλευρο ΑΗ2 Κ ι Η 3 είναι εγγράψιμο. Από το εγγράψιμο τετράπλευρο BH3 H2 C έχουμε: φ = Β. Από το εγγεγραμμένο τετράπλευρο ΒΜ ι Κ ι Η 3 έχουμε: ω= Β. Άρα είναι ω= φ και κατά συνέπεια το τετράπλευρο Α Η2 Κ ι Η 3 είναι εγγράψιμο. Με όμοιο τρόπο αποδεικνύουμε ότι και οι δύο άλλες τριάδες κύκλων, περνάνε από το ίδιο σημείο. Προφανώς τώρα οι ευθείες ΑΚ ι , ΒΚ υ CΚ 3 συντρέχουν, αν, και μόνο αν, συντρέχουν οι ευθείες AM ι , BM z , CM 3 (δεδομένου ότι τα σημεία Α, Κι , Μι , τα σημεία Β, Κ 2 , Μ 2 και τα σημεία C, K3 , M 3 , είναι συνευθειακά. Π ρ όβλη μα 3

Αν a, b, χ, y ε JR με { a , b) :;t: {Ο, Ο) και { χ, y ) :;t: {Ο, Ο) και ισχύουν a ( χ 2 - y 2 ) - 2bxy = χ { a 2 - b2 ) - 2aby b ( χ 2 - y 2 ) + 2axy = y { a 2 - b2 ) + 2abx, να αποδείξετε ότι χ = a και y = b . ΛίJσ η

Σύμφωνα με τον ορισμό της ισότητας μιγαδικών αριθμών, προκύπτει ότι το σύστημα των δύο δεδομένων εξισώσεων είναι ισοδύναμο με την εξίσωση : a χ 2 - y 2 - 2bxy J + b ( χ 2 - y 2 + 2axy = χ ( a 2 - b 2 - 2aby J + y ( a 2 - b 2 + 2abx 2 �( a + bi · χ 2 - y 2 + 2xyi = ( a 2 - b 2 + 2abi J · χ + yi �( a + bi · χ + yi ) = ( a + bi γ · χ + yi

[(

)

)

} [ ) [ )( ) [( ) J [ ) ( ) � χ + yi = a + bi (αφού ( α, β ) (Ο, Ο) και ( x, y ) (Ο, Ο)) � χ = a, y = b. [

:;e

:;e

ΕΥΚΛΕΙΔΗΣ Β' 75 τ.3/12

)

} ( )


------ Μαθηματικοί Διαγωνισμοί- Μαθηματικές Ολυμπιάδες -------

Π ρ ό βλη μα

4

Σημείο Μ βρίσκεται στο εσωτερικό κύκλου c( O,r) , όπου r = 15cm , σε απόσταση 9cm από το κέντρο του κύκλου. Να βρείτε τον αριθμό των χορδών του κύκλου C ( O,r ) που περνάνε από το ση­ μείο Μ και το μήκος τους είναι ακέραιος αριθμός. Λ ίJσ η Θεωρούμε τη χορδή ΑΒ που περνάει από το σημείο Μ και το κέντρο Ο του κύκλου, καθώς και την κά­ θετη προς αυτήν χορδή ΓΜΔ , οπότε το σημείο Μ είναι το μέσο της χορδής Γ Δ. Η χορδή ΑΒ έχει ακέ­ ραιο μήκος 30cm . Από το θεώρημα τεμνομένων χορδών έχουμε ότι: ΓΔ 2 = 6 · 9 + 1 5 ΓΔ 2 = 144 � ΓΔ = 1 2 � ΓΔ = 24. ΓΜ · ΜΔ = ΑΜ · ΜΒ � 2 )� 2 ( 2

( )

( )

Έτσι μέχρι τώρα έχουμε βρει δύο χορδές του κύκλου C (O, r) που περνάνε από το σημείο Μ και έχουν ακέραιο μήκος. Θεωρούμε τυχούσα χορδή ΚΛ του κύκλου C ( O, r) που περνάει από το Μ και έστω ΜΕ = χ, ΜΟΕ = θ, όπου Ε είναι το μέσο της ΚΛ, σχήμα

r�"' Γ

!

6. Αν υποθέσουμε ότι Ο � θ � 2: , τότε έχουμε θεωρήσει όλες τις 2

.

/

χορδές του κύκλου C ( Ο, r) που περνάνε από το Μ και τα άκρα

/

)��

τους Κ και Λ βρίσκονται στα ελάσσονα τόξα ΑΓ και ΒΔ , αντί­ στοιχα . Για κάθε μία από αυτές τις χορδές αντιστοιχεί και μία α­ κόμη που είναι η συμμετρική της ως προς τη διάμετρο ΑΒ. Για τη χορδή ΚΛ, αν συμβολίσουμε το μήκος της ως e (θ) , έχουμε

e(θ) = 2.J225 - 8 1συν 2 θ, ο � θ � 2: . 2 8 1ημ2θ > 0, θ ε ο,2: , έπεται ότι η συνάρτηση e (θ) είναι γνησίως αύ2 .J225 - 8 1συν 2 θ ξουσα στο διάστημα Ο, οπότε η συνάρτηση e ( θ) έχει σύνολο τιμών το διάστημα Σχήμα 6 ' Επειδή είναι e ( θ) =

( )

[ �] ,

[ (Ο) , ( ( �)] = [ 24,30] . Άρα το μήκος της χορδής (

ΚΛ

μπορεί να πάρει όλες τις ακέραιες τιμές του δια­

στήματος [24,30 ] . Αν λάβουμε υπόψιν και τη συμμετρική χορδή της ΚΛ ως προς τη διάμετρο ΑΒ, τότε τα πέντε μήκη 25, 26, 27, 28, 29 λαμβάνονται δύο φορές το καθένα, ενώ τα μήκη 24 και 30 λαμβάνονται από μία φορά. Έτσι έχουμε συνολικά 1 2 χορδές που περνάνε από το Μ με ακέραιο μήκος. Π α ρ ατή ρη σ η I :

Θα μπορούσαμε επίσης να χρησιμοποιήσουμε το θεώρημα μέγιστης και ελάχιστης τ ιμ ής

για τη συνεχή συνάρτηση και μέγιστη τιμή την

e

e (θ) = 2.J225 - 8 1συν 2 θ, Ο � θ � 2:2 , η οποία έχει ελάχιστη τιμή την e(o) = 24

(�) = 30 . Αυτό προκύπτει από την παρατήρηση ότι τα μήκη των χορδών είναι

αντιστρόφως ανάλογα από τα αποστήματά τους και ότι το μέγιστο aπόστημα λαμβάνεται για το ελάχιστο aπόστημα λαμβάνεται για

θ = 2: . 2

θ = Ο , ενώ

Π α ρ ατή ρη σ η 2 : Σημειώνουμε ακόμη ότι οι χορδές με ακέραια μήκη 25, 26, 27, 28, 29, μπορούν να κα­ τασκευαστούν γεωμετρικά, αφού αν θέσουμε ΚΜ = χ και ΜΛ = y , τότε έχουμε x + y = m, m ε {25, 26, 27 , 28 , 29} και xy = 144 = 12 2 • Έτσι εξασφαλίζουμε την ύπαρξη αυτών των χορδών με ακέραιο μήκος, χωρίς τη χρήση του διαφορικού λογισμού. ΕΥΚΛΕΙΔΗΣ Β . 75 τ.3/13


ΗΟΜΟ MA THEMA TICUS

Η Homo Mathematicus είναι μια στήλη στο περιοδικό μας, με σκοπό την ανταλλαγή απόψεων και την ανάπτυξη προβληματισμού πάνω στα εξής θέματα: I ) Τι είναι τα Μαθηματικά, 2) Πρέπει ή όχι να διδάσκονται, 3) Ποιοι είναι οι κλάδοι των Μαθηματικών και ποιο το αντικείμενο του καθενός, 4) Ποιες είναι οι εφαρμογές τους, 5) Ποιες επιστήμες ή κλάδοι επιστημών απαιτούν καλή γνώση των Μαθηματικών για να μπορέσει κάποιος να τους σπουδάσει. rιu τ ο 1 ι .: ι> ι "·ψγ{π::,;: τ η ς σηj i.ι1 .: : παράκληση ! τα κείμενα της στήλης αυτής, ως προς το περιεχόμενό τους και ως προς το επίπεδό τους, θα πρέπει να είναι συμβιβαστά με τα ενδιαφέροντα και το επίπεδο κατανόησης από μέρους των παιδιών.

Ι . "τι

�:π ι μi:λη ο : Κ ερ ιι σ ιφ ίδ η ς Γι6ηη ι:ίι•αι τα ι�ΙαΟημα τιιιά : "

επιστήμονες. Ήδη ο Descartes, μι}"ώντας για τη γεωμετρία, αναφερόταν στη μεταφυσική: « . . . όταν φαντάζομαι ένα τρίγωνο», έγραφε, «μολονότι ένα τέτοιο σχήμα δεν υπάρχει ίσως σε κανένα άλλο μέρος του κόσμου πέρα από τη σκέψη μου, ούτε υπήρξε ποτέ, αυτό δεν σημαίνει ότι παύει να υπάρχει μια ορισμένη φύση ή μορφή ή ουσία που καθορίζεται απ αυτό το σχήμα, που είναι αναλλοίωτη και αιώνια και την οποία ούτε την μαθηματικών αντικειμένων. Έχουν υποστηριχθεί επινόησα ούτε εξαρτάται με κανένα τρόπο από το [R. Descartes, «Meditations δύο εκ διαμέτρου αντίθετες απόψεις: ο «ρεαλισμός» πνεύμα μου» και ο «κονστρουκτιβισμός». Για τον «ρεαλιστή», metaphysiqus», V, Gallimard, collection Pleiade, που εμπνέεται απ ' ευθείας από τον Πλάτωνα, ο 1 970, σελ. 3 1 1 ] Για τους «κονστρουκτιβιστές», τα μαθηματικά κόσμος είναι γεμάτος από Ιδέες η πραγματικότητα των οποίων διαφέρει από την αισθητή αντικείμενα είναι ορθολογικές οντότητες που πραγματικότητα. Πολλοί σύγχρονοι μαθηματικοί υπάρχουν μόνο μέσα στη σκέψη του μαθηματικού θεωρούν τους εαυτούς τους «ρεαλιστές». Ο και όχι σ ' έναν πλατωνικό κόσμο, ανεξάρτητα από Dίeudonne, για παράδειγμα, γράφει στο βιβλίο του: την ύλη · δεν υπάρχουν παρά μόνο μέσα στους «Είναι αρκετά δύσκολο να περιγράψουμε τις ιδέες νευρώνες και στις συνάψεις των μαθηματικών που αυτών των μαθηματικών, οι οποίες εξ άλλου τα δημιουργούν, καθώς κι εκείνων που τα ποικίλλουν από τον έναν στον άλλο. Δέχονται ότι τα κατανοούν και τα χρησιμοποιούν. Σε μια εμφανώς μαθηματικά αντικείμενα κατέχουν μια "πραγματικό­ ακραία εκδοχή, αυτή την άποψη την έχουμε τητα " που διαφέρει από την αισθητή πραγματικότη­ ξανασυναντήσει στους εμπειριστές φιλοσόφους, τα (παρεμφερή, ίσως, με εκείνη που ο Πλάτων όπως στον Locke ή τον Hume. Ο Hume, λόγου αναγνώριζε στις "Ιδέες " του;)». Ο Cantor, αυτός ο χάρη, γράφει ότι «όλες οι ιδέες μας είναι αντίγραφα τόσο μεγάλος μαθηματικός, έγραψε: «Η ύψιστη των εντυπώσεών μας». Γι ' αυτόν, τα γεωμετρικά τελειότητα του Θεού είναι η δυνατότητά του να αντικείμενα προέρχονται αποκλειστικά από την δημιουργεί ένα άπειρο σύνολο, και η απέραντη εμπειρία . . » μεγαλοθυμία του τον ώθησε να το δημιουργήσει». [πηγή : «ΤΑ ΜΑΘΗΜΑτΙΚΆ ΚΑΙ Ο Ε ΓΚΕΦΑΛΟΣ», Βρισκόμαστε στην καρδιά του mathesίs dίνίnα, στην Jeane-Pieπe Changeux και Alain Connes, εκδόσεις καρδιά της μεταφυσικής. Αυτό είναι κάτι που μας ΚΆΤΟΠΤΡΟ (Σειρά: Εγκέφαλος και Νόηση), Αθήνα εκπλήσσει όταν πρόκειται για σοβαρούς 1 995]

Δύο κορυφαίοι επιστήμονες ο Jean-Pieπe Changeux ( Μοριακός Νευροβιολόγος, Γαλλική Ακαδημία Επιστημών) και ο Alain Connes (Μαθηματικός στο Col!ege de France, βραβείο Fieldes), συνομιλούν, μεταξύ τους, για τη φύση των Μαθηματικών. Απ' αυτή την εξαιρετικά ενδιαφέρουσα συνομιλία παραθέτουμε παρακάτω ένα μικρό απόσπασμα. «JPC. Ας έρθουμε τώρα στη φύση των

.

1 1.

'Ά υ η) το

.

Γιι'ί\'νης Κ �: p α σ ιφ ί<i η ς

ξ::ρ α τ::; "

Πόσα δεκαδικά ψηφία του αριθμού π (3 , 1 4 . . . ) έχουν υπολογιστεί μέχρι σήμερα [η απάντηση στο τέλος της στήλης] 1 1 1 . "οι συ ιι:.ρ ;ιά τι:ς τη ς στιί/. η ς γρ ά φ ο ιJ ι •-:φ ω το ι) ν "

Γ/μ (:Υ το Ο/μ α :

«

Ειι()ι)μω ιι

το υ t.' ι•αι·ί.\tcιι τ ο υ ιVίιιο ι J

γαλλικά: Σ cJ.νη Καπρ Cι.γκου/ Προ/.ι:;•()μιτα.

Λ ι;γ::ρ ο ι!

fπ α ι•;:π.

Λ ΙJ (:Υ ιιf

fΜ ετά.φρ αση απ() τα

Ο Νίκος Λυγερός συνηθίζει να μας ξαφνιάζει ευχάριστα, πότε με τα βαθυστόχαστα ΕΥΚΛΕΙΔΗΣ Β' 75 τ.3/14


ΗΟΜΟ MATHEMAτiCUS

σχόλια πάνω σε μαθηματικά θέματα, πότε με τους στίχους του, πότε με τα ζωγραφικά του έργα, πότε . . . . . Σήμερα δημοσιεύουμε μια ποιητική αναφορά στις τελευταίες ημέρες της ζωής του πρόωρα χαμένου γίγαντα της μαθηματικής σκέψης, του Evariste Galois. J-.- �·(.Ι{ψ Γ η;•

Όφειλε να συνεχίσει να γράφει ως να 'ταν τούτο ένα καθήκον παρά την κούραση και τη νύχτα τουλάχιστον όσο η μελάνη ζούσε. Διότι αυτό ήταν το τέλος, αυτό ήταν σίγουρο. Όφειλε ν' αφήσει ένα ίχνος ως να 'ταν τούτο διαθήκη προκειμένου οι άλλοι μαθηματικοί

ΠΝ!

1-.ΊιαΓi\/f' .• ..

ν' αγγίξουν την αλήθεια αυτής της παράξενης ομάδας. Κι έπειτα δεν υπήρχε επομένη εκτός από κείνη της μονομαχίας όπου το παράλογο νίκησε το ορθόν δίχως να 'χει να δικαιολογηθεί για τη διάπραξη ενός εγκλήματος κατά της aνθρωπιάς ενός ανθρώπου.

[πηγή : από την η λεκτρον ική αλληλογραφία μας με τον Ν . Λυγερό] J c μ ' '·' ·

Ο φίλος μου ο Αντώνης, όταν του τηλεφώνησα για να του ευχηθώ για τη γιορτή του, μου είπε να διαβάσω ένα δημοσίευμα αθηναϊκής εφημερίδας το οποίο, κατά τη γνώμη του, παρουσιάζει ενδιαφέρον. Πραγματικά, διάβασα το άρθρο και επειδή συμμερίστηκα το ενδιαφέρον αυτού του άρθρου, σας το παρουσιάζω .

. • ..

.

«Για τους περισσότερους το μεσημεριανό γεύμα με έναν συνάδελφο είναι μια χαλαρή υπόθεση όπου οι προβληματισμοί εξαντλούνται στο ποιο πιάτο θα διαλέξουν και αν θα πάρουν γλυκό. Για τον Ρικ Μέιμπρι και τον Πολ Ντάιερμαν όμως το ζήτη μα δε\' ήταν ποτέ τόσο απλό. Τους είναι αδύνατον. για παράδειγμα, να δουν στο τραπέζι τους μια πίτσα χωρίς να προσπαθήσουν να βρουν τη λύση στο μαθηματικό πρόβλημα του πώς να τη

μοιράσουν. «Τρώγαμε μαζί τουλάχιστον μία φορά την εβδομάδα»λέει ο κ. Μέιμπρι αναφερόμενος στις αρχές της δεκαετίας του 1 990, όταν οι δύο μαθηματικοί εργάζονταν στο Πολιτειακό Πανεπι­ στήμιο της Λουιζιάνας. «0 ένας από τους δυο μας έφερνε ένα τετράδιο και aρχίζαμε να κάνουμε σχεδιαγράμματα αφήνοντας το φαγητό μας να κρυώνει».

Το πρόβλημα που τους απασχολούσε ήταν το εξής: ας υποθέσουμε ότι στη βιασύνη του ο σερβιτόρος κόβει την πίτσα εκτός κέντρου, με όλες τις τομές να διασταυρώνονται σε ένα σημείο σχηματίζοντας ίσες γωνίες με τη γειτονική τους. Οι εκτός κέντρου τομές σημαίνουν ότι τα κομμάτια δεν θα έχουν το ίδιο μέγεθος. Επομένως δύο άτομα που παίρνουν εναλλάξ διαδοχικά κομμάτια, θα έχουν φάει ίσα μερίδια όταν τελειώσει η πίτσα και, αν όχι, ποιος θα έχει φάει περισσότερο; Φυσικά, θα μπορούσε κανείς να λύνει κάθε φορά το συγκεκριμένο πρόβλημα υπολογίζοντας την επιφάνεια κάθε κομματιού και προσθέτοντας τα κομμάτια του καθενός μεταξύ τους. Οι δύο ερευνη­ τές είναι όμως μαθηματικοί και δεν αρκούνται σε τέτοιου είδους λύσεις: ήθελαν μια θεωρητική κατασκευή χωρίς ακριβείς υπολογισμούς, έναν κανόνα που θα ισχύει πάντοτε και θα μπορεί να εφαρμόζεται για κάθε στρογγυλή πίτσα. Η προσπάθειά τους ξεκίνησε το 1 994, όταν ο κ. Ντάιερμαν έδειξε στον κ. Μέιμπρι μια διορθωμένη εκδοχή του θεωρήματος της πίτσας που επανεμφανίστηκε στο «Mathematics Magazine» προκαλώντας τους αναγνώστες να

αποδείξουν δύο συγκεκριμένες περιπτώσεις του προβλήματος, αυτή στην οποία η πίτσα κόβεται με 3 τομές και αυτή στην οποία κόβεται με 5 . Ο κ . Ντάιερμαν έλυσε γρήγορα το πρόβλημα των 3 τομών, το οποίο οι συγγραφείς του άρθρου είχαν λύσει αλλά δεν aποκάλυπταν στο δημοσίευ­ μα. Στη συνέχεια οι δύο συνεργάτες βρήκαν τη λύση για τις περιπτώσεις των 5 και των 7 τομών (οι οποίες έδιναν το ίδιο αποτέλεσμα με τις 3). Ενθουσιασμένοι με την επιτυχία τους, θεώρησαν ότι είχαν βρει την τεχνική που θα έλυνε πλήρως το πρόβλημα. Σε ένα μονό αριθμό τομών, τα aντιδιαμετρικά αντίθετα κομμάτια πηγαίνουν σε διαφορετικούς συνδαιτυμόνες, οπότε συγκρίνοντας κανείς τα μεγέθη τους μπορεί να βρει, για αυτά τα δύο, ποιος παίρνει περισσότερο και πόσο και στη συνέχεια να περάσει στο επόμενο aντιδιαμετρικά αντίθετο «ζευγάρι». Παρ ' ότι ακούγεται απλό, στην πράξη αποδείχθηκε σχεδόν αδύνατον να βρεθεί μια λύση που να καλύπτει όλους τους πιθανούς μονούς αριθμούς τομών. Οι δύο μαθηματικοί χρησιμοποίη­ σαν ένα γεωμετρικό τέχνασμα για να aπλοποιή­ σουν τη διαδικασία, εισάγοντας ένα παραλληλεπί-

Ί�

ΕΥΚΛΕΙΔΗΣ Β' 75 τ.3/1 5


-------

ΗΟΜΟ MATHEMAτiCUS

πεδο που σχηματίζεται από κάθε τομή και μια παράλληλή της γραμμή η οποία περνάει από το κέντρο της πίτσας. Και πάλι όμως η λύση δεν ήταν ικανοποιητική εφόσον απαιτούσε πολύπλοκους υπολογισμούς. Ήταν, όπως λένε, «άσχημη» . Όπως συμβαίνει με πολλές μαθηματικές σπαζοκεφαλιές, η απάντηση ήρθε σε στάδια- για διαφορετικές κάθε φορά πιθανές περιπτώσεις του

-------

προβλήματος. Η ευκολότερη προσφέρεται όταν τουλάχιστον μία τομή περνάει από το κέντρο της πίτσας: ένα γρήγορο σχήμα μπορεί να δείξει ότι στην περίπτωση αυτή τα aντιδιαμετρικά κομμάτια είναι συμπληρωματικά μεταξύ τους οπότε μοιρά­ ζονται ίσα ανάμεσα στους δύο συνδαιτυμόνες, ανεξάρτητα από το πόσες είναι οι τομές.

Η ευ l(ολία των ζυγc.ί)ν

Τι γίνεται όμως όταν καμία τομή δεν περνάει από το κέντρο; Αν η πίτσα κόβεται μία φορά, η απάντηση είναι προφανής με το μάτι: όποιος τρώει το κέντρο τρώει περισσότερο. Αν γίνουν δύο τομές, που δίνουν τέσσερα κομμάτια, το αποτέλεσμα είναι το ίδιο: όποιος φάει το κομμάτι που περιλαμβάνει το κέντρο τρώει περισσότερη πίτσα. Όσο όμως οι τομές αυξάνονται, παρουσιάζονται διάφορες ανωμαλίες και η λύση τους οδήγησε, με τον χρόνο, στον σχηματισμό των τριών γενικών κανόνων που απαρτίζουν το θεώρημα της πίτσας. Ο σ κόπ::λος

Με τους μονούς αριθμούς τομών όμως τα πράγματα περιπλέκονται. Εδώ το θεώρημα της πίτσας λέει ότι αν κόψει κανείς την πίτσα με 3, 7, 1 1 , 1 5 . . . τομές χωρίς καμία τομή να περνάει από το κέντρο της, τότε αυτός που θα πάρει το κομμάτι που περιέχει το κέντρο της πίτσας θα φάει συνολικά περισσότερο. Αν όμως οι τομές είναι 5, 9, 1 3 , 1 7 . . . αυτός που θα πάρει το κομμάτι με το κέντρο καταλήγει να τρώει λιγότερο. Το θεώρημα ισχύει αν κάνει κανείς τους υπολογισμούς, απέβη όμως εξαιρετικά δύσκολο να αποδειχθεί από τους μαθηματικούς. Τόσο δύσκολο ώστε οι κκ. Μέιμπρι και Ντάιερμαν μόλις τώρα κατόρθωσαν να ολοκληρώσουν μια απόδειξη η οποία καλύπτει όλες τις πιθανές περιπτώσεις. Η προσπάθειά τους ξεκίνησε το 1 9 94, όταν ο κ. Ντάιερμαν έδειξε στον κ. Μέιμπρι μια διορθωμένη εκδοχή του θεωρήματος της πίτσας «Mathematics που επανεμφανίστηκε στο Magazine» προκαλώντας τους αναγνώστες να από­ δείξουν δύο συγκεκριμένες περιπτώσεις του προβλήματος, αυτή στην οποία η πίτσα κόβεται με 3 τομές και αυτή στην οποία κόβεται με 5 . Η «άσχημη» λύση Ο κ. Ντάιερμαν έλυσε γρήγορα το πρόβλημα

Ο πρώτος ορίζει ότι αν η πίτσα κοπεί σε ένα δεδομένο σημείο με ζυγό αριθμό τομών μεγαλύτε­ ρο του 2, θα μοιραστεί ίσα ανάμεσα στους δύο συνδαιτυμόνες που παίρνουν εναλλάξ διαδοχικά κομμάτια. Προτάθηκε για πρώτη φορά για 4 τομές (8 κομμάτια) από κάποιον Λ. Τζ. Απτον το 1 967 στο περιοδικό «Mathematics Magazine», ενώ η λύση που όριζε ότι η πίτσα μοιράζεται ίσα για οποιονδήποτε αριθμό ζυγών τομών ήλθε έναν χρόνο αργότερα ως απάντηση στην «πρόκληση» του Απτον.

των

μ ο \'{\)\'

των 3 τομών, το οποίο οι συγγραφε ίς του άρθρου είχαν λύσει αλλά δεν aποκάλυπταν στο δημοσίευμα. Στη συνέχεια οι δύο συνεργάτες βρήκαν τη λύση για τις περιπτώσεις των 5 και των 7 τομών (οι οποίες έδιναν το ίδιο αποτέλεσμα με τις 3). Ενθουσιασμένοι με την επιτυχία τους, θεώρη­ σαν ότι είχαν βρει την τεχνική που θα έλυνε πλήρως το πρόβλημα. Σε ένα μονό αριθμό τομών, τα aντιδιαμετρικά αντίθετα κομμάτια πηγαίνουν σε διαφορετικούς συνδαιτυμόνες, οπότε συγκρίνοντας κανείς τα μεγέθη τους μπορεί να βρει, για αυτά τα δύο, ποιος παίρνει περισσότερο και πόσο και στη συνέχεια να περάσει στο επόμενο aντιδιαμετρικά αντίθετο «ζευγάρι». Παρ ' ότι ακούγεται απλό, στην πράξη απόδεί­ χθηκε σχεδόν αδύνατον να βρεθεί μια λύση που να καλύπτει όλους τους πιθανούς μονούς αριθμούς τομών. Οι δύο μαθηματικοί χρησιμοποίησαν ένα γεωμετρικό τέχνασμα για να aπλοποιήσουν τη διαδικασία, εισάγοντας ένα παραλληλεπίπεδο που σχηματίζεται από κάθε τομή και μια παράλληλή της γραμμή η οποία περνάει από το κέντρο της πίτσας. Και πάλι όμως η λύση δεν ήταν ικανοποιητική εφόσον απαιτούσε πολύπλοκους υπολογισμούς. Ήταν, όπως λένε, «άσχημη» .

Ο π α ρ ιί.γων « κ υ 0 υ μ 6 ς αέρ ας»

Στα χρόνια που ακολούθησαν, ασχολήθηκαν κατά καιρούς και πάλι με το πρόβλημα, χωρίς όμως περαιτέρω επιτυχία. Η «φώτιση» ήρθε το 2006, όταν ο κ. Μέιμπρι βρισκόταν για διακοπές στη Γερμανία. «Ήμουν σε ένα ωραίο ξενοδοχείο, με ευχάριστο, δροσερό περιβάλλον και χωρίς κομπιούτερ»λέει.«Άρχισα να σκέφτομαι ξανά το

πρόβλημα και τότε όλα άρχισαν να λειτουργούν». Ως τότε οι δύο μαθηματικοί χρησιμοποιούσαν μοντέλα στον ηλεκτρονικό υπολογιστή για τα από­ τελέσματά τους. Μόλις όμως ο κ. Μέιμπρι άφησε την τεχνολογία στην άκρη, μπόρεσε να δει το πρόβλημα καθαρά. Επιστρέφοντας έβαλε ξανά τον υπολογιστή να δουλέψει για να βρει όλες τις

ΕΥΚΛΕΙΔΗΣ Β' 75 τ.3/16


-------

ΗΟΜΟ MATHEMAτiCUS

πληροφορίες που χρειάζονταν για να προχωρήσει τις σκέψεις του και τελικά όλα μπήκαν στη θέση τους. Η απόδειξη του θεωρήματος της πίτσας βρέθηκε, λοιπόν, επιτέλους και έχει δημοσιευθεί στην επιθεώρηση Aιηerican Mathemat i cal Monthly (Μάιος 2009). Ποιες πρακτικές εφαρμογές μπορεί να έχει; Προς το παρόν δεν διαφαίνεται καμία, αυτό όμως δεν ανησυχεί τον κ. Μέιμπρι. «Το παράξενο με εμάς τους μαθηματικούς»λέει «είναι ότι συχνά δεν μας ενδιαφέρει αν τα αποτελέσματα έχουν εφαρμογές γιατί τα ίδια τα αποτελέσματα � \'.

Ε π ι κ α ι ρ 6 τη τ α :

-------

είναι τόσο όμορφα». Πολλές φορές ωστόσο η χρησιμότητα των λύσεων τέτοιου είδους αφηρημένων μαθηματικών προβλημάτων εμφανίζε­ ται με απρόσμενους τρόπους: ένα ξεχασμένο μαθη­ ματικό παράδοξο του 1 9ου αιώνα για ένα μορφο­ κλασματικό είδος καμπύλης επανήλθε στο προσκή­ νιο ως μοντέλο για το σχήμα του ανθρώπινου γονι­ διώματος» [πηγή : εφη μερίδα "ΤΟ ΒΗ ΜΑ", 1 7/ 1 /20 1 0, © 2009 New Scientist M agazine, Reed Business Information Ltd]

Η πrι.;ιια) σμια μ α Οημα τιιιιί ιω ι ιγ) τη τα πι: ιιΟι:ί

Στις 5/1 0/20 1 0 "έφυγε" ο Israel Moiseeνich Gelfand, ένας από τους πιο επιφανείς μαθηματι­ κούς του 20ου αιώνα· πέθανε σε ηλικία 96 ετών. Ο Gelfand εργάστηκε σε πολλούς τομείς των μαθηματικών, κυρίως στη Συναρτησιακή Ανάλυση, Αρμονική Ανάλυση, Διαφορικές Εξισώσεις και Διακριτά Μαθηματικά. Αργότερα στη ζωή του, άρχισε να ενδιαφέρεται για τη θεωρητική Βιολογία. Ι I .,. Η απά ιιτη ση στο :

"Α υ η) το ξιiμ α τι:; "

Επιστήμονας της πληροφορικής έσπασε το ρεκόρ υπολογισμού των ψηφίων μιας διάσημης μαθηματικής σταθεράς, του αριθμού π, υπολογίζοντας σχεδόν 2, 7 τρισεκατομμύρια ψηφία που ακολουθούν μετά το 3, 1 4, κάπου 1 23 δισεκατομμύρια περισσότερα ψηφία σε σχέση με το προηγούμενο ρεκόρ. Ο Φαμπρίς Μπελάρντ, σύμφωνα με το BBC, χρησιμοποίησε έναν απλό επιτραπέζιο υπολογιστή για να κάνει το νέο υπολογισμό, που του πήρε 1 3 1 μέρες συνολικά. Ο νέος αριθμός-ρεκόρ του π χρειάζεται πάνω από ένα terabyte για να απόθηκευ­ τεί σε σκληρό δίσκο. Τα προηγούμενα ψηφία-ρεκόρ του π είχαν βρεθεί με τη βοήθεια τεράστιων υπερ-υπολογιστών, όμως ο Μπελάρντ υποστηρίζει ότι η δική του μεθο­ δος υπολογισμού είναι 20 φορές πιο απότελεσματι­ κή. Το προηγούμενο ρεκόρ με περίπου 2,6 τρισ. ψηφία κατείχε, από τον Αύγουστο του 2009, ο Νταϊσούκε Τακαχάσι του πανεπιστημίου Τσουκού­ μπα της Ιαπωνίας και του είχε πάρει 29 ώρες, αλλά με την υποστήριξη ενός σούπερ-κομπιούτερ 2.000 φορές πιο γρήγορου και χιλιάδες φορές πιο ακριβού από τον κοινό υπολογιστή που χρησιμοποίησε ο Μπελάρντ. Εκτιμάται ότι αν χρειάζεται περίπου ένα

Εκτός των άλλων ο Gelfand έγραψε περισσότερα από 30 βιβλία, το πιο γνωστό από τα οποία είναι το τετράτομο με τίτλο "Γενικευμένες Συναρτήσεις". Για πολλά χρόνια διηύθυνε το περίφημο σεμινάριο Gelfand στη Μόσχα. Μετά την αποδημία του στις ΗΠΑ (αρχές δεκαετίας του ' 90), συνέχισε εκεί τις επιστημονικές του εργασίες δευτερόλεπτο για να εκφωνηθεί ένας αριθμός, η πλήρης απαρίθμηση φωναχτά όλων των ψηφίων του π θα απαιτούσε πάνω από 49.000 χρόνια! Ο Μπελάρντ δήλωσε ότι διάβασε το πρώτο του βιβλίο του για τον αριθμό π όταν ήταν 1 4 ετών και έκτοτε παρακολουθούσε ανελλιπώς τις προσπά­ θειες υπολογισμού όλο και περισσότερων ψηφίων του. Όπως είπε, τον ενδιαφέρει ιδιαίτερα η πρακτική πλευρά του ζητήματος, καθώς ορισμένοι από τους αλγόριθμους που απαιτούνται για τον υπολογισμό του π, είναι χρήσιμοι για άλλα πράγμα­ τα στους υπολογιστές. Όπως ανέφερε, σχεδιάζει να δημοσιοποιήσει μια έκδοση του προγράμματος που χρησιμοποίησε για τον υπολογισμό του π, ενώ δεν απέκλεισε να επιμείνει για την ανακάλυψη και άλλων ψηφίων στο μέλλον. Όπως δήλωσε ο Αιβαρς Πίτερσον, διευθυντής της Μαθηματικής Ένωσης της Αμερικής, το νέο αποτέλεσμα αποτελεί τον τελευταίο κρίκο σε μια μακρά αλυσίδα προσπαθειών να διευρυνθεί το μήκος των γνωστών ψηφίων του «Π». Μεταξύ άλλων, ο Νεύτων είχε περάσει αρκετό χρόνο προσπαθώντας να βρει και άλλα ψηφία. [πηγή : εφη μερίδα «ΑΓΓΕΛΙ ΟΦΟΡΟΣ», 8/ 1 /20 1 0]

ΕΥΚΛΕΙΔΗΣ Β' 75 τ.3/17


�ι r', r ]1. ! ('\ ι I ·' ' - -

'I

1

1�1

- - - �co � � -Ji

�-

(Υ ·π�-� ιr;: 1 �-���. " -- -�

-��-

..

-

·

\_ι

,4\�\

/j'

�- ��

'

�.'-. f. -� � Ί._ι.

'-rι� -' u ..

·ι

1

o � o• · � c--� -� ---

Σύστη μα δύο γραμμικών εξισώσεων ε δύο α

Ε . Ε υστα θ ίου - Γ . Βλαχούτσ ικος

Αν θεωρήσουμε δύο γραμμικές εξισώσεις με δύο αγνώστους (αυτές παριστάνουν δύο ευθείες ( ε ι ) α χ + β1 Υ - γι και ( ε 2 ) ). Λέμε τότε ότι έχουμε ένα σύστημα δύο γραμμικών εξισώσεων. Το ( Σ) : ι -=._ . α2 χ + β 2 y - γ 2 Επίλυση του (Σ) είναι η διαδικασία εύρεσης ( εφ' όσον υπάρχουν) των κοινών λύσεων ( χ 0 ,y 0 ) των εξισώσεων και αν υπάρχουν λέμε ότι το (Σ) είναι Συμβιβαστό.

{

Γραφική επίλυση

Αρχικά παριστάνουμε τις ευθείες του συστήματος σε ένα καρτεσιανό σύστημα συντεταγμένων τότε : Αν οι ευθείες τέμνονται σε ένα σημείο λέμε ότι οι συντεταγμένες ( χ 0 ,y 0 ) του κοινού σημείου είναι μια λύση του συστήματος. Αν οι ευθείες είναι παράλληλες το σύστημα είναι αδύνατο, δηλαδή δεν έχει λύση . li Αν οι ευθείες ταυτίζονται το σύστημα έχει άπειρες λύσεις, δηλαδή άπειρα ζεύγη ( χ 0 ,y 0 ) που επαληθεύουν τις δύο εξισώσεις. �- :• Αλγεβρικές μέθοδοι επίλυσης α) Μέθοδος αντικατάστασης

Αρχικά λύνουμε τη μια εξίσωση ως προς τον έναν άγνωστο. Αντικαθιστούμε τον άγνωστο αυτό στην άλλη εξίσωση οπότε προκύπτει εξίσωση με έναν άγνωστο τον οποίο και υπολογίζουμε. Κατόπιν τον άγνωστο αυτό τον aντικαθιστούμε στην πρώτη εξίσωση και έτσι βρίσκουμε και τον άλλον άγνωστο.

"

{

j ι: - τ

_ - I ·'�

I

,\.

x - 2y = 10 . Να λυθεί το σύστημα : ( Σ ) : 2x + 3y = -29 ( Σ ) <=> μ ::

.

{

{

{

{

x = l 0 + 2y x = l 0 + 2y x = l0 - 14 x = l 0 + 2y <=> <=> ( χ , y ) = ( -4 -7 ) . <=> <=> 7y = -49 y = -7 2 ( 1 0 + 2y ) + 3y = -29 7 y + 20 = -29

:

(;( : / 11 ' ' ' 1' · ' • '1 (

'

{

Βρίσκουμε τον ίδιο άγνωστο και από τις δύο εξισώσεις και εξισώνουμε τα αποτελέσματα. 3χ - 2y = 5 Να λυθεί το σύστημα: ( Σ ) : -4χ + 3y = -7 ' ' \. ι, } ::-p -; -

{ {

3χ - 5 3χ - 5 y= 3χ - 5 3χ - 5 3·1-5 y= y= y = -2 2 <=> <=> <=> ( Σ ) <=> 2 <=> 2 <=> ( x, y ) = ( 1 , - 1 ) 2 4χ - 7 3χ 5 4χ 7 χ=1 -- = -9χ - 1 5 = 8χ - 14 χ=1 y= 3 2 3 y=

--

--

--

{

-

{

ΕΥΚΛΕΙΔΗΣ Β' 75 τ.3/18

-

{


' I]

�.

ιi '�";:τ i-.'[ ) j-:" (ιί t_(ι λ/

:(-;ϊ_ ) -Ιi' Τ � : λ �- ; (;-(;ί ;·j) \'

Μαθηματικά για την Α ' Λυκείου

Αρχικά πολλαπλασιάζουμε και τα δύο μέλη των εξισώσεων με κατάλληλους αριθμούς ώστε να προκύψουν στις εξισώσεις αντίθετοι συντελεστές σε ένα άγνωστο. Θα έχουν απόλυτη τιμή το Ε.Κ.Π. αυτών των συντελεστών. Κατόπιν προσθέτουμε κατά μέλη τις δύο εξισώσεις οπότε προκύπτει νέα εξίσωση με ένα άγνωστο, τον οποίο και υπολογίζουμε. Την τιμή αυτού του αγνώστου την aντικαθιστούμε σε μια από τις αρχικές εξισώσεις και έτσι υπολογίζουμε τον άλλο άγνωστο.

{_ --

2x - y x + y 3 Να λυθεί το σύστημα (Σ) : 22 l - 44y χ - + -- = 1 3 6 _

_ __ _ _

{2 ( 2x-y ) - ( x+ y ) =12 <=> {3x-3y=l2 <=> { x-y=4 2 <=> { 2x-2y=8 <=> { 2y=-1 2 ( x -2 ) + ( 1 -4y ) =6 2x -4y=9 2x-4y=9 ( - 1 ) -2x+4y=-9 x-y=4 :: H ( x y = , ) ( � =4 ::: Ξ :: Ξ � �· (Σ) <=>

( (!υ { � { i �

δ) Μέθοδος Οριζουσών, Γραμμικό σύστημα 2χ2

,.

Ορ ίζουσα

2

ης

τάξης

I ; �ι = αδ - βγ

Αρχικά υπολογίζουμε τις ορίζουσες :

2 .) \

.�

(Σ) _)

� ll

(j

+ βι y = γι

αχ (Σ) : α ι χ + β 2 2 y - γ2

α, α2 β 2

_

α 1 β 2 - α2 β 1 ,

Ι γι β ι = γιβ2 - γ2 β ι , Ο y = αι γ ι l = αι γ - α γι γ2 β2 l l α2 γ2 2 2 Αν Ο * Ο το (Σ) έχει μοναδική λύση την (x,y) = ( � , �) Αν Ο = Ο και Ο , * Ο ή Ο Υ * Ο το σύστημα είναι αδύνατο . Αν Ο = Ο και = Ο Υ = Ο ελέγχω το αρχικό σύστημα. Αν α 1 * Ο ή α 2 * Ο ή β 1 * Ο ή β 2 * Ο , τότε το σύστημα είναι αόριστο. Αν α 1 = α 2 = β 1 = β 2 = γ 1 = γ 2 = Ο, τότε το σύστημα είναι ταυτοτικό . Αν α 1 = α 2 = β 1 = β 2 = Ο και γ 1 * Ο ή γ2 * Ο, τότε το σύστημα είναι αδύνατο . Ομογενές Σύστημα 2χ2 αχ + β y = α και Ο = Ο Iαι {Τοαομογενές ι χ + β ι y =σύστημα έχει πάντοτε λύση την (x, y )=(O, O ) και ελέγχουμε αν έχει και άλλες λύσεις . Αν Ο * Ο το έχει μόνο την μηδενική λύση (x, y )=(O, O ) . Αν Ο = Ο το έχει άπειρες λύσεις διαφορετικές από την μηδενική (Αόρισρο) .

Οχ = ! .ί

{ Ο = Ι βι I =

Επίλυση Γραμμικού Συστήματος 2χ2

Ο,

:

0

<U OJ ![

(Σ) (Σ)

ΕΥΚΛΕΙΔΗΣ Β' 75 τ.3/19


{ Αρχικά υπολογίζουμε τις ορίζουσες D, D,, DY -1 1 =λ(2λ-1)+1+λ άρα D, =2λ2 -λ+1+λ=2λ2 +1 και D = l λ1 -1λ Ι =λ2 +1 , D ', = 1 2λ-1 1+λ λ D.v = Ι λ1 2λ1 +λ-11 = λ(l +λ) -1(2λ -1) =λ+ λ- - 2λ + 1 =λ- -λ+ 1 . Παρατηρουμε οτι για καθε, λ R , ειναι , το (Σ) εχει , μοναδικη, λυση, χ = -DD' = 2λλ-2+1+ 1 και y = λ2λ"-λ+1+ 1 D Επομενως Α l: Κ Η Σ Η

Να λύσετε το σύστημα λχ - y = 2λ - 1 χ + λy = 1 + λ Λύ σ η

?

=F-

ο

?

,

7 -

,

Ε

,

--=7 --

ΑΣ Κ Η Σ Η

Δίνονται οι ευθείες (ε 1 ) : 5χ + λy = 2λ - 1 και (ε 2 ) : y = 3 - λχ . Να προσδιορίσετε τις τιμές του λ Ε R ώστε να τέμνονται.

Οι εξισώσεις των ευθειών ορίζουν το σύστημα (Σ) : {5χλχ ++λyy ==32λ-1 Για να τέμνονται οι ευθείες θα πρέπει και αρκεί το (Σ) να έχει μία λύση. Δηλαδή πρέπει και αρκεί D Ο όπου : D = � � �� = ( λ2 -5 ) , δηλαδή λ ευθείες τέμνονται μόνον όταν λ� { .J5, -.JS} . ΛίJσ η

=F-

=F-

και λ * -.JS . Άρα οι

.J5

ΑΣ Κ Η Σ Η

Δίνεται το γραμμικό σύστημα 2χ2 με αγνώστους x,y αν για τις ορίζουσες Ο, Ο, , ΟΥ ισχύει 40 , 2 + 40 Υ 2 + 902 + 40, - 120 Υ - 60 s -1 1 (I) Να βρείτε τη λύση του. ΛίJσ η

'Εχουμε : (1)� ( 2D, )2 + 2 · 2D, + 1-1 + ( 2D Y )2 -2·2DY ·3 +32 -3 2 + (3D)2 -2 ·3D ·1 + 12 -1 1 ::::; -11 � ( 2D , + 1 )2 + ( 2D Y -3 )2 +(3D -1/ - 11 ::::; -1 1 ( 2D , + 1 )2 + ( 2D), -3 )2 +(3D - 1) 2 ::::; Ο (II) Επειδή το πρώτο μέλος της είναι άθροισμα μη αρνητικών αριθμών θα ισχύει: ( 2D, + 1)2 = ( 2DY -3)2 =(3D-1)2 =0 Επομένως 2D , + 1 = Ο και 2D), -3 = Ο και 3D-1 = Ο � ' = _ ..!._2 και D = �2 και D =.!.3 έτσι 1 3 D 3 , D 2 -2 y 1 Χ ο = D = -1- =- 2 , Υο =-=-=-3 D 3 2 �

(Π)

D

Υ

ΛΣ Κ Η Σ Η

α) Αν για τους πραγματικούς αριθμούς κ,λ,μ ισχύει: l κ-�2 =l κ-�2 � λ-�2 • Να δείξετε ότι λ= μ ή κ=μ. β) Αν ένα γραμμικό σύστημα 2χ2 έχει μοναδική λύση (x 0 , y 0 ) η οποία πληρεί την 3χ0 + 2y 0 = 4 και για τις ορίζουσες Ο, Ο, , ΟΥ ισχύει η ισότητα ID-0.1 2 =I D-1:\ 1 2 �η -1:\ 1 2 (1). Ν α βρείτε την λύση ( χ0 , y 0 ) . Λύ σ η α)

Από την ισότητα l κ -λ12 = l κ -μl2 + l λ -μl 2 προκύπτει ( κ-λ )2 = ( κ-μ )2 + ( λ-μ / ΕΥΚΛΕΙΔΗΣ Β' 75 τ.3/20


Μ αθηματικά για την Α ' Λυκε ίου

� κ 2 + λ2 - 2κλ = κ 2 + μ 2 - 2κμ + λ2 + μ 2 - 2λμ � 2μ 2 - 2κμ - 2λμ + 2κλ = Ο μ 2 - κμ - λμ + κλ = Ο ή μ(μ - κ) - λ(μ - κ) = Ο � (μ - κ)(μ - λ) = Ο � μ=κ ή μ=λ β) Επειδή το γραμμικό σύστημα έχει μοναδική λύση θα ισχύει D :;t Ο επίσης από την (Ι) και το (α) ερώτημα προκύπτουν : D = D Y ή D, = D Y . Επομένως : D Με D = D Y έχουμε y0 = DY = 1 οπότε: 3χ 0 + 2y 0 = 4 � 3χ 0 = 2 � χ 0 = �3 . Άρα (x0 , y0 ) = �3 , 1 Με D, = Dy και D :;t: O έχουμε x0 = y0 οπότε: 3χ0 +2y0 =4�5χ0 = 4�χ0 = �5 . Άρα (x0 , y0 ) = i5 , i5

( ) ( )

Τ Ρ Ι Ω Ν Υ Μ Ο Δ Ε Υ Τ Ε ΡΟΥ Β Α Θ Μ Ο Υ

δευτέρου βαθμού μιας μεταβλητής χ λέγεται κάθε πολυώνυμο δευτέρου βαθμού που έχει μορφή : f(χ) = αχ 2 + βχ + γ, α,β,γ Ε R, α :;t: Ο Ο ρ ισμός : Π ο λυωνυ μική συνάρτη σ η δ ευτέρου βαθμού είναι η συνάρτηση με τύπο f(x) = αχ 2 + βχ + γ, α, β, γ Ε R, α :;t Ο και πεδίο ορισμού το R. Ο ρισμός : Ρίζ α τριωνύμου f(x) = αχ 2 + β χ + γ , λέγεται κάθε ρίζα της εξίσωσης αχ 2 + β χ + γ = Ο . Ο ρ ισμός : Δια κ ρ ίνου σ α ενός τριωνύμου ονομάζουμε την παράσταση Δ = β 2 - 4αγ . Το Ε Ι ΔΟΣ των ριζών ενός τριωνύμου εξαρτάται από το πρόσημο της διακρίνουσας , (αντίστοιχα θέματα έχουμε μελετήσει στο είδος ριζών της αχ 2 + βχ + γ = Ο , α, β, γ Ε R, α :;t Ο ) Ο ρ ισμός : Τρ ιώνυ μο

I . Μ ο ρφ ές τ ρ ιων ύ μου

Παρατηρούμε ότι : f(x) = ax 2 + βχ + γ τότε 4αf(χ) = 4ax 2 + 4aβχ + β 2 - β 2 + 4αγ � 4αf(χ) = (2αχ + β) 2 - (β 2 - 4αγ) � 4αf(χ) = 4α 2 (χ + 1._ ) 2 - Δ 2α � 4αf(χ) = 4α 2 cx + 1.._ ) 2 � f(x) = α (χ + 1.._ ) 2 - ( Ι ) 2α 4α 2α 4α Από την μελέτη της ( 1 ) προκύπτουν τα επόμενα:

-�]

[

α.

-�]

[

[

- [ �)']

[

: [ :) ]

Αν Δ<Ο το τριώνυμο γράφεται : f(x) = α (χ + t/

Στην περίπτωση αυτή το τριώνυμο έχει μορφή γινομένου του α επί ένα άθροισμα δύο τετραγώνων. Έτσι όταν Δ<Ο το Τριώνυμο δεν μηδενίζεται στο R και δεν αναλύεται σε γινόμενο παραγόντων πρώτου βαθμού. β. Αν Δ=Ο το Τριώνυμο γράφεται : f(χ) = α(χ + 1._) 2 ή f(χ) = α(χ - ρ) 2 αφού ρ 1 = ρ -ο = - 1._ = ρ . 2α 2α Στην περίπτωση αυτή το τριώνυμο έχει Μορφή γινομένου του α επί ένα τετράγωνο πρωτοβάθμιου ως προς χ Διωνύμου. ' γ. Αν Δ>Ο το Τριώνυμο γράφεται : f(x) = α (χ + )' α

[[

)[

)]

= α χ + 1._ + JΔ x + l_ - JΔ τότε f(χ) = a(χ - ρ 1 )(χ - ρ 2 ) , αφού ρ 1 2 = -β ± JΔ . 2α 2α 2α 2α 2α Στην περίπτωση αυτή το τριώνυμο έχει Μορφή γινομένου του α επί μία διαφορά δύο τετραγώνων. ·

Έτσι όταν Δ>Ο το τριώνυμο αναλύεται σε γινόμενο παραγόντων . 1 1 . Π ρ όσ η μο των τιμών της συνάρτη σ η ς : f(x) = αχ 2 + βχ + γ , α, β, γ Ε R, α :;t Ο Θα μελετήσουμε το πρόσημο του f (x) για τις διάφορες τιμές του χ Ε R .Το πρόσημο της Διακρίνουσας επηρεάζει το πρόσημο του γινομένου af(x) όταν χ Ε R . ΕΥΚΛΕΙΔΗΣ Β' 75 τ.3/2 1


Μαθη ματικά για την Α ' Λυκείου

Έτσι γνωρίζοντας τις Μορφές του Τριωνύμου διακρίνουμε τις παρακάτω περιπτώσεις: α ) Αν Δ<Ο προκύπτει : af(x) = a ' χ + � ' + �:, > Ο για κάθε χ ε R , αφού -Δ>Ο.

[(

)

]

Στην περίπτωση αυτή το τριώνυμο για κάθε χ Ε R έχει ίδιο πρόσημο με το α. 2 11 J Αν Δ =Ο προκύπτει : af(x) = a 2 χ + � > Ο για κάθε χ Ε R - . α Στην περίπτωση αυτή το τριώνυμο για κάθε χ Ε R έχει ίδιο πρόσημο με το α, εκτός από την τιμή χ = _ l_ (Διπλή ρίζα ) που το μηδενίζει. 2α Αν Δ>Ο προκύπτει : af(x) = a 2 (x - ρ 1 )(χ - ρ 2 ) με ρ 1 < ρ 2 . Στην περίπτωση αυτή διαπιστώνουμε ότι : � ) Αν χ < ρ 1 ή χ > ρ 2 .Δηλαδή αν χ Ε (-οο, ρ 1 ) υ (ρυ +οο) έχουμε (χ - ρ 1 )(χ - ρ 2 ) > 0 δηλαδή: af(x) > O που σημαίνει ότι το τριώνυμο f(x) έχει ίδιο πρόσημο με το α όταν ο χ παίρνει τιμές μικρότερες της μικρότερης ρίζας ή μεγαλύτερες της μεγαλύτερης ρίζας. u i ) Αν ρ ι < χ < ρ δηλαδή αν χ Ε (ρ ι , ρ ) 2 2 - t l---- -- i τότε (χ - ρ 1 )(χ - ρ 2 ) < Ο δηλαδή 1 f( x ) ομόση μο του d�- ετερόσημο του α 'Γο μόσημο του α af(x) < Ο που σημαίνει ότι το τριώνυμο f(x) έχει αντίθετο πρόσημο του α όταν ο χ παίρνει τιμές ανάμεσα στις ρίζες. Εποπτικά :

( )

<[--x__l__

{ :}

___ __

.

---- -.>

:\ ρ •�. σ ψ υ l: υ ιιR π ψ ι! σ μ υ τ υ

Ι ) Η συνθήκη Δ<Ο εξασφαλίζει ότι για κάθε χ Ε R ισχύει: f (χ) =F Ο 2) Η συνθήκη Δ<Ο εξασφαλίζει ότι για κάθε χ Ε R το τριώνυμο έχει σταθερό πρόσημο. Έτσι: Γίνεται θετικό όταν α>Ο και αρνητικό όταν α<ο.

Δίνεται το τριώνυμο φ ( χ) = ( 3 - λ ) χ 2 + 2 ( 1 + λ) χ + λ2 + λ + 2 αν για κάθε λ =F 3 ισχύει φ ( χ ) > Ο . Να μελετήσετε την μονοτονία της συνάρτησης f ( χ) = ( 1 - λ ) χ + 7 . Επειδή για κάθε λ Ε R - {3} ισχύει φ(χ) > Ο θα είναι: 3 - λ > Ο (l ) και Δ < Ο (2), ' Εχουμε: ( l ) :::::> λ < 3 . Όμως Δ [ 2 ( Ι + λ ) Τ - 4 · (3 - λ) ( λ2 + λ + 2 ) = 4 ( Λ: - 3λ2 + 7λ - 5) (1). Έτσι (2) :::::> Λ: - 3λ2 + 7λ - 5 < Ο :::::> Λ: - 2λ2 - λ2 + 5λ + 2λ - 5 < Ο :::::> λ2 ( λ - Ι) - 2λ( λ - Ι ) + 5 (λ - Ι) < ο :::::> (λ - Ι) ( λ2 - 2λ + 5 ) < ο :::::> λ - ι < ο :::::> λ < ι ( αφού το λ2 - 2λ + 5 έχει Δ1 = - ι 6 < Ο , επομένως για κάθε λ Ε R ισχύει 'Κ - 2λ + 5 > Ο ) Άρα: ( ι ),(2) :::::> λ < Ι , οπότε η συνάρτηση f (χ) = (Ι - λ) χ + 7 είναι γνησίως αύξουσα.

=

Α ρχικά παραγοντοποιούμε το Ρ (χ) , έτσι Ρ ( χ) = Α (χ) · Β (χ) · Γ (χ) · ... · Φ (χ) κατόπιν προσδιορίζουμε το πρόσημο κάθε παράγοντα ξεχωριστά και τέλος το πρόσημο του Ρ ( χ) . Α ( χ) · Β ( χ) · Γ ( χ) · ... · Φ ( χ ) ;:::: Ο Με όμοιο τρόπο λύνουμε την aνίσωση ή Α ( χ) · Β ( χ) · Γ ( χ) · ... · Φ ( χ) � Ο .

Να προσδιορίσετε το πρόσημο της παράστασης: Ρ( χ) = χ4 ( χ 2 - 2χ + 1 ) ( χ2 - 3 ) ( χ2 - χ + 1 ) ( χ2 - 5χ + 6) . Θα προσδιορίσουμε το πρόσημο κάθε παράγοντα του Ρ ( χ) . χ 4 > Ο <=> χ Ε JR * . ΕΥΚΛΕΙΔΗΣ Β' 75 τ.3/22


Μ αθη μ ατικά για την Α ' Λυκείου

χ 2 - 2 χ + 1 > ο <=> (χ - Ι) 2 > ο <=> χ * 1 . το χ 2 - 3 > ο <=> ( χ + .J3 ) ( χ - .J3 ) > ο <=> χ < -.J3 ή χ > .J3 : Το χ 2 - χ + 1 έχει διακρίνουσα Δ = -3 < Ο , άρα χ 2 - χ + Ι > Ο για κάθε χ ε IR . Το χ 2 - 5χ + 6 έχει διακρίνουσα Δ = 1 > Ο και ρίζες ρ 1 = 3 , ρ 2 2 . Έτσι έχουμε: χ2 -5χ + 6 > 0<::::> χ < 2 ή χ > 3 . τέλος για να βρούμε το πρόσημο του Ρ ( χ) κατασκευάζουμε τον πίνακα : =

-

--

-.J3 �-4 �� -��=� � + ---=--- -r χ

_ - � - -_ _

-CIJ

+α:>

3

2

ο

+__L_±_�� -· + J + ι + +__-'fCD"----_+_--+- - +__ li + I + + + + ---f---·------ ------ φ -�--+--: + + + :;. ι + +

_____

_

__ill_

__ _ _ +__

+ χ 2 - 2χ + 1 + 2 χ -3 �_x + I + +6 x' _ :_-_-_c-:_D-=_-=_-:_-_-__Jc+-�----_:-=_-=_-:_φ-=_-=_-:_.......1. ) --:----�--)_-_-ι

-

+--__ _ __ _____ ____

_::

_

j-

_

__

.

_ _

__ __ _ _ _ __ ____ _ _

_ _

___

-�-�-:

__

, Α( )

Α( )

-

-

��--= �

__

___

- -1

__

---1 -�L__:_�,

ι;.) Ανισώσεις της μορφής : Β ( χχ ) � Ο (1) η Β ( χ) � Ο (2). χ Με Β (χ) 7= 0 έχουμε: ( l ) <=> A ( x ) · B (x) � O <::::> ... (2) <=> Α ( χ) · Β ( χ) � Ο <::::> ...

��:� � Γ ( χ ) (1) ή ��:� � Γ ( χ ) (2).

!)\ Κλασ μ ατικές Ανισώσεις της μορφής:

Απαλοιφή παρανομαστών κάνουμε μόνο αν γνωρίζουμε το πρόσημο της παράστασης Β ( χ) 7: Ο . ' Αν δεν γνωρίζουμε το πρόσημο της παράστασης Β ( χ) μεταφέρουμε όλους τους όρους της ανίσωσης στο I ο μέλος, οπότε καταλήγουμε σε ανίσωση της μορφής /\ :t κ �-α: ι-ι

, χ - 3 χ+ 2 Ν α λυ θ ει, η ανισωση : -- -- �

---=2 --

χ+2 χ 3 χ -χ-6 -

: ��:� � Ο η ��:� � Ο .

(I)

--

χ - 3 - χ + 2 - 10 � ο (Ι) · · ανισωση ορι' ζ εται μονο για χ 7: -2 και χ :;t 3 . τοτε: <::::> χ + 2 -χ - 3 χ2 - χ - 6 2 2 - 3) - + 2 - \ Ο > Ο <::> χ 2 - 6χ + 9 - χ 2 - 4 - 4χ > Ο <::> -1 0χ + 5 > Ο <::::> (χ χ + 2(χ ) ( )(χ - 3) (χ + 2) (χ - 3 ) ( χ + 2) { χ - 3) <=> ( - 1 Οχ + 5 )( χ + 2 )(χ - 3 ) � Ο (11). Κατασκευάζουμε τον πίνακα προσήμων :

Η

·

-

-

-

-

--

-

-

- -χ

-

-

-

-\ Οχ + 5 I χ+2 χ-3 Ρ ( χ) -�-

-00

-

'

I�

[ )

- -- - -

-

+ +-

-

-

-

-2 -

-

-

-- -

-

-

+ + -

Ι/2 CD ' I

()

Άρα: (Ι) <::::> χ ε (-οο,-2) u � ,3 . ΕΥΚΛΕΙΔΗΣ Β' 75 τ.3/23

+ +

3

I

-

tI '

+ + -

I

-------i


Μαθηματικά για την Α ' Λυκείου

ΛΣ Κ Η Σ Η

Να μετατρέψετε το τριώνυμο 4ου βαθμού : φ( χ) = 4a 2 x 4 - (β 2 + 4α 2 γ 2 )χ 2 + β 2 γ 2 , α, β, γ Ε R,α :;e Ο σε γινόμενο πρωτοβαθμίων παραγόντων ως προς χ. Λ ίισ η

Αν θέσουμε χ 2 = y 2 0 παίρνουμε το τριώνυμο: φ(y) = 4a 2 y 2 - (β 2 + 4α 2 γ 2 )y + β 2 γ 2 που έχει Διακρίνουσα : Δ = (β2 4α2 γ 2 ) 2 _ 4 . 4αz β 2 γ 2 = β4 + 16α4γ4 + Sα2 β 2 γ 2 _ 16α2 β 2 γ 2 = β4 + 16α4 γ4 _ 8α2 β 2 γ2 = ( β 2 _ 4α2 γ2 ) 2 2 0 β 2 + 4α z γ z ± �( β 2 _ 4α z γ z ) 2 β z + 4α 2 γ 2 ± ( β z _ 4α 2 γ 2 ) , Τοτε Υ ι 2 = 8α 2 8α -' Δηλαδή Υ ι = 4αβ 22 , y 2 = γ 2 άρα φ(y) = 4α 2 (y - 4αβ 2, )(y - γ 2 ) = ( 4α 2 y - β 2 )(y - γ 2 ) έτσι 2 2 z φ(χ) = ( 4α z χ - β )( x z - γ ) ή φ(χ) = (2aχ + β)(2aχ - β)(χ + γ)(χ - γ) . +

·

Λ Σ Κ Η Σ Ι-1

Να δείξετε ότι η εξίσωση : χ 2 - 2(λ - l)x + λ - 3 = Ο (1) για κάθε λ Ε R έχει ρίζες πραγματικές και άνισες. Κατόπιν να προσδιορίσετε τις τιμές του λ Ε R ώστε η εξίσωση να έχει ρίζες ομόσημες. Λίιση

α) Υπολογίζουμε την διακρίνουσα : Δ = 4(λ - 1) 2 - 4(λ - 3) = 4 [ λ2 + 1 - 2λ - λ + 3 J

= 4(λ2 - 3λ + 4) ,το πρό σημο της Δ προσδιορίζεται από το πρόσημο του τριωνύμου λc - 3λ + 4 έχει Δ 1 = 9 - 1 6 = -7 < Ο και α = 1 > Ο . Άρα για κάθε λ Ε R , λ2 - 3λ + 4 > Ο οπότε Δ > Ο για κάθε λ Ε R που σημαίνει ότι η ( I ) έχει ρίζες ρ 1 , ρ 2 πραγματικές και άνισες. β) Άρα: ρ 1 , ρ 2 ομόσημες <::::> Ρ > Ο <::::> λ - 3 > 0 <::::> λ > Ι <::::> λ > 3 που

ι\ Σ Κ Η Σ Η

Να βρείτε τις τιμές του a Ε R ώστε το τριώνυμο: f(x) = (3 - α)χ 2 - (7 - 4α)χ + 2 + α να γίνεται θετικό . . \ ίι σ η

Έχουμε Δ = [ -(7 - 4α)] 2 - 4(3 - α)(2 + α) = 49 + 16α 2 - 56α - 4(6 + 3α - 2α - α 2 ) = 49 + 1 6α 2 - 56α - 24 - 12α + 8α + 4α 2 = 20α 2 - 60α + 25 Για να γίνεται το τριώνυμο f(x) > Ο για κάθε χ Ε R πρέπει και αρκεί 3 - α > Ο (1 ) και Δ < Ο (2). Έτσι : (1 ) <::::> α < 3 και (2) <::::> 20α 2 - 60α + 25 < 0 <::::> 4α 2 - 1 2α + 5 < 0 , (3). Αλλά: Δ 1 = (-12) 2 - 80 = 144 - 80 = 64 , α 1 -, = 1 28± 8 τότε α 1 = 22 , α 2 = _!_2 Άρα: (3) <::::> 4(α - 22 )(α - _!_2 ) < 0 <::::> α Ε _!_2 , 22 . Τελικά: (1) , (2) <::::> α E _!_2 , 22 n (-oo,3) <::::> a E _!_2 , 22 .

( J

( J

( J

Λ Σ Κ Η l: Η

Δίνεται το τριώνυμο f(x) = 3χ 2 - (2λ + 3)χ + λ, λ Ε R (Ι) Να βρείτε τις τιμές του λ Ε R ώστε η παράσταση ρ; + ρ; να παίρνει την ελάχιστη τιμή της . Λίιση

Για να έχει η (Ι) ρίζες πρέπει και αρκεί Δ 2 Ο , όπου Δ η διακρίνουσα της 2λ 2λ +3 2 -2λ + 3 , ρ 1 · ρ = λ . Αν y = ρ 21 + ρ 2 τοτε ' ' εχουμε ' 2 = 2ρ ρ = ρ + ρ ) y ρ1 + ρ2 = ( Τοτε · 1 1 3 2 2 2 3 2 3 3

( J

ΕΥΚΛΕΙΔΗΣ Β' 75 τ.3/24


:

:

Μαθηματικά για την Α ' Λυκείου

4λ2 + + 1 2λ 2 λ 4λ 2 + 9 1 2λ - 6λ = , δηλαδή: 4λ2 + 6λ + 9 9y = 0 ( 1 ). 3 Για να έχει η (I) ρίζα ως προς λ Ε R πρέπει και εκεί Δ ι � Ο (2). ' Εχουμε: (2) <=> 6 2 - 4 · 4(9 - 9 y) � Ο <=> 4y - 3 ;:::: Ο <=> y � � . 4 Επομένως η ελάχιστη τιμή της παράστασης είναι y = � . 4 � Για y = είναι Δ ι = Ο , οπότε από την ( Ι ) έχουμε λ= - __i_ = - � . 4 2·4 4 Για λ = - � < Ο έχουμε αγ=3λ<0, οπότε Δ=β2 -4αγ>Ο. Άρα η τιμή λ = - � είναι δεκτή και είναι η τιμή 4 4 για την οποία ελαχιστοποιείται το ρ � + ρ � . =

_

_

ΆΣ Κ Η Σ Η

Να προσδιορίσετε τις τιμές του κ Ε R ώστε η εξίσωση : ρίζες αρνητικές.

( κ - 3)χ 2

+ (κ - l )x + (κ + 2) = Ο να έχει δύο

Λ ίJ ση

Για να έχει δύο ρίζες αρνητικές πρέπει και αρκεί: α.:Ο, Δ ;:::: Ο, Ρ > Ο, S < Ο . Έχουμε: α.:Ο<:::>κ;z:3 . Τότε: Δ � Ο <=> (κ - 1) 2 - 4(κ - 3)(κ + 2) ;:::: 0 <=> -3κ 2 + 2κ + 25 ;:::: Ο I - 2 Ji9 1 + 2 Μ . <:::> 3 Κ 2 - 2 Κ - _? )- :ς 0 <:::> Κ Ε , 3 3 κ+? Ρ > Ο <=> --- > Ο <=> (κ + 2)(κ - 3) > Ο <=> κ Ε (-oo, -2) u (3, +oo) . κ-3 κ-] S < Ο <:::> --- < 0 <:::> (1 - κ)(κ - 3) < 0 <=> ( κ - 1) ( κ - 3) > 0 <=> K E (-oo, I) u (3, +oo) . κ-3 Τ ο σύνολο τιμών του κ είναι η τομή των συνόλων λύσεων των τεσσάρων ανισώσεων. I - 2M _ 1 + 2 Ji9 , 2 υ 3, . Ά ρα.. Κ Ε 3 3

[

( J

[

ΆΣ Κ Η Σ Η

]

]

Ν α βρείτε τα χ Ε R ώστε να ορίζεται το κλάσμα y = χ2 - 5χχ + 9 Κατόπιν γι' αυτούς τους πραγματικούς αριθμούς να δείξετε ότι το κλάσμα παίρνει τιμές στο διάστημα 1\ , 1 . •

[- ]

Λ ίJ ση

Το τριώνυμο g(x) = χ 2 - 5χ + 9 έχει Δ = - 1 1 < Ο άρα για κάθε χ Ε R ισχύει g(x) > Ο . Δηλαδή το κλάσμα ορίζεται για κάθε χ Ε R . χ Τότε y = <=> yx 2 - 5yx + 9 y - x = 0 <=> yx 2 - (5y + 1)x + 9y = O ( 1 ) x- - Sx + 9 Αν y = Ο τότε: ( 1 ) <=> Οχ 2 - χ = Ο <=> χ = Ο . Άρα η y = Ο είναι μία τιμή του κλάσματος. Αν y ,: Ο , επειδή χ Ε R πρέπει Δ ;:::: Ο. Όμως Δ � Ο � [ -(5y + 1) ] 2 - 4y9y � Ο 2 � 25y 2 + 1 + l O y - 36y ;:::: Ο � 1 1/ - 1 O y - 1 ::; Ο (2). Το τριώνυμο φ(y) του πρώτου μέλους της (2) έχει ?

διακρίνουσα: Δ = ( - 1 0) 2 - 4 · I I · ( -1) = 1 44 > Ο και ρίζες Υ ι = 1, y 2 = _ __!_ . 'Ετσι: 11 (2) <=> __!_ ::; y ::; 1 . Άρα το κλάσμα παίρνει τιμές στο διάστημα 11 _

ΕΥΚΛΕΙΔΗΣ Β' 75 τ.3/25

[-_!_,Ι], αφού Ο [-_!_,ι] . 11 11 Ε


Μ α θη ματικά για την Α ' Λυ κείου

Γ � · � . :- i:ι' \,' fι

i

ι '-·

'

11

•· '

'

;J , · ; '• '

Σπύρος Κ αρδαμίτσης - Σωτήρης Ε . Λουρίδας «.

. . Γραμμή δε μήκος απλατές. Επιφ άνεια εστίν, ό μήκος και πλάτος μόνον έχει.

Σχήμα εστίν το υπό τινος ή τινώ ν όρων περιεχόμενον . . . »

( Ευκλείδης 365 π . Χ .)

του κύκλου με τις πλευρές ΒΓ, ΑΓ και ΑΒ Δίνεται κύκλος κέντρου Ο. Έστω ΑΒ χορδή αντίστοιχα και Ρ η τομή της ΙΔ με την ΕΖ, να του κύκλου αυτού και Ι το μέσο της . Θεωρούμε δείξετε ότι η ΑΡ διέρχεται από το μέσο της ΒΓ ΓΔ, ΕΖ τυχαίες χορδές του κύκλου αυτού που Θεωρούμε την παράλληλη ΚΛ από το σημείο διέρχονται από το Ι. Αν Μ, Ν είναι τα σημεία τομής των ΓΖ, ΕΔ με την ΑΒ αντίστοιχα, να Ρ στην πλευρά ΒΓ του τριγώνου (Κ σημείο της ΑΒ, Λ της ΑΓ) , επομένως αρκεί να αποδείξουμε αποδειχθεί ότι lM = ΙΝ Λ

Λ

Λ

Λ

Είναι Ζ = Δ και Γ = Ε ως εγγεγραμμένες σε ίσα τόξα γωνίες, συνεπώς τα τρίγωνα ΖΓΙ και ΙΕΔ ΓΖ ΕΔ , ομοια , ειναι αρα = - ( I) , ΓΙ ΕΙ

ότι το σημείο Ρ είναι το μέσο του ΚΛ ή το τρίγωνο ΙΚΛ είναι ισοσκελές. Προς τούτο αρκεί να δείξουμε με βάση σχήμα που ακολουθεί, ότι χ = y Λ

Λ

Το τετράπλευρο ΙΡΚΖ είναι εγράψιμο αφού ΙΖΚ = ΙΡΚ = 90° . Επίσης το τετράπλευρο ΙΛΕΡ Λ

Λ

είναι εγραψιμμο αφού I R L = ΙΕΛ = 9 0" Άρα χ = Χ ι και Υ = Υ Όμως το τρίγωνο ΙΕΖ είναι ισοσκελές οπότε Χ ι = y 1 άρα και χ = y δηλαδή το τρίγωνο Ι Κ Λ είναι ισοσκελές. Λ

Φέρνουμε τα aποστήματα ΟΚ και ΟΛ των χορδών ΖΓ και ΔΕ, τότε είναι : ΓΖ ΕΔ ΓΚ = και ΕΛ = (2) 2 2 Τ α τρίγωνα Κ Γ Ι και ΙΕΛ έχουν τις γωνίες Γ και Ε ίσες και από τις σχέσεις ( 1 ) και (2) έχουμε ότι: ΓΖ ΕΔ ΓΚ ΕΛ - = - :::? - = ΓΙ ΕΙ ΓΙ ΕΙ άρα είναι όμοια οπότε ω = φ. Από τα εγγράψιμα τετράπλευρα ΚΟΙΜ και ΟΛΝΙ έχουμε ω = ωι και φ = Ψ ι -

-

Λ

Λ

Λ

Λ

Λ

επομένως είναι: ω 1 = φ 1 , οπότε τα τρίγωνα ΟΙΜ και ΟΙΝ είναι ίσα, άρα ΜΙ = ΙΝ.

Δίνεται τρίγωνο ΑΒΓ και ο εγγεγραμμένος του κύκλος ( Ι, ρ). Αν Δ, Ε, Ζ τα σημεία επαφής

Λ

Λ

Λ

Λ

Λ

1 Ό

Λ

ι

Δ

Λ

Λ

Δίνεται ορθογώνιο τρίγωνο ΑΒΓ ( Α =90°) και ο εγγεγραμμένος κύκλος του. Αν Δ, Ε, Ζ είναι τα σημεία επαφής του κύκλου με τις πλευρές ΒΓ, ΑΓ και ΑΒ αντίστοιχα και ΕΡ η κάθετη στην ΖΔ, να αποδείξετε ότι: α) ΑΡΕ =45° β) Η ευθεία ΡΓ είναι μεσοκάθετος της ΔΕ γ) ΑΡΓ =90° α) Τα ΑΖ και ΑΕ είναι εφαπτόμενα στο κύκλο

ΕΥΚΛΕΙΔΗΣ Β' 75 τ.3/26


Μ αθη ματικά για την Α ' Λυκείου

τμήματα, συνεπώς είναι ίσα, άρα το τρίγωνο ΑΖΕ Επομένως ισοσκελές. ορθογώνιο είναι ΑΕΖ = ΕΖΑ = 45" . Το τετράπλευρο ΑΕΡΖ είναι εγγράψιμο γιατί Α+ Ρ 9 0" + 90" 1 80° άρα ω = ΑΡΕ = ΑΖΕ 45° Λ

Λ

Λ

=

Λ

Λ

=

=

Λ

Λ

Λ

Γ ι = Ζ ι . Αλλά

άρα είναι εγγράψιμο, επομένως Λ

= Α ( θεώρημα χορδής και εφαπτομένης) επομένως Ζ ι Α ( 1 ) Γι

Λ

Λ

=

Λ

Λ

Στο τετράπλευρο ΑΕΗΖ είναι Ε+ Ζ 90" + 90" = 1 80", άρα είναι εγγράψιμο, οπότε ΕΗ Ζ+ Α = 1 80° και λόγω της ( I ) έχουμε Λ

Λ

Λ

Λ

ΕΗΖ+ Ζι = 1 80" άρα ΔΖ // ΕΗ. Όμοια αποδεικνύεται ότι ΔΕ // ΗΖ, επομένως το τετράπλευρο ΕΗΖΔ είναι παραλληλόγραμμο. = �·· )

β) Είναι ΑΕΖ = Ε.2 45'' ( απ ό το θεώρημα χορδής και εφ α πτομ έγης). συνεπώς το τρίγωνο ΔΡΕ είναι ορθογι;η ·ιο κ αι ισοσκελές, άρα ΡΕ = Ρ Δ. Ισχύει ότι ΓΕ = Γ ..1 ( σ αν εφ απτόμενα τμήματα από το σημείο Γ στο" ίδιο κύκλο), επομένως τα σημεία Ρ κ αι Γ ισαπέι.ου" απ ό τα άκρα του τμήματος ΕΔ, πράγμ α πο υ σημ αίνει ότι η ευθεία ΡΓ είναι μ εσοκάθετο; το υ ευθύγραμμου τμήματος ΕΔ. γ) Επειδ1Ί το τρίγωνο ΔΡΕ είναι ορθογώνιο και ισοσκελές η ΡΓ ως μεσοκάθετος είναι και διι.οτ ό μος της γωνίας ΕΡΔ άρα ΕΡΓ =45° Λ

=

Λ

iT I

Λ

Λ

Θεωρούμε τρίγωνο ΑΒΓ ,ώστε Β- Γ = 90° . Αν το ύψος ΑΔ του τριγώνου ΑΒΓ τέμνει στο σημείο Ε την εφαπτομένη του περιγεγραμμένου κύκλου του τριγώνου αυτού στην κορυφή Γ, να συγκριθούν τα ευθύγραμμα τμήματα ΕΑ και ΕΓ. Προφανώς το τρίγωνο ΑΒΓ είναι αμβλυγώνιο στην κορυφή Β. Έστω Ζ σημείο της πλευράς ΑΓ τέτοιο ώστε ΒΖ.lΒΓ.

Λ

Δ ίν εται τρ ίγωνο ΑΒΓ εγγεγραμμένο σε κύκλο. Ο ι ε φ απτό μενες του κύκλου στα ση μ εία Β και Γ τέμνονται στο Η. Από το σημείο Η φέρνουμε ΗΕ .l ΑΒ, ΗΖ .l ΑΓ και ΗΔ .l ΒΓ. Να αποδείξετε ότι το τετράπλευρο ΕΗΖΔ είναι παραλληλόγραμμο. Λ

Λ

Λ

Παρατηρούμε ότι: ΔΑΒ = ΑΒΖ Γ . Αυτό σημαίνει ότι η ΕΑ είναι εφαπτομένη στον κύκλο (γνωστή πρόταση ), επομένως ΕΑ ΕΓ. (Εφαπτόμενες αγόμενες από το σημείο Γ στον ίδιοι κύκλο). =

Θεωρούμε κυρτή γωνία xOy και σημεία Α, Γ της πλευράς Οχ διαφορετικά της κορυφής Ο με ΟΑ ΟΓ και Β, Δ της Oy διαφορετικά της κορυφής Ο με ΟΔ 08. Έστω Ε το σημείο τομής των ΑΒ και ΓΔ. α) Να αποδειχθεί ότι οι περιγεγραμμένοι κύκλοι στα τρίγωνα ΕΑΓ και ΕΔΒ δεν είναι δυνατό να εφάπτονται. <

Λ

Λ

Στο τετράπλευρο ΔΓΖΗ είναι Δ+Ζ= ιχ1' +'Χ1' 1 80°, =

ΕΥΚΛΕΙΔΗΣ Β' 75 τ.3/27

<


Μ αθη ματικά για την Α ' Λυκείου

β) Αν Μ το δεύτερο σημείο τομής τους , αποδείξατε ότι το τετράπλευρο α ΓΜΔ είναι εγγράψιμο.

Λύ σ η :

α) Αν οι κύκλοι εφάπτονταν τότε στο σημείο επαφής τους Ε θα θεωρούσαμε την κοινή τους εξωτερική εφαπτομένη ΖΕΗ (Η προς το μέρος του 0), οπότε θα είχαμε ότι οι ευθείες ΑΓ και ΔΒ είναι παράλληλες (γιατί;) πράγμα άτοπο, αφού τέμνονται στο σημείο Ο.

κυρτό τετράπλευρο είναι εγγράψιμο, είναι η εξής: Δύο γωνίες που οι κορυφές τους είναι δύο διαδοχικές κορυφές του τετραπλεύρου και σχηματίζονται από τις δύο διαγωνίους και από τις δύο απέναντι πλευρές είναι ίσες. Άσκ η σ η 7''

Θεωρούμε ισοσκελές τρίγωνο ΑΒΓ (ΑΒ=ΑΓ) και τον περιγεγραμμένο του κύκλο (0, ρ). Έστω Η σημείο της πλευράς του ΑΒ ώστε

OH .l AB.

ο ·:

'

z

Η '

Στην προέκταση της ΟΗ παίρνουμε ΗΔ=ΟΗ. Θεωρούμε σημείο Ε της πλευράς ΒΓ, ώστε ΔΕ παράλληλη στην ΑΓ. Να αποδειχθεί ότι: ΔΟΕ 90° Λ

=

Λύ σ η :

- - -

β) Έστω Μ το δεύτερο σημείο τομής των περιγεγραμμένων κύκλων στα τρίγωνα ΕΑΓ και ΕΔΒ. Θεωρούμε την κοινή χορδή τους ΕΜ οπότε

Παρατηρούμε ότι το τετράπλευρο ΑΔΒΟ είναι ρόμβος οπότε η ΒΔ είναι ίση και παράλληλη προς την ΑΟ, γεγονός που οδηγεί και στην ισότητα ΔΒΓ = 9 0° ( 1 ) Θεωρούμε τώρα την διάμετρο ΒΟΖ οπότε ΖΑΒ = 9 0° πράγμα που σημαίνει ότι οι ΑΖ και Λ

Λ

Λ

Λ

Λ

παίρνουμε: ΑΓΜ = ΜΕΒ = ΜΔΒ , ισότητα που οδηγεί στο ότι το τετράπλευρο ΟΓΜΔ είναι εγγράψιμο σε κύκλο καθότι μία εσωτερική του γωνία ισούται με την απέναντι από αυτή εξωτερική του. ΣΧΟ Λ Ι Ο I : Το σημείο Μ ονομάζεται σημείο του Miquel. Τελικά οι περιγεγραμμένοι κύκλοι στα Δ Δ Δ Δ τρίγωνα Ε Α Γ, Ε Β Δ, Ο Γ Δ, Ο Α Β διέρχονται από το ίδιο σημείο. ΣΧΟ Λ Ι Ο 2 : Χρησιμοποιήθηκαν τα θεωρήματα i) Μία ικανή και αναγκαία συνθήκη, ώστε το κυρτό τετράπλευρο να είναι εγγράψιμο σε κύκλο είναι μία εσωτερική του γωνία να ισούται με την απέναντι εξωτερική του. ii) Μία ικανή και αναγκαία συνθήκη, ώστε ένα

Λ

Λ

ΔΟ είναι παράλληλες και επομένως ΖΑΓ = ΟΔΕ (οξείες γωνίες με πλευρές παράλληλες) Όμως Γ ΑΖ = ΓΒΖ � ΕΔΟ = ΓΒΖ = ΕΒΟ οπότε το τετράπλευρο ΔΟΕΒ είναι εγγράψιμο σε κύκλο ,άρα ΔΟΕ = ΔΒΕ = 1 80° (2). Από τις σχέσεις ( 1 ) και (2) παίρνουμε ΔΟΕ = 9 0 ο Λ

Λ

Λ

Λ

Λ

Λ

Λ

Λ

Σχό λ ιο :

Παρατηρήσαμε ότι ,από το ισοσκελές τρίγωνο ΑΒΓ έχουμε: ΑΟ κάθετη στην ΒΓ, οπότε ΔΒ κάθετη στην ΒΓ σαν παράλληλη στην ΑΟ.

ΕΥΚΛΕΙΔΗΣ Β' 75 τ.3/28


Μαθηματ ι κά γ ι α τη Β ' τάξ η του Λυκ ε ίου

Εκθετι κή-Λογαριθμική συνάρτηση

Κυριακοπούλου Κωνσταντίνα

Ο Ρ Ι Σ Μ ΟΙ-Χ Ρ Η Σ Ι Μ ΕΣ Ε Π ΙΣΗ ΜΑΝΣ Ε Ι Σ

Εκθετική συνάρτηση με βάση το θε­ τικό αριθμό α, με α * 1 ονομάζουμε τη συνάρτηση f : IR � IR με τύπο f(χ)=α χ . Προφανώς έχει σύνο­ λο τιμών το (Ο,+ οο ) . 2. Η γραφική παράσταση της εκθετικής συνάρτη­ σης f(x) = αχ είναι: Ι .ΟΡΙΣΜΟΣ

Ο < "· <

I

προς την y=x (διχοτόμος 1 ης και 3ης γωνίας αξό­ νων)

υ > Ι

2.5

1 .5

0.5

-2

-

15 .

-1

-0.5

0.5

1.5

3.Αν α> 1, τότε η f(x)= α χ είναι γνησίως αύξουσα, δηλαδή για χ ι < χ 2 ισχύει f(x ι ) < f(x 2 ) , ενώ αν Ο<α :;e 1 η f(x)= α χ είναι γνησίως φθίνουσα δηλα­ δή για χ ι < χ 2 ισχύει f(x ι ) > f(x 2 ) . 4.Μια εξίσωση της μορφής ( f(x) )g( x ) = 1 επαλη­ θεύεται για εκείνα τα χ για τα οποία ισχύει: f(x)= 1 ή g(x)=O και f(x) * Ο ή f(χ )=-1 και g( χ)=άρτιος 5. ΟΡΙΣΜΟΣ Λογαριθμική συνάρτηση με βάση το θετικό αριθμό α με α * Ι ονομάζουμε τη συνάρτηση f : (0,+ οο ) � IR με τύπο f(x)= logα χ :Εχει δε σύνολο τιμών το IR . •

6 . Αν α> Ι , τότε η f(x)=

logα χ είναι γνησίως αύ­ ξουσα, δηλαδή για Χ ι < Χ 2 ισχύει f(x ι ) < f(x 2 ) , ενώ αν Ο<α :;t: 1 η f(x)= logα χ είναι γνησίως φθί­ νουσα δηλαδή για χ ι < χ 2 ισχύει f(x ι ) > f(x 2 ) . 7 . Οι γραφικές παραστάσεις των συναρτήσεων f(x) = αχ και g(x)= logα χ είναι συμμετρικές ως

8.Για Ο<α * Ι και θ>Ο ,θ ι >Ο,θ 2 >Οκαι κ ε IR ισχύ­ συν τα παρακάτω: logα αχ = χ ι α οg. θ = θ logα Ι = Ο logα α = Ι logα (θ ι · θ 2 ) = logα θ ι + logα θ 2 •

• • • •

( :�)

logα

logα θκ = κ · logα θ

ΕΥΚΛΕΙΔΗΣ Β' 75 τ.3/29

= logα θ ι - logα θ 2


Μ α θη ματικά για την Β ' Λυκείου

9,Παρατηρήστε ότι: Αν χ · y>Ο,τότε log α (χ . y) = log α I χ . Υ Ι = lo g α I χ I + log α I Υ I καθώς και log α

[�) = logα �Y l = logα χ - logα I

Υ

I

I

IΥI .

Άρα logx 2 =loglxl + loglxl=2loglx Ι . Γενικά για κάθε ν Ε Ζ ισχύει : log α X 2 v = 2v log α I χ I ,με X ::f. Ο 1 Ο"Δεν πρέπει να ξεχνάμε τους περιορισμούς, όπου αυτοί απαιτούνται , κατά την λύση των λογαριθμι­ κών εξισώσεων και ανισώσεων.

Δίνονται οι συναρτήσεις f(x) = 5'

( J'

-(�)' + 2010 ,

[ 2' )

μένη του κοινού σημείου της γραφικής παράστα­ σης της g και του άξονα χ ' χ είναι η λύση της εξίlη 5 σωσης g (χ)=Ο.Με χ > έχουμε: 2 e2 - 5 e2 x - 5 =Ιη 1 <=> . 1 <=> g(x)=O <=> Ιη -e' + 1 e' + 1 e 2' - e ' - 6 = Ο <=> e ' + 2 · e ' - 3 =Ο <=>

[x )

)(

(

)

e ' =-2(αδύνατη) ή e ' =3 <=> x=ln3 , η οποία είναι

δεκτή αφού ικανοποιεί τον περιορισμό ,γιατί 2 1 η3= 1η 9> 1η 5 .Άρα η γραφική παράσταση της g τέ­ μνει τον χ ' χ στο Κ(lη3 ,0). 111

?\ σ κ η σ η

α) Να βρείτε για ποιες τιμές του α Ε JR η συνάρS-α ' είναι γνησίως αύξουσα στο JR τηση f(x)= α-2

5 h(x) = -31 - 3 ' και g(χ)= Ιn � ' e +1 α) Να αποδείξετε ότι η f(x) είναι γνησίως αύ­ ξουσα . �-4' β)Αν α= '!..2 να λυθεί η εξίσωση: f(2x)=�-6 2 -2 β) Να αποδείξετε ότι η h(x) είναι περιττή. 1- και 510 +-1γ) Να συγκριθούν οι αριθμοί � +512 α)Η συνάρτηση f είναι γνησίως αύξουσα στο JR 1024 δ) Να βρεθεί το σημείο τομής της γραφικής πα­ όταν και μόνο όταν -- ( Ι ). ράστασης της g(x) με τον χ'χ. >1 α)Έστω χ 1 , χ 2 Ε JR , με χ ι < χ 2 .Τότε αφού 5> 1 και I 1 ' 1 ' , - < 1 θ α ισχυουν 5'' < 5'' και - > 2 2 2 ' ' ' Ι ' Ι ' I ' I ' +2010 < - 2 => - 2 +2010<-

( )' ( )'

(2) => - (2) ( ) ' ( ) I '' 1 '' ' 2 ' ( ( 2 + 20 1 0 < 5' Άρα 5' ) + 20 1 0 )

Δηλαδή f(x ι ) < f(x 2 ) ,οπότε f(x) γνησίως αύξουσα. β) Το πεδίο ορισμού της h(x) είναι το JR . Άρα για κάθε χ Ε JR -χ Ε JR και

(l)-x -3-' = 3 ' -(l) ' =-h(χ),άρα περιττή. γ)Αποδείξαμε ότι η f(x) είναι γνησίως αύξουσα. Άρα: 9< 1 0 => f( 9 ) <f( I O) => 1 9 I ιο 5 9 - ( 2 ) + 20 1 0 < 5 ι ο _ ( 2 ) + 20 1 0 ::::> h(-x) =

Ι Ι _ => 9 + _1_ < ι ο -15 + 5 5 9 _ __ < 5 ι ο 5 12 1 024 1 024 512 δ)Ένας αριθμός αριθμός χ Ε JR ανήκει στο σύνολο e ' + 1 ::f. Ο ορισμού της g αν , και μόνο αν : e z ' 5 (I ). >0 e' + Ι !η 5 (1) <=> e 2 ' - 5 > Ο <=> e 2 ' > 5 <=> χ > .Η τετμη2 _

_

{-_

( 1 ) <=>

{

(

{--� : �

J

{�: � {�; α-2

-1 > 0

<=>

2 - 2α >0 α-2

α-2 α ::f. 2 α ::f. 2 <=> --'-<=> 2 < α < 5 2(5 -- α) --'- <=> 2(5 α)(α - 2) > 0 >0 α-2 β)Αν α= ,τότε f(x)= 3' .'Εχουμε: 2 f(2x) = � · 6' - · 4' <=> 3 2 ' = · 6' - · 4' <=> 2 2 2 2 6' 3 9 ' = -5 · 9' = -5 · 6' - -3 · 4' <=> -4' 2 4' 2 2 2

2

{

( ) (�)lx (�)' � [( ) ] [( ) ] ( ) () () () () () (%) (%}

+ =ο -�. <=> � = � . i ' - � <=> 2 2 2 2 2 22 ' 2 4 3 ' 3 ' 3 3 ' <=> 2 - ] . 2 - 2 = Ο <=> 2 - 1 = 0 3 ' 3

3 '

3' 3

3 '

1ή = = <=> 2 = 2 ή 2 -2 ο <=> 2 = 2 2 ' <=:> χ=Ο ή χ= Ι = ή λ σ •η� -c�η J ' �

Να λυθούν οι εξισώσεις :

ΕΥΚΛΕΙΔΗΣ Β' 75 τ.3/30

3

0


) 2 x- l ( 1 1 > + 2 α) 5 •

25 γ) 8 ' + 1 = συνχ

Μ αθηματικά για την Β ' Λυκείου

=

625 β) 13 ' - 14Ji3 ' + 13 = Ο δ) (χ 2 - 7Χ + 1 1)3 ' - 6 = 1

5 2 x +l . ( 215 ) 2 x -l = 625 <=> 5 2 x +l . 5 -2( 2 x - l ) =54 <::::> 52x+l . �χ+2 =� <::::> )2χ+3 = � <::::> -2Χ+3=4 <::::> χ�..!.2 β)13' -14J13 ' +13=0 <::::> J132' -14Jl3' +13=0 ' =ω m =ω <=> ( m' )2 -14Jl3' +13=0 <=> {ν13 ω2 -14ω+13=0 <::::> {Jl3' =ω <=> Jl3' = 1 ή J13' = 13 <=> χ=Ο ή χ=2 ω=1ήυ=13 γ) 8' + 1 = συνχ <=> 8' = συνχ -1 Για κάθε χ ε IR είναι 8' > Ο ενώ συνχ- 1 � Ο ,άρα η εξίσωση είναι αδύνατη . δ)Η εξίσωση ( x c -7χ ι ι) 3' - 6 = ι επαληθεύεται για εκείνα τα χ για τα οποία : χ2 -7χ+11=ι <:::2 :> x 2 -7x +l0=0 <::::> χ=2 ή χ=5 3χ-6=0 και χ -7χ + 11 Ο .Έχουμε 3χ-6=0 <=> χ=2. τιμή αυτή είναι δεκτή αφού 22 -7 . 2 + 1 1 = 4Η-ι4 + 1 1 =ι * ο . Έχουμε: χ 2 - 7χ +ι ι = -1 και2 (3χ-6)ι2=0άρτιος. χ2-7χ + 11 =-1 <=> χ -7χ+ <=> χ=3 ή χ=4. Για χ=3 είναι: 3χ-6=3 3-6=9-6=3 περιττός άρα η τιμή χ=3 απορρίπτεται Για χ=4 είναι 3χ6=3 4-6=6 άρτιος άρα η χ=4 δεκτή. Τελικά η ε­ ξίσωση έχει λύσεις τις χ=2 ή χ=5 ή χ=4 . α)

{

r;::; '

+

=ι.

·

.

·

,

(-)Ιχ+91-Ι

Να λυθούν οι ανισώσεις : l >α) 81 < 3 3 ' -8 β) 41 256 , 9 _ 2 γ) 24'� <{ 48J6) δ) log ( tog ( x 2 - 19χ + 100 ) } < 0

χ+ 4

x-s

+4)<0 <=>( χ)Ι +9Ι(-4,-2) (2,+οο) -3(χ( +2)(χ-2)(χ Ι Ι 9Ι Ι + ( ) ) 4 1 <=> -1 χ > -1 β) -41 χ > 4 256( ) 4 χ Επειδή � < 1 , η � είναι γνησίως φθίνουσα ο­ πότε θα είναι Ιχ+91- 1 <4 <=> Ιχ+91<5 <=> -5<χ+9<5 3<=> - ι 4<χ<-4 3 9 γ) 24 χ -� <(4 8-!6γ χ - <=> [(2J6) 2 T2 < [(2J6) τ-9 <=> ( 2-!6)l x -J < ( 2-!6)δχ -lΊ . Επειδή 2)6 > 1 η είναι γνησίως αύξουσα ,οπότε θα είναι 2-!6)'<6χ-27 (2χ-3 <=> 4χ>24 <=> χ>6 . ' δ) Εχουμε: \ o g (I o g (x 2 -19χ+100))<0 <=> χ 2 -19x+l00>0 <=> log (x 2 -19χ+100)>0 log (Iog ( x 2 -\9χ+100))<0 χ 2 -19χ +ι οο > ο <=> log ( χ 2 -19χ + 100) > log 1 log (log ( χ 2 -19χ + 100)) < log 1 χ2 -ι9χ+100>0 {χ--19χ+100>1 <=> χ--19χ+100>1 <=> lcg( x2 -19x+100) <1qs10 lqs( χ- -19χ + 100) <1 -19χ+99>0 <=> {χε.ΙR <=> {χχ22 -19χ+ <χ> < Ο (χ -9)(χ -10) <0 <=> χ ε (9, 10)

{

J

ε

υ

J

7

Να βρεθούν τα πεδία ορισμού των συναρτήσε­ ων: α) f(x) .J-5 2 ' + 6 5 ' - 5 =

·

( (

β) g(x)�Ιn(ln(x-1)) γ) h(x) = log ιog ( χ+ 9) j j δ) λ(χ)= 5χ + 13 3 Ιοg(ημχ + -2 )

JJ

81 χ+x-s4 < 33 ' -8 αληθεύει για χ -4 . Έ χουμε: 81 '+4 <33 '-8 <::::> (3 4 ) ' +4 < 3 3 ' - 8 <::::> 3 < Υ'-8 . Επειδή 3> 1 η 3 ' είναι γνησίως αύξουσα, οπότε θα α)Για να ορίζεται η συνάρτηση f(x) πρέπει και αρ­ ' --< 4(χ-χ+45) 3χ-8 <=> 4χ-20-(3χχ+4-8)(χ +4) <0 κεί: -5 2 '5'+6·5'Υ -520 (1) . 5' = Υ ειναι <=> { <=> 1 5' � 5 � (1) <=> { 2 4χ-20-3χ -12χ+8χ+32 1�y�5 +6y-520 -y ο < <=> <=> χ+4 1 <=> Ο � χ 1 . Επομένως το πεδίο <=> 5° � 5' �5 2 -3χ + 12 ορισμού της συνάρτησης f είναι το Α=[ Ο ,1] . --χ+4 <ο <=> -3(χ 2 -4)(χ + 4) <ο <=> =ι.

α)Η ανίσωση χ-5

χ -5

4( χ-5 ) χ+4

ΕΥΚΛΕΙΔΗΣ Β' 75 τ.3/31


Μ αθη ματικά για την Β ' Λυκείου

Για{ναχ -Ιορίζεται η συνάρτηση g(x) πρέπει και ο > αρκεί: In(x -Ι) > Ο (Σ) (Σ) <::::> { χχ -Ι> Ι > Ι <::::> { χχ >> Ι2 <::::> χ > 2. Επομένως το πεδίο ορισμού της συνάρτησης g είναι το Β=(2, +οο ). ορίζοεται η συνάρτηση h(x) πρέπει και γ) Για{ναχ +9> αρκεί: log� (χ+ 9) >ο (Σ) . (Σ) ς; { �:j�: +9)>ιοgj ι ς; { ::;: ι χ > -9 <::::> -9 <χ < -8. Επομένως το πεδίο ορι­ <::::> {χ<-8 σμού της συνάρτησης h είναι το Γ=(-9,- 8). δ) Για να ορίζεται η συνάρτηση λ(χ) πρέπει και ημχ + �2 >0 αρκεί: ( 3 ) (Σ). log ημχ +-2 :;t: O ο (Σ)<::::> ημχ +�>� <::::> ημχ > -�� <::::> ημχ :;t: ημ (-�) ημχ +-:;t:2 1 ημχ :;t: --2 Επομένως το πεδίο ορισμού της συνάρτησης λ είναι 7π π το Δ={χ ε IR / x:;t: 2κπ--6 και χ :;t: 2κπ+ -,6 κ ε Ζ } β)

!{

Α σ κ η ση

{

6 '1

Να υπολογιστεί η τιμή των παραστάσεων: log3 - lo g 27 + -51 log 243 + log 9 Α= log 180 - 21o g J6 'og, 4 3 'og ( τ ::g, :) J7 g, Γ=-----,-----,--Β= lo g e -log 7 ( lo g 3 3 7 ) · 61og•54 3

Λ \J σ η :

'•

-

'

Ι og 243 + log 9 l o 3 -l o 27 + -l g g 5 J6 α) Α= log 180-2lo g Ι log 3 -log 33 + -l5 og 3 5 + log 3 2 log 180-log J62 log 3-3log 3 +-5I · 5log 3 + 2log 3 log 180 -log 6

ι οο

3 lo g 3 ι ο g 3 = lo g 3 = ιolog180 g 6 ιοg9 30 ιοg 10 + ιοg 3 Ι + ιοg 3 ) (' 9 ) ( " 3 ο ι g β) Β= ι ο g 3 e τ = log 3 .!2_2 - 2 = ιοg 2 9 ) ) 19 ιοg2 3 - - ιog3 (-192 - ιιοοgg2332 - ιog3 (-2 2log2 3 2 1

_

log, J

ι οg

,

_

_

I

- log 7 4

72 3 2 · ιοg 7 ( 7ιοg3 3 ) 54 J4 · 9 18 1 --= 54 -54 =-3 •

7 log, J4

·9 ιοg, 7 . 54

Ά σ κ η ση

7 '1

Δίνεται η συνάρτηση f(x) = log(� - χ) α)Να βρεθεί το σύνολο ορισμού της f(x). β)Να δείξετε ότι η f(x) είναι περιττή. γ)Να βρεθούν τα σημεία τομής της γραφικής παράστασης f(x) με την ευθεία y=2. Λ \J ση : α) ε IR

Ένας αριθμός χ ανήκει2 στο σύνολο ορισμού της f αν , και μόνο αν : {νχχ +Ι2 +1;::: -χΟ > Ο (Σ). (Σ) <::::> � > χ <::::> χ ε IR , γιατί για κάθε χ ε IR έχουμε : J + I > R χ χ .Jχ 2 + > χ .Άρα το σύνολο ορισμού της f είναι το IR . β) Για κάθε χε IR έχουμε (-χ) ε IR και επίσης : ( Jx2 +1 +χ) · ( Jx2 +Ι-χ) 2 f(-χ)= log(�(-χ) + I +χ) = ιοg-'----� 2 �-χ I =-log(νx - +1 -x) =-f(x). =ιοg �-χ Άρα η f είναι περιττή. γ) Οι τετμημένες των κοινών σημείων της γραφι­ y=2 είναι οι της ff(χ)=2. και τηςΜευθείας κής παράστασης λύσεις της εξίσωσης: ε χ ε IR έχουμε: f(x)=2 <::::> ιοg<)χ2 +l-x)=2 <::::> ιog(Jx2 + 1-χ)=ιοg ιΟΟ <::::> .Jχ 2 + Ι -χ = 100 <::::> � = 100 + χ <::::> { χχ+2 +1=100 >οχ+100γ, <::::> {χ>-100 χ- +Ι=χ2 +200x+ lO.αxJ <::::> ( χ>-ΙΟΟ 9.999 <=> χ=- -m 9. -200 χ= �

Χ2

� ι;:::

I

�---'­

{

?

200

ΕΥΚΛΕΙΔΗΣ Β' 75 τ.3/32


Μ αθη ματικά για την Β ' Λυκείου

. . .

Ά ρα το σημειο τομης ειναι το

Άσκηση sιι Ν α βρεθούν οι τιμές του χ

9 . 999 , 2 ) . Μ( - -200

ε

(Ο, ;) έτσι ώστε

οι αριθμοί log ( ημ 2 χ ) ,-21og 2 και I og 1 + συν2χ 2 να αποτελούν με τη σειρά που δίνονται διαδοχι­ κούς όρους αριθμητικής προόδου. ΛίJση : Για να αποτελούν οι αριθμοί log ( ημ 2 x ) , -2 \og 2 , ι 1 + συν2χ διαδοχικούς όρους αριθμητικής og ---2 προόδου πρέπει και αρκεί: 2( -2 \og 2 )= \og ( ημ-χ ) + log 1 + συν2χ ( 1 ). 2 Για να ορίζεται η παραπάνω σχέση πρέπει και αρημ 2 χ > ο κεί: ---1 + συν2χ > 0 ( Σ ) . 2 ημχ ημχ (Σ) <=> <=> συνχ 7:- Ο ,που ισχύουν για συν-χ > Ο 'Εχουμε: κάθε Ε ( 1 ) <=> -4 1 og 2 = \οg ημ 2 χ + Ιοg συν 2 χ <=> log T4 = Ιοg(ημ 2 χ · συν 2 χ) I = ημ-χ · συν-χ <=> -1 = 4ημ-χ · συν-χ <=> <=> 16 4 1- = ημ-' 2 χ <=> ημ2 χ= -1 η' ημ2 χ=- -I . Απο' αυτες' 4 2 2 δεκτή είναι μόνο η ημ2 χ= -1 αφου' 2 Ο<χ< � <=> 0<2χ<π ,οπότε ημ2χ>Ο. 2 'Εχουμε ημ2χ= ..!.. <=> ημ2χ=ημ � <=> 2χ=2κπ+ � ή 2 6 6 5π 5π π 2χ=2κπ+ - Κ Ε 2. <::::> χ=κπ+ - η' χ + Κ Ε 2. . 6 ' 12 ' 12 π π 5π π <κπ<'Όμως Ο<χ<-π <=> Ο<κπ+ - <- <=> -2 12 2 12 12 <=> --Ι <κ<-5 <=> κ=Ο ' επειδή κ Ε Ζ. . Οπότε για 1 2 12 π . Επίσης Ο<χ<-π <=> Ο< κ=Ο έχουμε χ=2 Ι2 5π π 5π π 5 κπ+ - < - <=> -- <κπ<- <=> -- <κ<-Ι <::::> k=O' 12 12 Ι2 2 Ι2 Ι2 ο

{

χ

{

"#-ο (ο, Ξ} ο

{

"#-ο

=κπ

-

ο

5π . ' για κ= εχουμε ' χ= επειδη' κ ε !L.J'77 .οποτε Ι2

Άσκηση 9'� Σε μία αριθμητική πρόοδο είναι α =ln4 και α 2 =Ιη32.Να δειχθεί τι το άθροισμα των ν πρώ­ των όρων της δίνεται από τον τύπο ν sν = · ( 3 ν + 1) · 1η 2 . 2 β)Αν χ>Ο και Ιοgχ=9,να υπολογιστεί η τιμή της �4 παράστασης: log χ Μ νIχ �χ 4 � γ)Να λυθεί η εξίσωση: Ιο gω 3 χ : 15 = 1 ΛίJσ η: 32 3 =3\ η2 α)Είναι ω=α 2 -α =Ιη32-Ιη4=\η-=Ιη8=Ιη2 4 1

)

(

I

ω

και sν = �[ 2α, +(ν - 1) ] = �[ 2 · lη4 + ( ν - Ι ) · 3 · 1η 2 ] =

= -ν ( 4 \η 2 + 3 ν \η 2 - 3 \η 2 ) = -ν ( 3 ν + Ι ) \η 2 2 2 1ν x 4 �x if;. 3 1 4 �� = Iog ν χ χ · χ β)Εχουμε: log �χ � �χ �� J� 4 . 60 χ==4 �·χ-4-1 � ρ = log = l og χ χ log -3 Jχ ιρ J�X I 3 . Χ Ι 5 ρ ·

=

64

=

=

χ3ο" =Iogx �3 0 = 36 logx= 36 · 9 =1 0 8 =log ' 30 30 χ 30 28

x+2

7:-

l

γ) Πρέπει και αρκεί: χ + 2 > Ο 3χ + Ι 5 > Ο 4 I δηλαδή χ E (-2, - ) u (- I , +oo) . Επομένως για χ Ε (-2, - Ι ) υ (- 1 , +οο) έχουμε: Ιο 3χ:Ι5 =1 <=> lc>g,ω 3χ: I 5 = 1ogx+2 ( χ +2) <=>

ω(

g

)

(

)

3χ + Ι 5 ' = χ + 2 <=> χ=7 που ειναι --δεκτη.' 4 Άσκη ση ι οιι Να λυθούν τα συστήματα: ' χ 1" Υ = � e α) β) % ( χ-2) -Ι% ( Υ -3) =0 log(x+ 1)+ log(y-2)=31og2 2 ι 2+ Ιnχ-' = S'" Y

{

ΕΥΚΛΕΙΔΗΣ Β ' 75 τ.3/33

{


γ)

{33y.χ ·

Μ αθη ματικά για την Β ' Λυκείου

7 Υ = 147 γ = 63

{�Υx +y=3 ��χ = 63 <:::::> {Χy=2= 1

. \ 1>ση :

Αρχι κ ά πρέπει και αρκεί να εί ν αι χ>Ο και y>O και το σύστημα ισοδύναμα ράφεται : Δίνεται η συνάρτηση f(x) Ιη(ημχ + συνχ + 5) {lηΙ x·+ nlηy=l3 ηn ye-4 <:::::> { ln x·lγ η y=-4 α) Να δείξετε ότι ln(S - .J2) f(x) ln(S + .J2) ln x= ln y-4 2 2 3 Ι χ =2 Ι β) Αν α ν είναι γεωμετρική πρόοδος με α 1 = f( O ) 2 η y=-4 Ι η y-4 · ηy+4=0 και λ '21 να βρεθει, ποιος ορος , της ει,ναι ο Ι η 12 96 <=> {(!η χ = !η )Υ -41 <=> { !lηη χy-4 = !η Υ -4 16 lny=2 γ) Να λυθεί η εξίσωση: log if;. + �log x = log x )2 =0 <:::::> { ln y=2 { <:::::> {(!ηΙηχy-2 =!η y-4 Ιη χ =!η y-4 <:::::> lnx=-2 α) Επειδή για κάθε χ ημχ+συνχ +5>0 η συ­ y= e2 νάρτηση ορίζεται ια κάθε χ γ <:::::> X =­1 'Εχουμε : η μχ+συνχ= J2 ·η μ ( χ+�) . Άρα e2 χ-2>0 f(x)=ln [ J2ημ ( χ+�)+5] και >0 (Σ). β) Αρχικά πρέπει και αρκεί να είναι χy-3 +1 >ο � ημ ( χ+�)� -J2 � J2ημ ( χ+�)� J2 y-2 >0 χ> 2 και το σύστημα ισοδύναμα γράφεται: 5-J2 �J2ημ ( x+�)+5�5+J2 (Σ) <=> { y>3 O& (y-3) {χ-2 = y-3 <:::::> συνάρτηση Ιηχ είναι γνησίως άυξουσα , οπότε {lIO&og[(X( x+1-2)) =I· ( y-2 )] =logi ( x+1) ( y-2) = 8 Ιη(5-J2) �!η [ J2ημ ( χ+�)+ 5]:;:; Ιη(5 + J2) => χ = Υ -1 = Υ -1 {xy-2x+y-10=0 � f(x) � lη(5 + J2) <:::::> {χy(y-1 ) -2(y-1) +y-10=0 <:::::> β)Ιη(5-J2) α =f(O)=lη6. Είναι χ{ = Υ -1 <=> { χ = Υ - l Ι ν 1296 η 1 \ 1 v ( 6 =α · λ => = η · ) α y2 -2y- 8 = Ο y = 4ήy = -2 2 J6 τιμή y=-2 απορρί π τεται αφού πρέπει να ι σ χύει 4 Ιη 6 = Ιη 6 41η 6 = Ιη 6 => 2 v-Ι =4 y>3 άρα y=4 οπότε χ=4-1=3 η οποία επαληθεύει , 16ν-1=2=>ν=3 2ν-Ι 16 2ν-Ι τον περιορισμό χ>2. Ά ρα η λύση του συστήματος => είναι η (x,y)=(4,3 ) χ >0 { χ 3 =147 γ) Πρέπει και αρκεί να είναι : ( 1 ). χ {3 =147 l o χ 2: Ο g . γ) 3y . γ = 63 <:::::> 3y3' γ .}= 63Υ . γ =147·63 (Ι)<:::::> { χχ 2:>0 <:::::> χ 2: 1 . Με χ2: 1 έχουμε : 47 {3' . γ =147 {3' . γ =147 log�+J!og x =log x <:::::> .!.ιog x+�log x = log x {33y.χ ·Τ=1 . = 63 3 x+3 y7.χ7=63 x+y =9261 <:::::> 3y21x+. y =21= 633 <:::::> 3yx+y=3 <:::::> 2log χ -3νrι::::-:; = ο <:::::> { �l2ω-og-3χ =ω ω=Ο 3 3 �log x =ω <:::::> lo x=O ή lo x=-9 <:::::> χ=1 ή {33yχ .. γ7 -=χ =63147 <:::::> <:::::> }Υ · γ =763 g 4 3 g ω= ήω= Ο 2 x+y=3 x+y=3 x=J0 4 α)

Ά σ κ η ση 1 1 '�

=

\

=

.\ί> ση :

{

Ε�

Ε�.

-I

Ι =>

=>

<:::::>

Η

·

1

I

ν

Η

=>

γ

<:::::>

<:::::>

=>

J

I

γ

γ

<:::::>

{

ιu�

9

ΕΥΚΛΕΙΔΗΣ Β' 75 τ.3/34

λ

ο

<:::::>


- Μέτρηση Κύκλου

Κανονι κά Πολύγωνα

Στα πλαίσια αυτού του άρθρου, είναι δύσκολο να μιλήσουμε για την τεράστια αξία της Ευκλείδειας Γεωμετρίας. Πέρα από τη χρησιμότητά της σε τε­ χνικές εφαρμογές, από την Αρχαιότητα μέχρι σή­ μερα, επιπλέον προάγει την κριτική σκέψη και ε­ ξοικειώνει κάποιον στην αποδεικτική διαδικασία. Παρόλα αυτά, τα τελευταία χρόνια παρατηρείται μία τάση υποτίμησης' της Γεωμετρίας στα σχολεία και ιδιαίτερα στη Β Λυκείου, με τη λογική ότι, οι προαπαιτούμενες γνώσει ς της, για τα Μαθηματικά ' κατεύθυνσης της Γ Λυκείου, είναι "περιορισμέ­ νες". Όμως πολλά θέματα Πανελληνίων εξετάσεων, που έχουν τεθεί κατά καιρούς, απαιτούν και γνώσεις Γεωμετρίας. Αλλά εκτός αυτού, δεν θα πρέπει να υπάρχει αδιαφορία από τους μαθητές για το συ­ γκεκριμένο μάθημα, γιατί είναι ένα εργαλείο για τη συνέχιση των σπουδών τους σε πολλές σχολές, καθώς και χρήσιμο στην καθημερινή τους ζωή. 360° κεντρικη' γωνια' ων = -ν ισχύει γωνία I . Τ Υ Π Ο Ι Σ Τ Α K A � O !\ I K A Π Ο Λ Υ ΓΩ 'Ι Α

Δύο κανονικά πολύγωνα με τον ίδιο αριθμό πλευρών είναι όμοια, οπότε ισχύει η σχέση: �λ: = !._R' = �α: = Ρ�Ρν και �Ε : = [�λ: )2 Στις εγγραφές (ισόπλευρου - τετραγώνου και καν. εξαγώνου) έχουμε: α3 = Rημ30° 2R λ3 = 2Rσυν30° = R J3 R..fi λ4 = 2Rσυν45° = R .J2 α4 Rημ45° = -2 α6 = Rημ60° = -RJ32- λ6 = 2Rσυν60° = R

=

=

� χ όλιο

Για να εγγράψουμε ένα κανονικό πολύγωνο με ν πλευρές σ' ένα κύκλο, θα πρέπει να τον χωρίσουμε σε ν ίσα τόξα και να ενώσουμε τα αντίστοιχα ση­ μεία. Ο χωρισμός όμως του κύκλου, με κανόνα και δια­ ήτη, σε ν ίσα τόξα δεν είναι πάντοτε δυνατός. Ο βGauss απέδειξε ότι αυτό μπορεί να ί ν ει όταν: γ · ν=2 " ρ 1 ρ 2 . .ρ ν όπου ρ ι , ρ2, ... , ρν φυσικοί πρώτοι αριθμοί της μορφής: 2 2' όπου κ, μ ε + Ι,

Περίμετρος Ρ = ν· λν και εμβαδόν Ε ν = 2 Ρν ·αν Τόξο σε μοίρες Μήκος Μήκος τόξου κύκλου πRμ0 L = 2πR f= -180° Εμβαδόν Εμβαδόν κυκλικυκλικού κού τομέα δίσκου α� -;fλ2 = R 2 όπου αν: το απόστημα και η Ε = πR2 (Ο.ΑΒ) = πR 2 μο 360° πλευρά του. •

ν

+

Ν*

1 1 . Τ Υ Π Ο Ι Σ Τ Ο !\ Κ Υ Κ Λ Ο

1

Θανάσης Τσιούμας

λv:

ΕΥΚΛΕΙΔΗΣ Β' 75 τ.3/35

Τόξο σε rad Μήκος τόξου f=α·R Εμβαδόν κυκλικού τομέα (Ο.ΑΒ) =..!.. αR 2 2 (όπου α είναι α ριθμός rad)


Μ αθη ματικά για την Β ' Λυκείου

Ση μείω ση

2 θα έχουμε Επειδή S=α·R και (Ο.ΑΒ)=-α·R 2 ότι (Ο.ΑΒ) = -S21 · R 1

ο

Με σύρμα μήκους 20 m να περιφράξουμε ανθό­ κηπο σχήματος κυκλικού τομέα. Να βρείτε την ακτίνα r ώστε (Ο.ΑΒ) = max .

,....-..._ Θα πρέπει να εκφράσουμε το εμβαδόν του κυκλι­ κού τομέα ως συνάρτηση του r. Το μήκοςI τόξου θ rad είναι = r ·θ ενώ (Ο.ΑΒ) =-Γ 2 ·θ Η περίμετρος του κυκλικού τομέα θα είναι 2r + rθ = 20, οποτε. θ = 20-2r , Ο<r < IΟ r .. 1 -20-2r =10r-Γ, rε(Ο, 10) Οποτε. (Ο.Α,---.Β)=Ε(r)=-Γ 2 2 < Επειδή -1 Ο 10άρα το τριώνυμο E(r) = -r2 + 1 Or για β2α ---= r = --= 2(- 1) 5 παρουσιάζει max το Λυκείου το Eιnax =25 . Βέ β αια το μέγ ιστο στη βρίσκουμε κυρίως με τη βοήθεια των παραγώγων. ΛίJ ση

s

L

7

7

Π α ρ{ιδειγμα

Στο σχήμα είναι: ΑΟΒ = 90° και Τ ι = Τ2 να α­ ποδείξετε ότι ΑΟΒ = 1 rad

7

Γ

Λ υ μένες Α σκι] σεις

Να αποδείξετε ότι μια αυλή, μπορεί να κα­ λυφτεί μόνο με τρία είδη πλακιδίων, που είναι ίσα μεταξύ τους και έχουν σχήμα κανονικού πο­ λυγώνου. I.

Α

ΛίJση

Ισχύει: Τ1 +Τ=Τ2 +Τ�(ΟΑΒ)=(Ο.Α�Γ)� ..!_ S · R = ..!_ · R · R οπότε S = R που σημαίνει ότι η Έστω από ένα σημείο Ρ είναι τοποθετημένα κ ίδια 2 2 πλακίδια σχήματος κανονικού ν-γώνου Αν φν η AOB=I rad γωνία κανονικού ν-γωνου360"τότε θα ισχύει: κ· Ψv = 360° <::::>κ · (180"- ν ) =360" <::::>κ =2+_±_ ν-2 Στο βιβλίο κατεύθυνσης Λυκείου υπάρχει η πα­ Όμως κ φυσικός αριθμός οπότε θα πρέπει το ν - 2 ρακάτω άσκηση (στα μέγιστα - ελάχιστα) που ανα διαιρεί το 4, άρα ν= 3, ν = 4, ν = 6. 1 παιτείται η χρήση του τύπου Εκ.τ. = -αR 2 2 , α σε Επομένως, μπορούν να τοποθετηθούν: rad, όμως οι πιο πολλοί μαθητές τον αγνοούν!! Για ν = 3, κ = 6 πλακίδια σχήματος ισόπλευ­ ρου τριγώνου 4 Για ν = 4, κ = 4 πλακίδια σχήματος τετραγώ­ νου και Για ν = 6, κ = 3 πλακίδια σχήματος κανονικού εξαγώνου Δ

Λ ύ ση

Σ χό λιο

Γ

Ά σκη ση

σελ. 2 9 1 ( Γενι κές π α ρ αγώγων) ι

Β

ο

2.

Να υπολογίσετε την πλευρά, το απόστημα

ΕΥΚΛΕΙΔΗΣ Β' 75 τ.3/36


Μ αθη ματικά για την Β ' Λυκείου

και το εμβαδόν του περιγεγραμμένου τριγώνου σε κύκλο (Κ. R) ως συνάρτηση της ακτίνας R. Ε

Λί) ση

Έστω Δ, Ε, Ζ τα σημεία επαφής του τριγώνου με τον κύκλο . Α

72 ο Άρα Σ8Μ=8ι +82 =72° και Μι =--=-=36 2 2 Επομένως η γωνία Σ ι = 180°-(72° + 36°) = 72° οπότε το τρίγωνο ΜΣ8 είναι ισοσκελές δηλαδή ΜΣ = Μ8 ( 1 ). Η γωνία Σ 2 =Σ ι = 72° και Κ= 72° άρα και το ΑΔ Έχουμε: ΚΔ R και εφ60° =-R άρα τρίγωνο ΚΑΣ είναι ισοσκελές δηλαδή ΑΣ ΑΚ R (2) ΑΔ = Rεφ60° = RJ3 (1) ), (2) έχουμε Α8 οποτε, Α8 2ΑΔ δηλαδή Από (1ΜΑ-Μ8 Είναι ΑΔ == ΜΑ - ΜΣ =ΣΑ ΚΑ R 2 ii) Τα ισοσκελή τρίγωνα ΚΑΜ και ΣΚΜΜΑείναιΜΚό Α8 = 2RJ3. Το εμβαδόν 1 ΑΓ ημ60° = -(2Rν3) 1 � ?--J3 = 3R - μοια διότι ΑΚΜ = ΚΣΜ = 144° άρα ΜΚ = ΜΣ (Α8Γ) = -Α8 2 2 2 ΜΑ = R η, ΜΑ · Μ8 = R . , R και από (1), (2) ειναι Μ8 3. Δίνεται κανονικό πεντάγωνο ΑΒΓΔΕ εγγε­ γραμμένο σε κύκλο (Κ, R). Έστω Μ το μέσο του 4. Σε κύκλο (0, R) είναι εγγεγραμμένο κανονι­ ,.... -. τόξου ΒΓ. Να αποδειχτεί ότι: κό εξάγωνο με εμβαδόν Ε6 • Περιγράφουμε, στον �

=

(ΑΒ)

=

=

=

?

·

·

2

i) ΜΑ - ΜΒ = R ii) MA · MB = R 2

ίδιο κύκλο, κανονικό εξάγωνο με εμβαδόν Ε ' 6· Ε'6 • Να υπολογίσετε το λόγο -

Λ ύ ση

Έστω ότι η ΑΜ τέμνει την Κ8 360στοοΣ. i) Η γωνία Α8Γ = φ5 = 180ο --5-= 1 . Επειδη, Κ8 η διχοτόμος της γωνίας Α8Γ τότε η 8 2 = 54°, διότι επίσης 360° 72° και Μ το μέσο του (8Γ) = --= 5 �

oso

8Γ.

Ε6

Λ ί) ση

Για το εγγεγραμμένο κανονικό εξάγωνο έχουμε λ6 = R a6 = R J3 . Το περιγεγραμμενο, κανονικο, εξάγωνο θα έχει πλευρά λ' 6 και aπόστημα α' 6 R. Επειδή τα δύο κανονικά εξάγωνα είναι όμοια με λόλ'γο ομοιότητας R ; <=> -λ'6 2 θα έχουμε: _λ66 = αa66' <=> _λλ'66 = ----τ; Rν3 λ6 ν3 2 ' ' �: = ( �: ) �: = (�) :: = � '

--

2

=

=

ΕΥΚΛΕΙΔΗΣ Β' 75 τ.3/37


Μ αθη μ ατικά για την Β ' Λυκείου

5.

Από σημείο Σ που απέχει απόσταση 2R από κλο. το κέντρο Ο κύκλου (0, R) φέρουμε εφαπτόμενο τμήμα ΣΑ του κύκλου. Αν Β είναι το σημείο τομής του ευθύγραμμου ΟΣ με τον κύκλο, να υπολογίσετε σε συνάρτηση του R, την περίμετρο και το εμβαδόν του μεικτόγραμμου τριγώνου ΑΒΣ.

Σ

Ρ

Στο ορθογώνιο τρίγωνο ΓΡΒ η γωνία ΓΒΡ = Α 30° (χορδή - εφαπτομένη) οπότε ΓΡ = ΡΒ . Θέτουμε ΓΡ = χ οπότε ΡΒ = 2χ. Ισχύει Έχουμε Α = 90°. Άρα το τρίγωνο ΟΑΣ είναι ορ­ ΡΓ · ΡΑ2 = ΡΒ2 <=> χ · ΡΑ = (2χ) 2 <=> ΡΑ = 4χ άρα θογώνιο. ΟΑ R ΓΑ = 3χ = 3ΡΓ. ,Ειναι ημΣ = ΟΣ = 2R = '2 οποτε, = 30ο ii) Η Β ΟΓ = 60° άρα το Ο Γ Β είναι ισόπλευρο Επομένως Ο = 60°. με εμβαδόν (Ο Γ Β) = R 24J3 Από το ορθογώνιο ,Εχουμε ΑΣ = 2R συν30° = 2RJ3 = Rν3 και τρίγωνο ΒΑΡ έχουμε 2 RJ3ΣΒ = 2R - R = R ΒΓ-' = ΓΡ · ΓΑ <=> R -' = 3χ · χ <=> χ = 3 1 I Έστω S περίμετρος του μεικτόγραμμου τρι­ γώνου ΣΑΒ , τότε Επομένως το εμβαδόν του Γ Β Ρ είναι πR60ο R 2 J3 (l) 1 1 Rfi R S = (ΑΣ)+τ� + ( ΒΣ) = R ν.J + 0 +R η (Γ ) Β =-x Ρ) = (ΓB · R =· · P 80 1 2 3 6 2 πR Το εμβαδόν = RJ3 + + R 3 60ο πR 2 - R 2 J3 = r B) R Τ = (O (O.rn) = I άρα S = 3 (3J3 + π + 3). 4 360° Αν Τ είναι το εμβαδόν του μεικτόγραμμου τρι­ = πR6 2 - R 24J3 (2) γώνου ΣΑΒ , τότε Επομένως 2 J3 2 2 J3 R 2 2 = � R · 2Rημ60° πR 60° Τ= 360° Τ = (ΓΒΡ) - ΤI (= R - πR + R = ( 5 f3 - π) 6 4 6 2) 6 αρα, Τ = R 22J3 - πR6 2 = -R62 (3J3 - π) iii) Είναι 4Ri)

=

Λ iΗΗ ]

1

Δ

Σ �

Δ

-- .

r:;

--

@

η

Δ

r:;

ΑΒ

Δ

Δ

(I)

--

-

-

Ρ

6.

Δ

--

Ε(ΣΟΑ) - Εκ τομ (ΑΟΒ ) --

'

--

Στο σχήμα η ΑΒ = 2R είναι διάμετρος, Α = 30° και η ΒΡ είναι εφαπτόμενη του κύκλου. ί) Ν α αποδείξετε ότι ΡΓ = .!_3 ΑΓ ίί) Να βρείτε το εμβαδόν Τ του γραμμοσκια­ σμένου χωρίου ίίί) Να βρείτε τη δύναμη του Ρ ως προς τον κύ-

Ρ

--

-

?

�ιo. RJ = ΟΡ2 - R2 = ΡΒ2 <=> �ω. RJ = (2χ)2 = 4χ 2 = -3-

7. Το σχήμα δείχνει δύο ημικύκλια. Το μεγα­

λύτερο έχει κέντρο Ο και ακτίνα 4 cm, ενώ το μικρότερο είναι το (Κ, 3cm) και διέρχεται από το Ο. Να υπολογίσετε:

ΕΥΚΛΕΙΔΗΣ Β' 75 τ.3/38


Μ αθηματικά για την Β ' Λυκείου

Α

Δ

α) Το εμβαδόν (Ο Ρ Κ) β) Τις γωνίες ω και Κ = φ γ) Το εμβαδόν του κυκλικού τομέα (Κ.ΟΡ) δ) Το εμβαδόν (Ο.ΡΒ) ε) Το εμβαδόν του γραμμοσκιασμένου χωρίου S �

------.

Λ ιΊ σ η

Θα πρέπει να υπολογίσουμε την ακτίνα R του κύ­ κλου. . R 3R 2 .J3 = Είναι Ε = 6α 6 λ6 = 6 · R.J3 (2) 2·2 2 2 Επίσης Ε 4 = 4α;λ4 = 2R J2 . R J2 = 2R z (3) 2 Από ( 1 ), (2), (3) έχουμε: 3 R 2 .J3 - 2R 2 = 3 ν'3 - 4 <=> 3R 2 - 4R 2 = 2(3.J3 - 4) .J3 2 <=:> R2 (3ν'3 -4) = 2(3J3 -4) οπότε R 2 = 2 <=> 1R = 21 Επειδή Α Β λ6 η κεντρική γωνία Α ΟΒ = � rad 3 Δ Άρα Τ = (Ο.ΑΒ) - (Ο Α Β) = -ι R 2 · θ - -ι R · R · ημθ 2 2 .J3 = -ι R - · -π - -1 R - · ημ -π = -ι · 2 · -π - -ι · 2 · 2 3 2 3 2 3 2 2 .J3 = 2π - 3.J3 = -π - 3 2 6 6

α) Παρατηρούμε ότι ΟΚ ΚΡ 3cm ως ακτίνες του μικρού ημικυκλίου κα ι ΟΡ 4. Από τον τύπο του Ήρωνα: Ε = �τ( τ - α)( τ - β)( τ - γ ) όπου τ = 3 + 3 + 4 = 5 2 προκύπτει (Ο Ρ Κ) = �5(5 - 3 ) (5 - 3)(5 - 4) = #5 = 2JS cm 2 =

=

=

Δ

Δ

ι

α

r; β) Επειδή (Ο Ρ Κ) = - · 3 · 3 · ημφ (=) 2ν5

έχουμε 2 ημφ = 4J5 και από τους τριγωνομετρικούς πίνα--

9 κες βρ ί σκου με φ = 84°. Το τρίγωνο ΟΡΚ είναι σοσκελές άρα 2ω + 84° = 1 80° <=> ω = 48°. 84 0 · 2 π 3 = 2, ι Οπ cm2 γ ) Το εμβαδ όν (K.Or) = �

ι-

360° 48° · 4 = 2, 1 3π cmδ) Επισης (Ο.ΡΒ) = --π 360 0 ε) Το ζητούμενο εμβαδόν του γραμμοσκιασμένου χωρίου S θα είναι : Δ S = Ε Ι)μικ< Κ . 3 Ι - [ (Ο Ρ Β) + (Κ.Ο Ρ) - (Ο Ρ Κ)] = π · 32 = -- (2, ι 3π + 2, 1 0π - 2ν5r; ) = 0, 37π + 2ν5r; cm 2 2 '

7

7

.

=

7

,

Δίνεται το τεταρτοκύκλιο ΟΑΒΟ ακτίνας R και το ημικύκλιο C. Αν η κάθετη στο μέσο Μ της ΟΑ τέμνει το ημικύκλιο C στο Δ και το τε­ ταρτοκύκλιο στο Γ. Να βρείτε: ί) Το εμβαδόν του κυκλικού τμήματος Τ1 της χορδής ΑΓ ίί) Το εμβαδόν Τ του μεικτόγραμμου τραπεζίου ΟΒΓΔΟ 9.

.\ί1ση

Β

i'ι

Στο διπλανό σχήμα είναι: ΑΓ = λ4, ΑΒ = λι; και Ε 6 - Ε 4 = 3J3 - 4 (1). Να βρείτε το εμβαδόν του γραμμοσκιασμένου χωρίου Τ. i ) Η ΓΜ ΕΥΚΛΕΙΔΗΣ Β' 75 τ.3/39

είναι μεσοκάθετη του ΟΑ , άρα ΓΑ

=


Μαθη ματικά για την Β ' Λυκείου

J3

J3

ΓΟ = R δηλαδή το τρίγωνο ΟΓΑ είναι ισόπλευρο Δ R2 R οπότε ΓΜ = και (Γ Ο Α) = -- (1) --

4

2

J3

Δ 600 R2 = Επομένως 'Γ.1 = (Ο.ΑΓ) - (ΓΟΑ) = - πR2 4 360° πR 2 _ R 2 J3 = (2) 4 6 ii) Το ζητούμενο εμβαδόν Τ προκύπτει αν από το τεταρτοκύκλιο εμβαδού ΟΑΒΟ 2 πR (Ο.ΑΒ) = - (3) αφαιρέσουμε: �

4

--

Ο εμβαδόν του τομέα ΜΟΔΜ δηλαδή 90 ο . R 2 πR 2 ( ) = 4 π (Μ.6Δ ) = 16 2 360° Δ 1 R R R 2 J3 ' (Μ Γ Α) = - · - · = -Το εμβαδον 2 8 2 2 Το Τι Επομένως από ( 1 ), (2), (3), (4 ) έχουμε: 2 πR 2 _ πR 2 _ πR 3 _ R 2 _R = Τ= 4 16 6 4 8

()

• •

J3

--

J3] J3

(

Στο διπλανό σχήμα ΓΣ είναι διάμετρος με ΓΣ = 2R, η γωνία φ = 3ω και το εμβαδόν του κυκλικού τομέα (Ο.ΑΣ8) = !3 πR 2 (1) Ι Ο.

Γ

Σ

α) Να υπολογιστούν οι γωνίες ω, φ β) Ν α βρεθεί ο λόγος !ι όπου τ τ2 μοσκιασμένα κυκλικά τμήματα.

I'

α) Έχουμε

τ2

τα γραμ­

ω + φ = - και φ = 3ω οπότε 3 � π 2π 2π π ω + 3ω = - <=> 4 ω = - άρα ω = - και φ = 3 6 2 3 π 5π β) Η γωνία ΑΟΓ = π - ω = π - - = 6 6 Δ ------. Επομένως Τ1 = (Ο.ΑΜΓ) - (Ο Α Γ) = 1 5π 1 5π = - · - · R 2 - - R · R · ημ - = 6 2 2 6 5π 5π - 3 2 = R 2 _ ..!_ R 2 = R (2) �

12

4

( )

.....----.

12

=

4

( � ) R 2 (3) π 2

2

5π - 3 Από (2), (3) προκύπτει ότι: Ιι = Τ2 3π - 6 Ε ρ ωτή σης Σ - Λ Να κυκλώσετε το Σωστό Σ ή το Λάθος Λ των πα­ ρακάτω ισχυρισμών: i) Η κεντρική γωνία ενός κανονικού ν-γώνου εί­ ναι ίση με την εξωτερική γωνία του ν-γώνου. � . \ ii ) Το απόστημα ενός κανονικού ν-γώνου περιγε­ γραμμένου σε κύκλο (0, R) να είναι μεγαλύτερο από την ακτίνα. Σ Λ iii) Η γωνία ενός κανονικού ν-γώνου είναι πάντα μεγαλύτερη από την κεντρική του γωνία. Σ Λ ίν) Σε κανονικό ν-γωνο μπορεί να είναι αv=R. l: Λ ν) Σ' έναν κύκλο (0, R) μπορούμε με κανόνα και διαβήτη να εγγράψουμε κάθε κανονικό ν-γωνο.Σ Λ νί) Ένα τρίγωνο έχει μήκη πλευρών λ3 , λ4 και R J5 τότε το τρίγωνο είναι αμβλυγώνιο. Σ Λ νίί) Αν τόξο ακτίνας R = 2cm έχει μήκος R = 3π 3π d ' το το' ξο θ α ειναι ' Σ Λ τοτε ra . 2 νίίί) Αν ο λόγος των εμβαδών Ε και Ε ' δύο κύ16 ' ' ' ' λογος κλων ειναι - τοτε ο αντιστοιχος των μη25 '

άρα

Δ

1 1 Είναι Τ-, = (Ο.ΒΝΓ) - (Ο Γ Β) = - πR 2 - - · R · R =

4

' Σ Λ κων τους L και L ' θ α ειναι 5 ix) Αν το εμβαδόν ενός κυκλικού τομέα κύκλου 5π 2 (0, 2cm) είναι cm τότε το μήκος του τόξου θα

ΕΥΚΛΕΙΔΗΣ Β' 75 τ.3/40

4


Μ αθηματικά για την Β ' Λυκείου

5π ' ειναι cm

2

Σ

Λ

χ) Αν σε κύκλο (0, R) έχουμε τα σημεία του Α, R2 1 Β έτσι ώστε ΑΟΒ = - rad τότε (ΟΑΒ) = - Σ Λ 4 2 Π ρ ο η : ιν ό μενες

Α

Α σκ1Ί σεις για λί> ση

I . Να αποδείξετε ότι οι τομές των διαγωνίων κα­ νονικού πενταγώνου, είναι επίσης κορυφές κανο­ νικού πενταγώνου.

2. Δίνεται κανονικό εξάγωνο ΑΒΓΔΕΖ. Αν οι δι­ αγώνιες ΑΕ και ΓΖ τέμνονται στο Ρ και η ευθεία ΒΡ τέμνει την ΕΖ στο Σ, να αποδείξετε ότι: 1 ΣΖ 1 α) ΡΖ = - ΓΖ β) - = ΣΕ 2 4

3. Σε κύκλο (0, R) είναι εγγεγραμμένο κανονικό

ν-γωνο με εμβαδόν Ε και περιγεγραμμένο κανονιΕ 3 , , ' Ε ' τετοια κο, ν-γωνο με εμ β α δ ον ωστε - = -. Ε' 4 Να βρείτε το ν. (Απ. : ν = 6)

Δίνεται τρίγωνο ΑΒΓ εγγεγραμμένο σε κύκλο (0, R). Αν γ = �α2 + β 2 - αβ , να υπολογίσετε το εμβαδόν των κυκλικών τμημάτων Τ ι , Τ2 ΠΟυ η χορδή ΑΒ χωρίζει τον κύκλο. 6.

(Απ.

Είναι συνΓ = .!.2 οπότε ΑΒ = λ3. = RJ3 )

7.

Δίνεται κύκλος (0, R) και η διάμετρος του ΑΒ. Γράφουμε τον κύκλο (Α, λι). Να βρείτε το εμβαδόν του κοινού μέρους των δύο κύκλων. (Απ . : λ4 = R.Ji και το ζητούμενο εμβαδόν είναι

Ε = (π - l)R2)

Γ

Δ

4. Στο διπλανό σχήμα η ΓΔ είναι παράλληλη με την ΑΒ και ΓΟΔ = 90°. (Ο είναι το κέντρο του η­ μικυκλίου ΑΒ). Να αποδείξετε ότι Ε = Ει + Ε 2 .

Είναι Ει = Ε 2 = μ. Επίσης πR 2 ) καθώς και Τ + Ε = -4

(Απ. :

Τ+

πR 2 2μ = -4

Στο σχήμα δίνονται: ΑΒ=4α, ΟΓ=2χ. Να υπο­ λογιστεί το χ σε συνάρτηση του α, ώστε το γραμ­ μοσκιασμένο χωρίο να έχει εμβαδόν Ε ίσο με το

Τρεις κύκλοι με ακτίνες R 1 = J3 - \, R 2 = J3 + 1 και R 3 = 3 - .J3 εφάπτονται ανά δύο εξωτερικά. Να βρείτε το εμβαδόν του καμπυλόγραμμου τριγώνου, που ορίζεται από τα σημεία επαφής των κύκλων. 8.

(Απ. :

5.

1

- του εμ β αδου, του κύκλου. 4

α 2

(Απ. : χ = - )

2J3 - π3 (l0 - 4J3) )

Απαντήσεις στις ερωτήσεις Σ - Λ Σ-Λ-Λ-Λ-Λ-Σ-Σ-Σ-Λ-Σ

ΕΥΚΛΕΙΔΗΣ Β' 75 τ.3/41


Αντ. Κυριακόπουλος - Χρυστ. Κυβερνήτου - Αθαν. Μαλαφέκας !

.

ε:

l<. '\/ Κ \ ΟΣ

εξίσωση του κύκλου C με κέντρο το σημείο Κ( α, β) και ακτίνα ρ>Ο είναι: (χ _ α ) 2 + ( y _ β) 2 = ρ2 .

�.n.Η

Υ

ε

c

\

Μ(Χο, ο) Κ(α,β) k .2.

ο

)

χ

Η εξίσωση της εφαπτομένης ε ενός κύκλου

C : (x - α ) z + ( y - β) z = ρ z

σ' ένα σημείο του Μ(χο, Υ ) είναι:

(

χ -α

� . 3 . Κάθε

ο

) ( - α ) + ( y - β)( Υ ο - β) = ρ 2 . χ

ο

κύκλος στο επίπεδο έχει μία εξίσωση της Γ B 2 2 μορφής: χ + y + Αχ + y + = Ο ( Ι)

A z + Bz (στην οποία ισχύει: 4 Λ νηστ σ Ιi φ ω ς

παριστάνει

Α 2 + Bz 4

Γ

>

Γ

>

Ο ).

Μία εξίσωση της μορφής (I) έναν κύκλο μόνο αν: Ο . Τότε, το κέντρο του κύκλου

Α , - -Β ) και η ακτινα . Κ( - . του ειναι ειναι 2

ο

.

2 ρ = Α2 + Β2 - Γ . 4 z 2 Γ Αν A + Β = Ο, τότε η εξίσωση (Ι) παρι4 Α , --Β ) . . το σημειο . Κ( - στανει 2 2 2 Γ 2 < Ο, τότε η εξίσωση (I) είναι Αν Α + Β 4 n.T i Σ

.\ ίJ (iJ] .

Ένας κύκλος με κέντρο Κ( α, β) και ακτίνα ρ είναι ζητούμενος αν, και μόνο αν, α>Ο, β>Ο και ρ = d( K , x x ) = d( K , y y ) = d(Κ, ε) � [ 3 α - 4β - 12[ � ρ = [ β [ = [ α[ = .)3 -, + 4 -? '

'

{

{

ρ = α, β = α ρ=α β=α � [ 3α-4β- 1 2 [ � ι α + 1 2 ι = 5 α � α= 5 α = 3 (αφού α > Ο) ( α + 1 2 = 5 α ή α + 1 2 = -5 α ) β = 3 � β = α, ρ = α ρ=3

{

'

{

Άρα: Κ(3,3) και ρ=3 . Συνεπώς, η εξίσωση του ζη­ τούμενου κύκλου, είναι: (χ - 3) 2 + ( y - 3) 2 = 3 2 .

2. Να δείξετε ότι οι κύκλοι: C : χ 2 + y 2 - 4χ - 2y + 3 Ο και C' : χ 2 + y 2 - 10χ - 4y + 21 = Ο τέμνονται ορθογώνια, δηλαδή τέμνονται και οι εφαπτόμενες αυτών στα κοινά τους σημεία είναι κάθετες. =

Λ iJ σ η ,

)

Ο κύκλος C έχει κέντρο το Κ(2, Ι) και ακτίνα: ρ = Ι 6 4 _ 3 = Jl .

;

Ο κύκλος c · έχει κέντρο το Λ(5,2) και ακτίνα:

)

R = l 00 + 16 2 I = J8. 4

Δύο κύκλοι C(Κ,ρ) και C '(Λ,R) τέμνονται ορ­ θογώνια αν, και μόνο αν: (ΚΛ)2

=

ρ2

+

R

2

Έχουμε: ( ΚΛ ) 2 = (5 - 2) 2 + (2 - 1) 2 = 10 p 2 + R 2 . Άρα, οι κύκλοι C και C · τέμνονται ορθογώνια .

αδύνατη. . \ l: �� li

3χ - 4y - 12 = Ο και στους άξονες χ'χ και y'y.

1. Να βρείτε την εξίσωση του κύκλου που το κέντρο του είναι στην πρώτη γωνία των αξόνων και εφάπτεται στην ευθεία

=

3. Να βρείτε τις εξισώσεις των ευθειών, οι ο­ ποίες διέρχονται από το σημείο M(l,-2), τέ­ μνουν τον κύκλο C : χ 2 + y 2 - 14χ + 10y + 49 Ο

ΕΥΚΛΕΙΔΗΣ Β' 73 τ.3/42

=


Μαθη ματικά για την Β ' Λυκείου

σε δύο σημεία Ε και Ζ και το μήκος της χορδής 4. Θεωρούμε τον κύκλο C : (χ - 1) 2 + y 2 = 4 2 και την ευθεία ε : y = 2χ + 3. ΕΖ είναι 8. 1) Να δείξετε ότι ο κύκλος C και η ευθεία ε δεν έχουν κοινό σημείο. Η εξίσωση του κύκλου C είναι της μορφής: 2) Από ένα σημείο Μ της ευθείας ε φέρνουμε τις χ 2 + y 2 + Αχ + By + Γ = Ο με Α = - 1 4, Β = 1 0 εφαπτόμενες στον κύκλο C και ονομάζουμε Α και Γ = 49 . Έτσι το κέντρο του C είναι Κ(7, -5) και Β τα σημεία επαφής. Να δείξετε ότι, όταν το και η ακτίνα του είναι: σημείο Μ διαγράφει την ευθεία ε, η ευθεία ΑΒ z 2 2 2 διέρχεται από ένα σταθερό σημείο. Α :B 14 : 10 = - 49 = 5 . -Γ =

ρ

Οι συντεταγμένες των κοινών σημείων του κύ­ κλου C και της ευθείας ε, αν έχουν, είναι οι λύσεις του συστήματος: ε (χ - 1) 2 + y 2 = 4 (χ - 1) 2 + (2χ + 3) 2 = 4 - -, � <=> <=> y = 2χ + 3 y = 2χ + 3 : · Η./', \C Et �>,.\ 5x 2 + l 0x + 6 = 0 ( 1 ) �- - - - .... <=> � ι 5 Κ( Ί ,-5) y = 2χ + 3 Μ(7,-2) \ Η διακρίνουσα της εξίσωσης ( 1 ) είναι Δ=-20<0. Άρα, το σύστημα είναι αδύνατο και συνεπώς ο κύ­ Ονομάζουμε Η τη ν προβολή του Κ στην ε, οπότε κλος C και η ευθεία ε δεν έχουν κοινό σημείο. Ε Η=ΗΖ Έτσι. έχουμε: 2. ) Έστω ότι (λ, μ) είναι οι συντεταγμένες ενός ση­ 2 2 μείου Μ της ε, οπότε μ = 2λ + 3 και άρα: (ΕΖ) = 8 <=> ( Ε Η ) = 4 <=> �5 - (ΚΗ) = 4 <=> Μ(λ, 2λ + 3) . Έστω ακόμα ότι Α( χ 1 , y 1 ) και <=> (ΚΗ ) = 3 <=> d(Κ, ε) = 3 . Ο ι εξισώσεις των ευθειών που διέρχονται από το Β(χυ y 2 ) . Η εξίσωση της ευθείας ΜΑ, είναι: (χ - l)(x 1 - I) + yy 1 = 4 και επειδή αυτή διέρχεται Μ, είναι: χ = 1 και y+2=λ(χ -1) <::::>λχ -y-λ-2=0 (λ ε!R) (1). από το Μ, έχουμε: (λ - Ι)(χ 1 - 1) + (2λ + 3)y 1 = 4. (2) Η ευθεία με εξίσωση : x =l δεν τέμνει τον κύκλο C Όμοια βρίσκουμε ότι: (γιατί;) και άρα δεν είναι ζητούμενη. (λ - 1)(χ 2 - 1) + (2λ + 3)y 2 = 4. (3) Μία ευθεία ε : λχ-y-λ-2=0 είναι ζητούμενη αν, Θεωρούμε τώρα την ευθεία με εξίσωση : και μόνο αν: (λ - Ι)(χ - 1) + (2λ + 3)y = 4. (4) 7λ + 5 - λ - 21 d(Κ, ε) = 3 <=> l = 3 <=> 1 2λ + 1 1 = νr:::;-: λ2 + I <=> Η ευθεία αυτή, λόγω της (2) διέρχεται από το Α Jλ2;Ϊ λ- + 1 και λόγω της (3) διέρχεται από το Β . Άρα, η εξί­ 4 4λ- +4λ + 1 = λ- +1 <::::> 3λ- + 4λ = 0 <=> λ = Ο η, λ = -3 σωση της ευθείας ΑΒ είναι η (4). Έχουμε: (4) <=> λ(x + 2y - l) - x + 3y - 3 = 0. Από την ( I ) : Έτσι, η ευθεία ΑΒ διέρχεται από το σημείο του Με λ=Ο, βρίσκουμε: οποίου οι συντεταγμένες είναι λύση του συστήμα­ -y - 2 = 0 <=> y = -2 (2). τος: Με λ = - � , βρίσκουμε: 3 χ=x + 2y - 1 = 0 x + 2y = l 4 4<=> <=> -- x - y + - 2 = 0 <=> 4x + 3y + 2 = 0 (3) -χ + 3y - 3 = Ο -χ + 3y = 3 3 3 Υ= Άρα υπάρχουν δύο τέτοιες ευθείες. Οι εξισώ­ Άρα, η ευθεία ΑΒ διέρχεται από το (σταθερό) σησεις τους είναι οι (2) και (3).

Έστω ότι μία ευθεία ε διέρχεται από το σημείο Μ και τέμνει τον κύκλο C στα σημεία Ε και Ζ. /

..

\

{

{

{

.

ο

Ί

Ί

(

)

c

ο

{

ΕΥΚΛΕΙΔΗΣ Β' 73 τ.3/43

{

{

� �


( _ Α , _ Β ) = ( - � , �) .

( )

Μαθη ματικά για την Β ' Λυκείου

3 4 μειο - 5 ' 5 . .

5. Θεωρούμε την ευθεία ε : 2χ - y - 3 = Ο και τον κύκλο C : χ 2 + y 2 - χ + y - 2 = Ο . α) Να δείξετε ότι η ευθεία ε και ο κύκλος C τέ­ μνονται. Ονομάζουμε Μ και Ν τα σημεία τομής. β) Να δείξετε ότι η εξίσωση: χ 2 + y 2 - χ + y - 2 + λ(2χ - y - 3) = Ο (1) για Κάθε λ Ε JR , παριστάνει ένα ΚύΚλΟ Cλ , Ο Οποίος διέρχεται από τα σημεία Μ και Ν. γ) Να βρείτε το γεωμετρικό τόπο των κέντρων των κύκλων Cλ , όταν το λ διατρέχει το JR

2 2 2 2 Ένα σημείο M(x,y) ανήκει στο ζητούμενο γεωμε­ τρικό τόπο αν, και μόνο αν, υπάρχει λ ε JR με: 2 -1 χ=2χ = -2λ + 1 λ = 2y + 1 <=> <=> 2χ = -2λ + Ι 2y = λ - 1 λ 1 Υ=

{

\ �

{

{

Αυτό συμβαίνει αν, και μόνο αν: 2χ = -2(2y + 1) + 1 <=> 2χ + 4y + 1 = ο . Άρα, ο ζητούμενος γεωμετρικός τόπος είναι η ευ­ θεία με εξίσωση : 2χ + 4y + 1 = Ο .

Λ ί) ση

Λύνουμε το σύστημα των εξισώσεων των γραμμών ε και C : y = 2x -3 2x -y-3 = 0 <=> 2 2 χ + i -x+y-2 =0 χ +(2χ -3)2 -χ+2χ -3-2 =0 y = 2x - 3 <=> 5χ 2 - 1 1χ + 4 = 0 Το σύστημα αυτό έχει δύο λύσεις, γιατί η διακρί­ νουσα της εξίσωσης: 5 χ 2 - 1 1 χ + 4 = Ο είναι Δ = 1 12 - 4 · 5 · 4 = 4 1 > 0 . Άρα, η ευθεία ε και ο κύκλος C τέμνονται. β) Έχουμε: (1) <=> χ 2 + y 2 + (2λ - 1)χ - (λ - 1)y - (3y + 2) = 0 (2) Αυτή είναι της μορφής : χ 2 + y 2 + Αχ + By + Γ = Ο με: Α = 2λ - 1, Β = -(λ - 1) και Γ = -(3λ + 2). Για κάθε λ ε JR , έχουμε: 5λ2 +6λ+10 (2λ-1)2 +(λ-1)2 Α2 + Er +3λ+2 -Γ >Ο, 4 4 4 γιατί η διακρίνουσα του τριωνύμου: 5λ 2 + 6λ + 1 Ο είναι αρνητική. Άρα, για κάθε λ ε JR , η εξίσωση (2), επομένως και η ( 1 ) παριστάνει ένα κύκλο cλ . Επειδή τα σημεία Μ και Ν είναι σημεία και των δύο γραμμών ε και C, οι συντεταγμένες τους επαληθεύουν τις εξισώσεις και των δύο αυτών γραμμών και άρα επαληθεύουν και την εξίσωση ( 1 ), για κάθε λ ε JR. Άρα, όλοι οι κύκλοι Cλ διέρχονται από τα σημεία Μ και Ν. γ ) Το κέντρο ενός κύκλου Cλ είναι: α)

{

{

{

2. Π Α ΡΆ ΒΟ Λ Η 2. 1 . Ο ρ ισμός Σ' ένα επίπεδο θεωρούμε μία ευθεία δ και ένα σημείο Ε εκτός της δ. Ο γεωμετρικός τό­ πος των σημείων του επιπέδου αυτού, τα οποία ισαπέχουν από το Ε και τη δ, ονομάζεται παραβο­ λή με εστία το σημείο Ε και διευθετούσα την ευ­ θεία δ. δ Υ c

Ε( f ,ο)

χ

2

χ=- -Ρ 2 2.2 Η εξίσωση της παραβολής C με εστία Ε( Ε. , ο) 2

και διευθετούσα

δ:χ

= _ Ε. (p =F- Ο) , είναι : 2 2 y = 2px.

Η εξίσωση της παραβολής με εστία Ε(Ο, Ε. ) 2 Ε. και διευθετούσα δ : y = _ (p =F- Ο) , είναι 2 χ 2 = 2py. 2.3 Η εφαπτομένη της παραβολής C : y 2 = 2px στο σημείο της Α(χ ο , Υ ο ) έχει εξίσωση : ΥΥο = p(x + X 0 ) . Η εφαπτομένη της παραβολής C : χ 2 = 2py στο σημείο της Α( χ ο , y ο ) έχει εξίσωση : ΧΧο = p(y + y o ). •

ΕΥΚΛΕΙΔΗΣ Β ' 73 τ.3/44


Μ αθη ματικά για την Β ' Λυκείου

θε y ε JR. Έτσι έχουμε:

Α Σ Κ Η 2: Ε Ι Σ

d

1

( y - 3 y + 1 2) 2

1 . Να βρείτε την εξίσωση της παραβολής C που έχει κορυφή την αρχή των αξόνων, την ε­ τριώνυμο: / - 3 y + 1 2 έχει ελάχιστη 3 Τοτε: . x = -1 y -? = -1 · -9 = -9 . στία της στον άξονα χ· χ και εφάπτεται στην ευ­ y = -. θεία ε : y 2χ + 1. 2 4 4 4 16 .

=

Λ\> ση

Η εξίσωση της C είναι της μορφής: / 2px . Η εφαπτομένη της C στην κορυφή της 0(0,0) είναι η ευθεία y ' y, διαφορετική από την ε. Έστω Μ ( χ ο , y ο ) ένα σημείο της C διαφορετικό από την κορυφή της, οπότε χ y :f:. Ο και Υο� = 2 pX0 ( l ) Η εξίσωση της εφαπτομένης της C στο Μ είναι: =

ο

ο

(I) 2 _Ε_ � = + (x )<=> Χ p Υο ΥΥο = p(x +�) ο Υ 2p <=> Υ = Υο Χ + 2 .

Έτσι, η ε εφάπτεται στη C αν, και μόνο αν, υπάρ­ χει Υ ο Ε IR με:

!

(9

= -

5

.

τιμή για

Άρα, οι συντεταγμένες του ζητούμενου σημείου

)

. . . αποσταση 3 Η υποψη ειναι: 16 2 d ιηi n = 5Ι (49 - 3 · }23 + 12) = 39 20 .

.

ειναι - , - .

3. Έστω μία παραβολή C : y z = 2 p x (p >O) με εστία Ε και ένα σημείο της Μ. Να δείξετε ότι ο κύκλος c· με διάμετρο το ευθ. τμήμα ΕΜ εφά­ πτεται στον άξονα y 'y. Λ \>ση

Έστω

M(x0 , y0) , οπότε:

_Ε_ = 2 Υ ο <=> Υ ο = 2 Υ; � ι p � 2y,

{

(1) c·

Α

χ

Αυτό συμβαίνει αν, και μόνο αν:

p = 2 · 2 <=> p = 4.

Άρα, η ζητούμενη παραβολή είναι C : y 2

= 8χ. Επειδή

Ε(Ε., ο) ,

οι συντεταγμένες του μέσου, έ-

2 2. Θεωρούμε την παραβολή C : y z 4χ και στω Κ, του ευθύγραμμου τμήματος ΕΜ, είναι: την ευθεία ε : 4χ - 3y + 12 = Ο . χ ο + � , �ο . Να βρείτε τις συντεταγμένες του σημείου της C, 2 του οποίου η απόσταση από την ευθεία ε είναι η Έτσι, έχουμε: ελάχιστη δυνατή. Ποια είναι η ελάχιστη αυτή 2 απόσταση; (ΚΜ)2 =(ΚΕ)2 = � +� - Ξ + ; = � + λ i> σ η 2px o = Χ� + Χ� - px o + p2 = (=I ) px o + p 2 + -Η απόσταση d ενός σημείου M(x,y) της C από την =

(

ε, είναι:

d = l 4 x ι- 23y +212l = -5I l 4x - 3y + 1 2 I . ν4 + 3 ' Εχουμε y 2 = 4χ και συνεπώς χ = ..!_ y 2 . 4 χουμε:

1

I

Το

' Ετσι, έ-

d = 51 4 41 y 2 - 3y + 12 = 51 1 y 2 - 3y + 12 I ,

όπου y Ε IR. Το τριώνυμο y 2 - 3y + 1 2 έχει Διακρίνουσα αρνη­ τική και συνεπώς y 2 - 3 y + 1 2 > Ο , για κά-

(

) ) ( J2 (

-�J2 �

4 4 16 4 4 4 16 2 = χ20 + � => (ΚΜ) = (ΚΕ) = � +� = � +� ( 1 ), γιατί p > Ο και Χ0 � Ο. Έστω τώρα Α η προβολή του Κ στον άξονα y ' y.

(

I I

J

Έχουμε: Α(Ο, 2'2..) και συνεπώς:

2 � ( ΚΑ ) = �2 +E.4 = �+ 2 Ε.4 (ΚΜ) = (ΚΕ). Άρα, ο κύκλος C · εφάπτεται στον άξονα y · y.

ΕΥΚΛΕΙΔΗΣ Β' 73 τ.3/45

� I


Μ αθηματικά για την Β ' Λυκείου

4. Να β ρείτε τις εξισώσεις των εφαπτομένων της παρα β ολής C : y 2 = 8χ, οι οποίες σχηματίζουν με την ευθεία ε : 3 χ - y + 5 = Ο γωνία 45°.

και σταθερό άθροισμα

Α '/ . \ i : <Ό\1

Έστω Μ ( χ ο , y ο ) ένα σημείο της παραβολής C, οπότε: (1) Υ � = 8χ ο Η εξίσωση της εφαπτομένης ε ' της C στο Μ , είναι: ΥΥο = 4(χ + Χο ) <=> 4χ - ΥΥο + 4 Χο = Ο (2). Γνωρίζουμε ότι: = ( 1,3) 1 1 ε και � = ( y , 4 ) 1 1 ε ' .

δ

0

_-"

Ε'

2α . y "

Ο

/

Ε

I

χ

.L. Η εξίσωση της έλλειψης με εστίες Ε ' ( -γ, Ο), Ε(γ, Ο) (γ 2 Ο) και σταθερό άθροισμα 2α (προφανώς γ<α ) είναι:

χ 2 2 = 1 , οπου , β = �α 2 - γ 2 . α β � Θέτουμε: θ = (δ: ) , οπότε: Ο � θ :::; 1 80 ° . Οι ευθείες ε και ε ' σχηματίζουν γωνία 45° αν, και μόνο Ι σχύουν: α z β και γ = Jα 2 - β 2 • -, + Υ

7

Λ

αν: (θ = 45° ή θ = 1 80° - 45° ) <::::> <=> ( συνθ = συν45° ή συνθ = -συν45° ) <=> j συνθj = J2 <=>

2

�� = J2 ι δι �ι 2

Ι I Yo + I2j = <=> JW )y� + 1 6 J2

+ I 2 ) 2 = -1 <=> 2 ( y0 Ι <=> I O · ( y � + Ι 6) 2 Υ ο - 6 y ο - 6 = Ο <::::> <=> ( Υ ο = 8 ή Υ ο = -2 ). ?

Από την ( 1 ) με Υ = 8 βρίσκουμε Χ0 = 8 και με ο 1

Υο = -2 βρίσκουμε Χο = 2 .

Η

εξίσωση

της

έλλειψης

με

εστίες

Ε ' (Ο, -γ), Ε(Ο,γ) και σταθερό άθροισμα 2 α είναι: χ-, + y -, = 1 , οπου ' β = �α 2 - γ 2 . α β- χ2 2 Εκκεντρότητα μιας έλλειψης C : -, + y , = Ι , α - βονομάζεται ο αριθμός e = 1_ . α ?

?

-

Ισχύουν: Ο � e < Ι και l .J1 - e 2

α

·;

· '

=

Η εφαπτομένη της έλλειψης C :

?

?

χ � + Υ: = Ι σ' α - β-

Ρ(χο , Υ ο) έχει εξίσωση : χχο + ΥΥ σ = I α2 β2 χ Η εφαπτομένη της έλλειψης C : : + Υ: = 1 σ' β- α ένα σημείο της P (x 0 , y0 ) έχει εξίσωση : ΧΧο + ΥΥ ο = 1 βz α2 ·

ένα σημείο της

Από τη (2):

Με Χ = 8 και Υ = 8 βρίσκουμε: ο ο

·

4x-8y +4· 8=0<::::> x -2y +8=0 (3). 1

Με Χ = και Υ = βρίσκουμε: ο 2 ο -2 ,

4x + 2y + 2 = 0 G2x + y + l = 0 (4) .

Άρα, υπάρχουν δύο τέτοιες εφαπτόμενες. Οι εξι­ σώσεις τους είναι οι (3) και (4 ) .

ο

?

?

1. θεωρούμε έλλειψη μία 2 2 χ y C:= 1 (α>β>Ο) και ένα σημείο της Ρ 2 +α β2 ι. : . Θεωρούμε ένα αριθμό α>Ο και σ' ένα επίπεδο δύο σημεία Ε ' και Ε με (Ε Έ) < 2 α . Ο διαφορετικό από τις κορυφές Β και Β Ό Οι ευ­ γεωμετρικός τόπος των σημείων Μ του επιπέδου θείες ΡΒ και ΡΒ ' τέμνουν τον άξονα χ ' χ στα ση­ αυτού, για τα οποία ισχύει: (ΜΕ ' ) + (ΜΕ) = 2 α , μεία Μ και Ν, αντιστοίχως. Να δείξετε ότι, όταν ονομάζεται έλλειψη με εστίες τα σημεία Ε ' και Ε το Ρ διαγράφει τη C (εκτός των κορυφών Β και Β '), το γινόμενο (ΟΜ) · (ΟΝ) είναι σταθερό. �

ΕΥΚΛΕΙΔΗΣ Β' 73 τ.3/46


Μ αθη ματικά για την Β ' Λυκείου

. \ ίJ σ η

εξίσωση της ευθείας ΚΛ, όπως γνωρίζουμε, εί­ ναι: y · 4 = 2(χ - 3) <=> χ - 2y - 3 = Ο (I ) Βρίσκουμε τα κοινά σημεία της ευθείας ΚΛ και της έλλειψης C' Προς τούτο, λύνουμε το σύστηχ - 2y-3=0 y= (x-3) y= � (x-3) z μα: x 2y 2 - +- =1 1 ' χ- +24 (χ-3)- = 3 χ- -2χ+ Ι = Ο 3 3 x=l <=> y = -1 Άρα, η ευθεία ΚΛ και η έλλειψη C' έχουν ένα μο­ ναδικό κοινό σημείο, το Ρ( Ι , - 1 ). Η εξίσωση της εφαπτομένης της C' στο σημείο της Ρ ( 1 , - 1 ), είναι: � 2 y · (- 1) = Ι <=> χ - 2y - 3 = Ο (2). + 3 3 Η (2) όμως είναι η εξίσωση ( 1 ) της ευθείας ΚΛ Άρα, η ευθεία ΚΛ εφάπτεται στην έλλειψη C' . Η

(I)

Τ ε·

Μ χ Β'

Επειδή Β (Ο, β) και Β ' (Ο, -β) , έχουμε: -β +β λΡΒ = Υ ο Και λ ΡΒ' _- Υ ο . χο χο Η εξίσωση της ευθείας ΡΒ είναι: Υ -β Υ - β = ο χ . (2) χο Η εξίσωση της ευθείας ΡΒ · είναι: Υ +β Υ + β = ο χ . (3) χο Από τη (2) με y = Ο , έχουμε: χ = -β χ Υο - β -β χ Άρα: Μ ( '' . 0 ) . Υ,, - β Από την (3) με y = Ο . έχου με: χ = � . Υο + β

-- -ο

__

.

--

Άρα: Ν( � , Ο) . Υο + β Έτσι έχουμε: (ΟΜ) · (ΟΝ) =

(I)

=

Χ2

ο 7 --

-χ � αz

. ! i

{

7

?

.

1

.

3. Να βρείτε την εξίσωση της έλλειψης C που έχει εστίες E'(-J6,0), E(J6,0) και εφάπτεται στην ευθεία ε : χ + 2y - 4 = Ο . ι 1) -ση

Έστω ότι μία έλλειψη C πληροί τις δοσμένες συν­ θήκες. Η εξίσωση της C θα είναι της μορφής: '

''

-βχ ο βχ ο · __ = β-χ, σ, Υσ - β Υο + β Υ� - β-

__

= α 2 ( σταθ ερο ) .

'

2. Από το σημείο Μ( -3,4) φέρνουμε τις εφα­ πτόμενες της παραβολής C : y 2 = 4χ και ονομά­ ζουμε Κ και Λ τα σημεία επαφής. Να δείξετε ότι η ευθεία ΚΛ εφάπτεται στην έλλειψη 2 2 =l. χ 2 _r_ C' : + 3 3

?

:� + �: = 1

(α>β>Ο) .

(1)

Θα έχουμε γ = J6 , οπότε: α2 - β2 = 6 . (2) Έστω ότι η C εφάπτεται στην ευθεία ε στο σημείο M( X0 , y 0 ) , οπότε : 2 2 �+�=] . (3) αz βz Έτσι, η εξίσωση της ε είναι: χχ ο , + Yf,o = 1 . (4) α- β Άρα, η εξίσωση : χ + 2y - 4 = Ο και η εξίσωση (4) παριστάνουν την ίδια ευθεία ε. Αν y ο = Ο , τότε το προηγούμενο προφανώς δεν συμβαίνει. Άρα y ο =f:. Ο . Έτσι, έχουμε:

ΕΥΚΛΕΙΔΗΣ Β' 73 τ.3/47


Μ αθη ματικά για την Β ' Λυκείου

χ Β'

Αντικαθιστώντας στην (3) βρίσκουμε: 2 2 _;_ � + � f_ = 1 <=:> α 2 + 4β 2 = 1 6 (5). α- 4 β- 2 Από τις (2 ) και (5) βρίσκουμε: α 2 = 8 και β 2 = 2 . Αντικαθιστώντας στην ( 1 ) βρίσκουμε ότι η εξίσω­ χ 2 y2 ση της C είναι: - + - = 1 .

[) [) 8

2

Λ ντ ι στρόφως

Δείχνουμε (όπως στην προηγούμενη άσκηση) ότι η έλλειψη χ 2 y2 C:-+=1 8

2

εφάπτεται στην ευθεία ε : χ + 2y - 4 = 0 Άρα, η μοναδική ζητούμενη έλλειψη είναι η χ 2 y2 C:-+-=1. 8

2

4.

Θεωρούμε έλλειψη μία 1 (α>β>Ο) και τις εστίες της α β Ε'(-γ,Ο) και Ε(γ,Ο) . Η κάθετη στον άξονα χ'χ από την εστία Ε, τέμνει τη C στο σημείο Μ. Η εφαπτομένη ε της C στο σημείο της Μ, τέμνει τον άξονα y 'y στο σημείο Ν. Να δείξετε ότι η κάθετη ε ' της ε στο Ν διέρχεται από την εστία Ε'. 2 2 χ y C:+ 2 2 =

εξίσωση της εφαπτομένης ε, είναι: !]__ + yy ο = 1 Από αυτή με χ=Ο βρίσκουμε: α2 β2 β 'β2 Υ = - . Άρα: Ν(Ο, -) . Υο Υο ? α -? y ο β-γ 'Εχουμε: λ = - , λ =αρα ,- . και α 2 Υο β-γ β -' α ?- Υ ο y = , χ. : ε' Έτσι, έχουμε (2). Υ ο β-γ Τώρα, αρκεί να δείξουμε ότι η εξίσωση (2) επαλη­ θεύεται από τις συντεταγμένες (-γ, Ο) του Ε' . Δηλαδή, αρκεί να δείξουμε ότι: β 2 αy ο -- = -( -γ ) , δη λαδη, αρκει:, Υο β 2 γ ισχύει λόγω της ( 1 ). Η

·

ε

ε'

4. V ll E P BO Λ H 4. 1 Ο ρ ισμός : Θεωρούμε ένα αριθμό α>Ο και σ' ένα επίπεδο δύο σημεία Ε ' και Ε με (ΕΈ) > 2α . Ο γεωμετρικός τόπος των σημείων Μ του επιπέδου αυτού, για τα οποία ισχύει: I CME ') - (ME) I = 2α , ονομάζεται υπερβολή με εστίες τα σημεία Ε ' και Ε και σταθερή απόλυτη διαφορά 2α . Υ Μ Ε'

Λ \J ση

χ

Η τετμημένη

του Μ είναι γ. Έστω Υ ο η τεταγμένη του. Επειδή Μ ε C , έχουμε: β4 1_2 + Υ ο2 = 1 � Υ 2 = β 2 α 2 γ 2 � Υ 2 = ( 1 ). ο ο α2 β2 α2 α2 _

4.2

Η

εξίσωση της υπερβολής με εστίες Ε ' ( -γ, Ο), Ε(γ, Ο) (γ>Ο) και σταθερή απόλυτη διαφορά 2α (προφανώς γ>α) είναι:

ΕΥΚΛΕΙΔΗΣ Β' 73 τ.3/48


Μ αθη ματικά για την Β ' Λυκείου

χ2 - y 2 = I , όπου β = �r-γ 2 - α 2 • z α βz Ισχύουν: γ > β και γ = �α 2 + β 2 . Η εξίσωση της υπερβολής με εστίες Ε '(Ο, -γ), Ε(Ο,γ) και σταθερή απόλυτη διαφορά 2 α είναι: Υ 20 - χ 2 = Ι , οπου β = Ιγ 2 - α 2 ν . α- β aσύμπτωτες της υπερβολής Οι 2 2 χ y C : -2 - α β -0 = Ι είναι οι ευθείες με εξισώ­ σεις: y = Ια χ και y = -Ια χ . 2 2 Οι aσύμπτωτες της υπερβολής C : �2 - ;2 = Ι --

της στον άξονα χ 'χ . Τότε θα έχει μία εξίσωση της χ2 - y 2 = I ( α>Ο, β>Ο) . ( 1 ) ' μορφης: 2 α βz Οι δοσμένες συνθήκες πληρούνται αν, και μόνο αν:

��� :�

α2 β2 I Αντικαθιστώντας στην ( Ι ) βρίσκουμε: 4 .3 2 x2 - L = I . (2) 3 β) Έστω ότι μία τέτοια υπερβολή έχει τις εστίες της στον άξονα y 'y . Τότε θα έχει μία εξίσωση της / χ2 μορφής =1 . α2 - β2 Οι δοσμένες συνθήκες πληρούνται αν, και μόνο Ι είναι οι ευθείες με εξίσωση: y = χ και αν: α fj α = β/3 y = -Ix . α=β α αδύνατο. <=> 9 4 <=> 2 χ 2 y2 = Ι = Ι β = -1 2 ..ι ...ι Εκκεντρότητα μιας υπερβολής C : - - 3β2 β α 2 β -0 - Άρα, υπάρχει μία μόνο τέτοια υπερβολή και η ονομάζεται ο αριθμός: e = lα . εξίσωσή της είναι η (2). Ι Ισχύουν: e > I και α = � . δύο εφαπτόμενες 4.5. Η εφαπτομένη της υπερβολής 2. Ν α δείξετε ότι υπάρχουν 2 / 2 L2 1 , οι οποίες είναι χ υπερβολής C : χ της C : -0 - -2 = Ι σ ' ένα σημείο της P (x 0 , y 0 ) 3 α- β κάθετες στην ευθεία ε : χ + 2y 7 = Ο . Μετά, να ο =Ι . � έχει εξίσωση: χχ βρείτε τη μεταξύ τους απόσταση. α -0° β Λ ίJ ση χ Η εφαπτομένη της υπερβολής C : Υ�- - : = 1 Έστω Ρ( χ ο , y ο ) ένα σημείο της υπερβολής C, οπόα βσ ' ένα σημείο της P (x 0 , y 0 ) έχει εξίσωση: τε: (1) χχο ΥΥο Η εξ ίσωση της εφαπτομένης ε ' της C στο Ρ , είναι: α2 β2 = I . ΧΧ 0 - ΥΥ3 ο = I <::::> 3 ΧΧ0 - ΥΥ ο - 3 = 0 (2). Λ l: Κ Η Σ Ε Ι Σ Γνωρίζουμε ότι: δ = (2,-Ι) 11 ε και δ ' = (y0 ,3 Χ 0 ) 11 ε Ό 1. Να βρείτε τις υπερβολές που έχουν κέντρο Έτσι, έχουμε: την αρχή των αξόνων, τις εστίες τους σε έναν ε ε ' <=> ' <=> ' = ο <=> 2y - 3χο = ο (3). δ l_ δ δ δ l_ o από τους άξονες των συντεταγμένων, aσύμπτω­ ύνο Λ υμε το σύστημα των εξισώσεων ( 1 ) και (3) τες y = ±J3x και διέρχονται από το σημείο και βρίσκουμε τις λύσεις: ( Χ 0 = 2, y ο = 3 ) και Μ(2,3). (χο = -2 , y0 = -3 ) . Αντικαθιστώντας στη (2) και Λ ίJ ση aπλοποιώντας, βρίσκουμε αντιστο ίχω ς τι; ε�ισc:)α) Έστω ότι μία τέτοια υπερβολή έχει τις εστίες •

0 -

{

{

-

=

-

?

?

_

ο

ΕΥΚΛΕΙΔΗΣ Β' 73 τ.3/49


Μαθηματικά για την Β ' Λυκείου

σεις: 2x - y - 1 = 0 (4) και 2x - y + l = O (5). Άρα, υπάρχουν δύο τέτοιες εφαπτόμενες. Οι εξι­ σώσεις τους είναι οι (4) και (5). Τις ονομάζουμε ε 1 και ε 2 , αντιστοίχως. Από την (4) με χ=Ο βρίσκουμε y = -I . Άρα, το σημείο Μ(Ο ,- 1 ) ανήκει στην ε 1 Συνεπώς: 12 · 0 -� ( -I) + Ii = 2r; = 2JS d(ε 1 ,ε -? ) = d(Μ,ε -? ) = ν5 5 ν4 + 1 •

-

3.

Έστω μία υπερ β ολή

χ2 y 2 C:=1 α2 - β2

. Ονο-

μάζουμε Κ την προβολή μιας εστίας της C σε μία aσύμπτωτη αυτής. Να δείξετε ότι η από­ σταση της αρχής των αξόνων Ο από το Κ, είναι ίση με την απόσταση του Ο από μία κορυφή της c. Έστω ότι Κ είναι η προβολή της εστίας Ε(γ,Ο) στην ασύμπτωτη ζ : y = Ια χ . ζ

β ρείτε τη γωνία των ασυμπτώτων της C, στην οποία βρίσκεται ο δεξιός κλάδος της. Οι ασύμπτωτες της C είναι ε 1 : y = Ια χ και

ε 2 : y = _ Ια χ . Η εφαπτομένη της C στην κορυφή Α (α,Ο) έχει εξίσωση: χ = α . Η εφαπτομένη αυτή, όπως βρίσκουμε εύκολα, τέμνει την ασύμπτωτη ε 1 στο σημείο Γ( α, β) και την ασύμπτωτη ε 2 στο σημείο Δ( α, -β). Η γωνία των ασυμπτώτων της C, στην οποία βρί­ σκεται ο δεξιός κλάδος της, είναι η γωνία θ των διανυσμάτων: ΟΓ = ( α , β) και ΟΔ = ( α , -β) . Έχουμε:

ΟΓ · ΟΔ Ι οrΙ ·Ι οΔΙ -

συνθ

-

χ

Ε

y= - αβ

y= αβ χ

' Εχουμε: y = Ι χ <:::> βχ - αy = Ο . 2 2 α (γι α τι, : β ?- = γ ?- - α-? ) = 2 α ?- γ γ'Ετσι, έχουμε: (ΕΚ) = d(Ε,ζ) = �γΙ ? = βγ? = β . �β- + α- Ν 2 - e 2 = -2 - 4 = - -I Από το ορθογώνιο τρίγωνο ΟΚΕ έχουμε: 2 4 2 e (ΟΚ) 2 = (ΟΕ) 2 - (ΕΚ) 2 = γ 2 - β 2 = α 2 => Άρα : (ΟΚ) = α = (ΟΑ ) = (ΟΑ ' ) . 2π 1 2π ' Όμοια, αν το Κ είναι προβολή της Ε στην ασύ­ συνθ = --2 <:::> συνθ = συν-3 <::> θ = 3 (γιατι Ο::;θ ::;π). μπτωτη ζ ' : y = _ Ια χ ή αν το Κ είναι προβολή της εστίας Ε ' (-γ, Ο) σε μία από τις ασύμπτωτες ζ ή ζΌ --

-

4. Η εκκεντρότητα μιας υπερβολής 1. Θεωρούμε δύο θετικούς αριθμούς α και β . Να δείξετε ότι, για κάθε ν ε Ν * , ισχύει: χ 2 y2 ν να ν + e = 2 . Να C : α 2 - β 2 = 1 (α>Ο, β>Ο) είναι η + β ν + � α β ( ν + 1) (1) I

ΕΥΚΛΕΙΔΗΣ Β ' 73 τ.3/50

I


Μ αθη μ ατικά για την Β ' Λυκείου

Επαγωγικά. "' Για ν= Ι , η ( Ι ) γίνεται: α 2 + β 2 � 2 α β . Για να δείξουμε αυτή, αρκεί να 2 αρκεί: δείξουμε ότι: α + β2 - 2α β � Ο , ( α - β γ � Ο , ισχύει. Έστω ότι η (Ι ) ισχύει για ν κ (κ Ε Ν* ) , δηλ. κ+ Ι + β κ+ Ι � α κ β ( Κ + 1 ) . ότι; Κα (2) =

Θα δείξουμε ότι ισχύει και για ν κ + 1 , δηλ. ότι: ( κ + Ι) α Κ+ 2 + β κ+l � α '+ 1 β( κ + 2) . =

Προς τούτο, αρκεί να δείξουμε ότι: β Κ+ 2 � α κ+Ι β ( κ + 2) - (κ + Ι) α κ+l (3) Κ+ κ κ+Ι Από τη (2) έπεται: β 2 � α β2 ( κ + Ι) - κα β . Έτσι, για να δείξουμε την (3), αρκεί να δείξουμε ότι: α κ β2( κ + ι) - κα κ+Ι β � ακ+Ι β( κ + 2) - ( κ + ι) α '+ 2 ' αρκεί: β2 (κ + Ι) - κα β - α β(κ + 2) + ( κ + Ι) α 2 � 0 , αρκεί: β2 ( κ + I) - 2 α β( κ + I) + (κ + Ι)α2 � Ο , αρκεί: ( κ + I )( α - β)2 � Ο , ισχύει.

{ χ + 5y + 1 15ω χ < 5,

933 y < 23 =

933 = 5(y + 23ω) + χ . Έχουμε: (Ι) Επειδή Ο � χ < 5 , η ( Ι ) είναι η ισότητα της διαίρε­ σης 933 :5. Εκτελούμε τη διαίρεση αυτή και βρί­ σκουμε πηλίκο Ι 86 και υπόλοιπο 3. Άρα χ = 3 και y + 23ω = ι 86 , οπότε 1 86 = 23ω + y (2). Επειδή Ο � y < 23 , η (2) είναι η ισότητα της διαί­ ρεσης ι 86:23, οπότε y = 2 και ω = 8 . Άρα, χ = 3 , y = 2 και ω = 8 . Όπως βρίσκουμε εύκολα οι αριθμοί αυτοί επαληθεύουν τις δοσμένες σχέσεις και άρα είναι οι μοναδικοί ζητούμενοι.

4. Να βρείτε τους φυσικούς θετικούς αριθμούς α και β για τους οποίους ισχύει: (7 α + 3 β - 16)(17 β + 15) = 32. (1) Έστω ότι για δύο αριθμούς α ,β Ε Ν * η (Ι ) ισχύει. Τότε: Ι Ίβ+ I5 l 32 => ιΊβ+ Ι5 � 32 =>ΙΊβ � Ι Ί => β � Ι και επειδή β � ι , έπεται ότι β = Ι . Αντικαθιστώντας στην ( Ι ), βρίσκουμε:

2. Ο α ριθ μ ός α2 + 2β , όπ ου α, β Ε Ζ , είναι τε­ τράγων ο ακέρ α ιου α ριθμού. Να δείξετε ότι ο (Ί α + 3β - 1 6)(1 Ί + 1 5) = 32 => Ί α - 1 3 = I => α = 2 . αριθ μ ός α2 + β είναι άθροισμα δύο τετραγώνων Άρα, τότε α = 2 και β = Ι . ακέρ α ιων α ριθμών. Με α = 2 και β = Ι βρίσκουμε εύ­ Έστω ότι α 2 + 2β = κ 2 , όπου κ Ε Ζ . Έχουμε κολα ότι η ( Ι ) ισχύει. Άρα: α = 2 και β = ι . κ 2 - α 2 = 2β και άρα ο αριθμός κ 2 - α 2 είναι άρ­ τιος. Αν ο ένας από τους αριθμούς κ και α είναι άρτιος και ο άλλος περιττός, τότε ο κ 2 - α 2 είναι περιττός, άτοπο. Άρα, οι αριθμοί κ και α είναι και οι δύο άρτιοι ή και οι δύο περιττοί. Συνεπώς, ο α­ ριθμός κ - α είναι άρτιος. Θέτουμε κ - α = 2λ , όπου λ Ε Ζ , οπότε κ = α + 2λ . Έτσι, έχουμε:

α 2 + 2β = κ 2 => α 2 + 2β = ( α + 2λ)2 => => α 2 + 2β = α 2 + 4 α λ + 4λ 2 => 2β = 4 α λ + 4λ 2 => β = 2 α λ + 2λ 2 => => α 2 + β = α 2 + 2 α λ + λ 2 + λ 2 => α 2 + β = ( α + λ)2 + λ2 •

3. Να βρείτε τους φυσικούς αριθμούς χ, y και ω , για τους οποίους ισχύουν: ΕΥΚΛΕΙΔΗΣ Β' 73 τ.3/51


Μαθηματ ι κά Γενι κής Πα ι δ ε ίας για τη Γ ' τάξ η του Λυκ ε ίου

lΎΧΑΙ Ο -Π Ι ΘΑΝ Ο -B E BAI O

Π αναγιώτης Π . Χριστό που λος - Θ αν ά σης Π . Χριστό π ου λος

Καθημερινά χρησιμοποιούμε λέξεις όπως τυχαίο, πιθανό , βέβαιο που συνδέονται με τη θε­ ωρία των πιθανοτήτων. Οι πιθανότητες αρχικά μπήκαν στη ζωή μας από τους Pascal και Fer­ mat το 1 654 για να αναλύσουν τυχερά παιχνίδια. Η πρώτη αξιωματική θεμελίωση έγινε το 1 933 από τον Ρώσο μαθηματικό Kolmogorov. Οι πιθανότητες αποτελούν βασικό εργαλείο όλων των επιστημών. Η πιθανότητα να συμβεί ένα γεγονός (ενδεχόμενο) μπορεί να εκφραστεί είτε ως κλά­ σμα μικρότερο της μονάδας, είτε ως δεκαδικός από [0- 1 ] είτε επί τοις εκατό.

Ι στορ ική Αναδ ρ ο μή στην ένν ο ια τη ς Π ιθ ανότητας

Ο Αριστοτέλης (384-322 π.Χ.) διατύπωσε τη διάκριση μεταξύ των λέξεων γνώση και γνώ­ Θεώρησε δηλαδή ότι η γνώση αφορά σε κάτι που είναι σωστό ή λάθος, ενώ η γνώμη σε κάτι που μπορεί να είναι σωστό ή λάθος. Έδωσε επίσης τις έννοιες του τυχαίου, του aπροσδόκητου και της σχετικής συχνότητας. Θεωρούσε όμως ότι το τυχαίο δεν είναι επιστημονική έννοια, ο­ φείλεται στη δική μας αδυναμία να ερμηνεύσουμε τα φαινόμενα και έδωσε το παράδειγμα: Ο­ ποιαδήποτε ανακοίνωση για το αποτέλεσμα μιας ναυμαχίας που θα γίνει την επόμενη μέρα θα είναι σωστή ή λάθος μετά το τέλος της ναυμαχίας. Πριν τη ναυμαχία καμία ανακοίνωση δε μπο­ ρεί να είναι αληθής. Ο Καρνεάδης (21 4- 1 29 π.Χ. ), που έζησε στα ελληνιστικά χρόνια, έδωσε μια πρώτη έννοια της πιθανότητας ως μορφής γνώσης, αρνούμενος την ύπαρξη κ ρ ιτη ρ ίου τη ς αλή θ ειας. Όπως γράφει ο Σέξτος ο Εμπειρικός, ο Καρνεάδης διέκρινε τρεις βαθμούς πιθανότητας (πιθανής γνώ­ σης). Ο πρώτος βαθμός πιθανότητας, η πιθ ανή φ αντασία, χρησιμοποιείται όταν ασχολούμαστε με κοινά πράγματα ή όταν δεν έχουμε καιρό, π.χ. κάποιος κυνηγημένος φτάνοντας σε ένα χα­ ντάκι φαντάζεται ότι μέσα στο χαντάκι είναι κρυμμένοι οι κυνηγοί του, οπότε χωρίς να το ξα­ νασκεφθεί αλλάζει κατεύθυνση και φεύγει από το χαντάκι. Ο δεύτερος βαθμός πιθανότητας, η απερ ίσπαστος φ αντ ασία, χρησιμοποιείται για σπουδαιότερα πράγματα όταν κάποια παράστα­ ση που μας δημιουργείται δεν έρχεται σε αντίφαση με άλλες παραστάσεις του ίδιου λογικού πλαισίου, π.χ. αν κινούμενοι σε σκοτεινό δωμάτιο δούμε ένα σκοινί στριμμένο αμέσως πηδάμε πάνω από αυτό γιατί το φανταζόμαστε ότι είναι φίδι, αλλά καθώς ξανακοιτάμε πίσω το σκοινί να είναι ακόμη ακίνητο, αποφασίζουμε ότι δεν είναι φίδι. Ο τρίτος βαθμός πιθανότητας, η δ ιε­ ξωδευ μένη φ αντασία, απαιτεί ένα ολόκληρο σύστημα από παραστάσεις να αποδειχθεί ότι έχει εσωτερική αλληλουχία και δεν αντιφάσκει με την εμπειρία, π.χ. στο τελευταίο παράδειγμα με μη .

ΕΥΚΛΕΙΔΗΣ Β' 75 τ.3/52


------- Μαθηματικά για την Γ Λυκείου το σκοινί, αφού δούμε ότι είναι ακίνητο, σκεφτόμαστε ότι ενδέχεται να είναι ακίνητο λόγω του χειμερινού κρύου, γι' αυτό παίρνουμε ένα ραβδί και το κουνάμε. Εφόσον ούτε και τώρα βλέ­ πουμε το σκοινί να κινείται καταλήγουμε να αποκλείσουμε ότι είναι φίδι. Ούτε όμως ο Καρνεά­ δης ούτε και κανείς άλλος στην αρχαιότητα, όρισε πο σ οτική έννο ια τ η ς πιθ ανότ η τας. Ο Spinoza ( 1 632- 1 677) πίστευε ότι η άγνοια της πραγματικότητας μας οδηγεί να αποδίδουμε στην τύχη ορισμένα γεγονότα. Η θεωρία Πιθανοτήτων αναπτύχθηκε από την ανάγκη να αντιμετωπισθούν πρακτικά προ­ βλήματα. Ο 1 7°ς αιώνας χαρακτηρίζεται από την ανάπτυξη του διεθνούς εμπορίου και την πλη­ ρωμή ασφαλίστρων, όπου έπρεπε να ληφθούν υπόψη τα ατυχήματα κατά τη μεταφορά. Επίσης η οργάνωση του κράτους με τα νέα δεδομένα απαιτούσε υπολογισμούς εσόδων και εξόδων. Γνωστοί μαθηματικοί συμβούλευαν τους ηγεμόνες για το ποσό που αναμένεται να συ­ γκεντρωθεί από φόρους, για το πλήθος των κατοίκων της χώρας ή του στρατού κλπ. Αναφέρε­ ται ότι ο Leonard Euler(1 707-1 783) έδωσε συμβουλές στο βασιλιά Frederick της Πρωσίας το 1 754 και το 1 763 για την τιμή πώλησης των κρατικών λαχείων. Η ανάγκη για μια αξιωματική θεμελίωση της θεωρίας πιθανοτήτων με μαθηματική αυστη­ ρότητα παρουσιάσθηκε από τον D . Hilbert στον κατάλογο των σπουδαίων άλυτων προβλημά­ των που έδωσε το 1 900. Η σημερινή αξιωματική θεμελίωση οφείλεται στον Α.Ν. Kolmogoroν το 1 933 που παρουσίασε τις πιθανότητες ως ειδική περίπτωση της θεωρ ίας μέτρ ου .

-------­

Π Ι Ν Α ΚΑΣ 1 ΣΤΗΛΗ Α

ΣΤΗΛΗ

ΔιατύπωσJΙ

Συμβολισμός

Πραγματοποιείται Το Α και το Β

Του λάχιστον ένα Α ή το Β )

Δεν

Β

AnB

εκ των δύο (το

πραγματοποιείται το Α

AuB

Α'

Ναι το Α και όχι το Β

Α-Β ή AnB'

Μόνο ένα εκ των Α, Β

( Α-Β) υ (Β-Α ) ή (Α n B ') u (B n A ')

Ε Υ Κ \ Ε Η Η Σ Β ' 7 5 τ.3/53

ΣΤΗΛΗ Γ

Διάγραμμα Venn

� �/

ι cS?� \

� ------Α

)

Α'

.�

I

___./)

I Q) . C


------

Ούτε το

Μαθηματικά για την Γ Λυκείου

(Α υ Β ) ' ή ( Α ' 11 Β ' )

Α ούτε το Β

Τ ο πο λύ ένα εκ των

-------

u�

� Β

�·

( A n B) '

Α, Β

ή

Α' υ Β '

ΑυΒ' ή ( Α ' 11 Β ) ' ή ( 8-Α ) '

Πραγματοποιείται το Α ή δεν πραγματοποιείται το Β

A n 8= 0 Ν ( Α υ Β )=Ν ( Α) +Ν (Β ) Ρ (Α υ Β )=Ρ (Α ) +Ρ ( Β )

Ειδικές περιπτώσεις

Ι . Α, Β ασυμβίβαστα

Α� Β 2.

Α υποσύνολο Β

Α Η=Α Α υ Β=Β BcA

3 . Β υποσύνολο Α

Α -,8=8 Α υ Β=Α

Α, Β ςΩ

Γενική μορφ ή

Ε ΠΙ ΤΟΥ ΤΥΠΟ Γ ΡΑΦ Ε ΙΟΥ . . . Γ Ι Α ΤΟ Τ Ε ΥΧΟΣ 74 . . .

1.

Στην άσκηση 4 ερώτη μα (γ) της Άλγεβρας Λυκείου εκ παραδρομής αντί να γρα-

---1- Ι ισχύει, αν η f είναι συνεχής οπότε 1 και η Γ συνεχής (πρόταση που θέλει από­

φεί

δειξη)

Α'

R

γράφτηκε

�.

2 . Στην Άσκηση 6 της Κατεύθυνσης Γ Λυ­ 1 1 κείου, ο ισχυρισμός l i m f - ( y) f - ( Ι ) = Ι , y -> 1

=

δηλαδή με ελεύθερη γραφή : χ

---1- Ι όταν

y

συνάδελφος Κ. Σερίφης έδωσε παρά­ δειγμα όπου y ---1- Ι ενώ χ ---1- -ω (προφανώς για μη συνεχή συνάρτηση) .

Ο

ΕΥΚΛΕΙΔΗΣ Β ' 7 5 τ.3/54


------- Μαθηματικά για την

Είναι

B c A Β άρα Ρ(Α) ::::; ή Ρ(Β) ::::; Ρ(Α _ Β) οπότε Ρ(Α Β ) max { P(A), Ρ(Β)} ::::; Ρ(Α Β) Ακόμη Ρ( Α Β) � Ο αλλά Ρ(Α _ Β) = Ρ ( Α ) + Ρ(Β)- Ρ(Α Β) ή Ρ(Α Β) = Ρ(Α) + Ρ(Β)- Ρ(Α Β) δηλα­ δή Ρ( Α ) + Ρ(Β)- Ρ(Α Β) � Ο άρα Ρ(Α Β ) ::::; Ρ(Α) + Ρ(Β) αν το Ρ(Α) + Ρ(Β) είναι μ εγ αλύ τερο της μονάδας τότε γ ρ άφου με Ρ( Α Β) ::; ι . Είναι Α ς Α Β ή

-------

άλλος τρόπος με τους είναι: ·

,. ., , .

και πως αυτός αξιοποιείται στις ασκήσεις και τα προβ λή ματα .

1)

1 και Ρ 2 4 Ρ(Ι ) (2)+ = Ρ (3) ποια είναι η πιθανότητα για 3 κάθε απλό ενδεχόμενο;

Αν

Ω={ 1 ,2,3}

'Εστω

Ρ(2)=χ

Ρ( 1 )=2Ρ(2)- -

τότε

-

i)

2)

Αν Ω={ 1 ,2 ,3,4,5,6} είναι ο δειγματικός χώ­ ρος που προκύπτει από τη ρίψη ενός μη α­ μερόληπτου ζαριού και ισχύει Ρ(ί)=z-ϊ-ι για 1 i � 3 και Ρ(ί)= -.- για i � S. 2ι 4 -

Β)=1 -

Ρ(1)=2χ- � και 4

1 1 Ρ(3)=χ2 + - (2χ- - ) 4 3 Ισχύει Ρ( 1 )+Ρ(2)+Ρ(3)=1 1 1 ι 2χ- - +χ+χ2 + - (2χ- - )= 1 4 3 4 1 χ2 +2χ+χ+ � χ- � - - - 1 =0 1 2χ2 +44χ-1 6=0 3 4 12 2 3χ + 1 1 χ-4=0 -1 1 13 χ1 = =-4 απορριπτεται ' 6 -1 1 + 1 3 1 χ2 = 6 3 1 Οπότε Ρ ( Ι )= � -� = 2_ , Ρ(2)= 3 , 3 4 12 Ι 1 5 9 3 Ρ(3)= - + - . - = - = 9 3 1 2 36 1 2

Α Β ' c A ή Α Β ' c B ' άρα Ρ(Α Β ')::; Ρ(Α) ή Ρ(Α Β ')::; Ρ(Β ')=1 - Ρ(Β) Έχουμε P(A)+P(B')-l :::;P (A Β ') ή Ρ(Α) - Ρ(Β) ::::; Ρ(Α Β ') δηλαδή max {O, Ρ(Α)-Ρ(Β) }:::;P (A-B):::;m in {Ρ(Α), 1 -Ρ(Β) } .

Ρ(Α' Β)=Ρ(Α') +Ρ(Β)- Ρ(Α Ρ(Α)+Ρ(Β)-Ρ(Β-Α)=

• . ι ·· .

Α' _ Β=Α' (Β ') '=( Α Β ') '= (Α-Β) ' και Ρ(Α' Β)= Ρ((Α B) ')=l -P(A-B) = 1 Ρ(Α)+Ρ(Α -Β ) .

Είναι

(Α Β)' = Α' Β' Δεν πραγματοποιείται ούτε το Α ούτε το Β , αλλά πραγματοποι­ ούνται ταυτόχρονα τα Α ' και Β ' . ii) (Α Β)' = Α' Β' Δεν πραγματοποιούνται ταυτόχρονα τα Α και Β, αλλά πραγ­ ματοποιείται το Α ' ή το Β Ό

Λυκείου

= 1-Ρ(Α)+Ρ(Β)-[Ρ(Β)- Ρ(Α Β)]= = 1 -Ρ(Α)+Ρ(Β)-Ρ(Β)+ Ρ(Α .· Β)= = 1-Ρ(Α))+ Ρ(Α - Β)

Ισχύουν τα παρακάτω :

Α Β c Α ή Α Β c Β άρα Ρ(Α Β)::; Ρ(Α) ή Ρ(Α Β)::; Ρ(Β) οπότε Ρ(Α Β)::; min {Ρ(Α), Ρ(Β) } Ακόμη Ο::; Ρ(Α - Β) επειδή ακόμη Ρ(Α Β) ::::; 1 ή Ρ(Α) + Ρ(Β)- Ρ(Α Β) ::::; 1 άρα Ρ(Α) + Ρ(Β)-1 ::::; Ρ(Α Β) αν Ρ(Α) + Ρ(Β)- 1 είναι αρνητικό τότε γράφουμε Ο :::; Ρ (Α Β) δηλαδή max {O, Ρ(Α) + Ρ(Β)1 } ::::; Ρ(Α Β).

Γ'

ς � Να βρε θ ούν οι π ιθ ανότητες των α­ πλών ενδεχομ ένων.

ΕΥΚΛΕΙΔΗΣ Β' 75 τ.3/55


------ Μαθηματικά για την Γ Λυκείου

β) Αν Α={ λ ε Ω I η f είναι γνησίως αύ­ ξουσα στο R} όπου f(x)=( λ-l)x 3-λχ 2 +2χ+λ Λύ ση 2 α) Αν i ::; 3 P( l )=T 1 - 1 =T = _!_ , 4 P (2)= T2- 1 =T3 = , Ρ(3)=Τ3 - 1 =2 -4 = 1 1 = _!_ αν i � 5 Ρ(5)= 2. 5 - 4 6 1 - 1 Ρ(6) 2.6 - 4 8

i

επίσης ισχύει

6

Σ Ρ(ί) = 1

άρα

i=l

1 1 1 I 1 13 - + - + - + Ρ (4)+ - + - = 1 Ρ(4)= - . 48 4 8 16 6 8 2 β) f ' (χ)=3 (λ- 1 )χ -2λχ+2 για να είναι η f γνησίως αύξουσα στο R πρέπει f ' (χ)� Ο . Αν λ= 1 τότε f ' (χ) =-2χ+2 μή δεκτή τιμή του λ γιατί η Γ αλλάζει πρόση μο στο Χ0 = 1 οπότε η f αλλάζει μονοτονία. Αν ληt: 1 τότε η Γ(χ) είναι τριώνυμο και για να είναι αυτό μεγαλύτερο ή ίσο από μηδέν για κάθε χ ε R πρέπει η διακρίνουσά του να είναι μικρότερη ή ίση του μηδενός και το α=3 (λ- 1 )>0. Αλλά Δ=4λ2 -24(λ- 1 )= 4λ2 -24λ+24 = 4(λ2 -6λ+6 ) 2 οπότε Δ::;Ο<::::>λ -6λ+6::;0 . : - r.

'

:-� 7

'

-. - οποτε Δ =3 6-24= 1 2 και λ 1 , 2 =�=---

λι =3- J3 και λ2 =3+ J3 • 3 - J3

ο -

ι

3

2

-

-

4 -

-

-

5

6 -

-

Πρέπει λ 1 ::;λ�Ξλ2 και λ> 1 επομένως δεκτές τιμές λ=2, λ=3 και λ=4 δηλαδή Α= {2,3 ,4 } και

p( Α ) =p( 2 )+p(3 ) +ρ( 4 ) = _!_ + __!_ + � = _!_!_ 8 1 6 4 8 24 Τ α πρ ο βλή ματα πιθανοτήτων και η α­

ντ ιμπώπισή τους Αφού μελετήσουμε προσεκτικά το πρό­ βλη μα φροντίζουμε να θέτουμε ως ενδεχόμενα Α και Β κάποια που περιγράφονται στο πρό­ βλη μα πιθανόν έμμεσα αποφ ε ύγου με γι α τα

------

Α , Β π ροτάσε ις σίJνθετες ή π ρ οτάσε ις με άρ­ νη σ η επίσης αν θέσουμε ως Α κάποιο ενδεχό­ μενο πρέπει να προσέξουμε να μη θέσουμε ως Β το συμπλή ρωμά του π.χ. αν Α τεθεί το ενδε­ χόμενο «είναι άνδρας» τότε δεν ονομάζουμε Β το ενδεχόμενο «είναι γυναίκα», αλλά Α ' . Για σύνθετες προτάσεις συμβουλευτείτε τον πίνα­ κα 1 . Α σ κή σε ις

1. Σε ένα σχολείο το 25% των μαθητών δεν έ­ χει ηλεκτρονικό υπολογιστή και το 15% έχει κινητό αλλά όχι ηλεκτρονικό υπολογιστή. α) Ποια η πιθανότητα αν επιλέξουμε τυχαία ένα μαθητή να μην έχει ούτε κινητό ούτε Η/Υ β) να μην έχει κινητό ή να έχει Η/Υ. ΛίJ ση Θέτω Α: «έχει Η/Υ», Β : «έχει κινητό» άρα Ρ(Α ' ) = 0,25 και Ρ(Α)= 1 - 0,25=0,75 Ρ(Β-Α)=Ο, 1 5 ή Ρ(Β)- Ρ( Α -ιΒ) = Ο, 1 5 α) «ούτε κινητό ούτε Η/Υ»: (Α _ιΒ) ' Ρ( Α_ι Β) ' = 1 -Ρ(Α _ΙΒ)= 1 -[Ρ(Α)+Ρ (Β )-Ρ ( Α r B ) ]

= 1 -[0, 75+0, 1 5]= 1 -0,90=0, 1 ο β) Ρ(Β Έ .r .�. ) =Ρ(Β ')+Ρ( Α )-Ρ(Α -ι Β ')= = 1 -Ρ(Β)+Ρ(Α)-Ρ( Α )+Ρ(Α - r B )= = 1 -[Ρ(Β)-Ρ(Α -ι Β) ] = 1 -0, 1 5=0, 8 5 .

2. Ένα Λύκειο έχει 80 μαθητές στη Γ τάξη εκ των οποίων 30 έχουν επιλέξει Θεωρητική κατεύθυνση και 45 μαθητές έχουν επιλέξει μαθηματικά γενικής παιδείας(ΜΓΠ) ως πα­ νελλαδικά εξεταζόμενο. Οι μαθητές της θε­ τικής και τεχνολογικής κατεύθυνσης που δεν έχουν επιλέξει μαθηματικά γενικής παι­ δείας είναι 1 Ο. Αν επιλέξουμε τυχαία ένα μαθητή. α) να βρεθεί η πιθανότητα να είναι της θεω­ ρητικής κατεύθυνσης ή να έχει επιλέξει μα­ θηματικά γενικής παιδείας, β) την πιθανότητα να είναι της θεωρητικής κατεύθυνσης και να έχει επιλέξει ΜΓΠ. Πόσοι είναι αυτοί οι μαθητές; Λ ύ ση

Α : «είναι

μαθητής θεωρητικής κατεύθυνσης» Β : «ο μαθητής έχει επιλέξει ΜΓΠ»

ΕΥΚΛΕΙΔΗΣ Β' 75 τ.3/56


------ Μαθηματικά για την Γ Λυκείου

Ρ(Α)=

Ν(Α) Ν(Ω)

=

30

=�

γ) Τα ευνοϊκά αποτελέσματα είναι Β = { αα, αβαα, αβαβα, β β, βαββ, βαβαβ } είναι Ν(Β)=6

80 8

4 5 _2_ Ρ(Β) = Ν(Β) Ν(Ω) 80 1 6 ' α) Ρ(Α -Β ')= Ρ(Α _ 8) '= 1 -Ρ( Α._ Β) άρα

=

=

άρα Ρ(Β)=

= _!_8 = 1 -Ρ(Α -· Β)

. Ρ(Α - Β ' )= !Q_ 80

------

����

=

1

ή Ρ(Α)=60% .

4. Σε ένα p et-shop υπάρχουν λιγότερα από 50 άσπρα και μαύρα γατάκια. Επιλέγοντας τυ­ ή Ρ(Α _Β)= 7_ και χαία ένα γατάκι η πιθανότητα να είναι αρ­ 8 σενικό-μαύρο είναι 30% ενώ η πιθανότητα β) Α·15 ___2 __!_ = Ρ( Β) =Ρ(Α)+Ρ(Β)-Ρ(Α· - Β ) = ' ' ' να ειναι ασπρο ειναι -1 . 16 8 16 3 N (A n B) , οποτε Ρ(Α -Β)= α ) Ποιά η πιθανότητα να είναι μαύρο­ Ν(Ω) θηλυκό; 1 β) Πόσα είναι συνολικά τα γατάκια; 80=5 μ αθητές. Ν(Α Β)= -

-

16

Λ ί! ση

Έστω Α: « άσπρο γατάκι» οπότε Α ' : «μαύρο γατάκι» Και Β: « αρσενικό γατάκι» οπότε Β ' : « θηλυκό γατάκι»

3. Στ α play off ενός πρωταθλήματος 2 ομάδες φτάνουν στον τελικό και παίζουν μεταξύ τους τόσους αγώνες ώστε κάποια από τις δύο να πετύχει 2 συνεχόμενες ή 3 συνολικά νίκες ώστε να ανακηρυχθεί πρωταθλήτρια. (Κάθε ομάδα έχει ίδια πιθανότητα νίκης σε κάθε αγώνα). α ) πό σ οι τ ο πολύ αγώνες θα γίνουν; β) Ποιά η πιθανότητα να ανακηρυχθεί πρωταθλή­ τρια μια ομάδ α μετά από 3 αγώνες; γ) Επί­ σης ποιά η πιθ ανότητα η ομάδα που θα κερ­ δίσει στον πρώτο αγών α να είναι πρωτα­ θλήτρια.

Δίνεται Ρ(Α)= _!_ αρσενικό-μαύρο: Β - λ και 3 3 Ρ( �·-. "". ) = 30 0/1'0 η' 10 α) μαύρο-θηλυκό: A 'nB ' ή (AuB) ' οπότε Ρ(ΑυΒ) ' = 1 -Ρ(ΑυΒ) ( Ι ) αλλά δίνεται 3 3 Ρ( ;.., .-. ..... . . ) = - <=> Ρ( ,, ...., ) = - <=> 10 10 .-

ΛίJ ση Συμβολίζου με α νίκη της πρώτης ομάδας και β νίκη της δεύτερης. .

.

--

·

-::

:=(:{'_-==-�

---

--

..

-

_ . ·... .-

.

ζ-{

---- β . . / __ _

� � {

-

-

-

,-. ..:: __ μ _ __ _. . .-

β

1:::{"

-

---

.:::.:.=_

β

- · ι,Λ

. _

Ρ(ΑυΒ) · Ν(Ω) δηλαδή Ν (ΑυΒ)= !i · Ν(Ω) αφού 30 Ν(ΑυΒ) είναι φυσικός αριθμός πρέπει li · Ν (Ω) 30 να είναι φυσικός και αυτό συμβαίνει αν Ν(Ω) είναι πολλαπλάσιο του 30, αλλά Ν(Ω) <50 οπότε Ν(Ω)=30

--

Ω= { αα, αβαα, αβαβα, αβαββ, αββ, ββ, βαββ, βαβαβ, βαβαα, βαα} Ν( Ω )= 1 0 α) Θα χρειαστούν 5 το πολύ αγώνες Ν(Α)=2 β ) Α= { αββ, βαα} -

-

Ρ( Α)=

Ν (Α) � ή Ρ(Α)=20%. Ν(Ω) 1 0 =

ΕΥΚΛΕΙΔΗΣ

-

<::::> Ρ( Β .·ι -Ρ(ΑΠΒ) = � 10 οπότε Ρ(ΑυΒ)= Ρ(Α)+ [Ρ( Ε ι - Ρ(Α Π Β)]= 1 3 1 = - + - = -9 3 1 0 30 επομένως από σχέση ( 1 ) έχουμε ' 19 11 P(Au B) = 1 -P(AuB)= I - - = 30 30 Β) υ Ν(Α ' Ρ(Α u Β) = ' Ν (Α υ Β)= αρα β) ειναι Ν(Ω)

_

3

'

..ι

·

(

. .

Β'

75 τ.3/57


------

1.

Μαθη ματικά για την Γ Λυκείου

Επομένως Ρ(Β)= � και Ρ(Α)= _2__ 5 25 Ας Β οπότε AnB=A και AuB=B

Ν α βρεθεί η ελάχιστη τιμή της παράστασης :

Π= Ρ2 (Α)·Ρ(Α')-Ρ(Α)'Ρ(Α ')+Ρ(Ω) , όπου Α ενδεχόμενο ενός δειγματικού χώρου Ω p(A')= 1 -x

Π( χ)= x2( 1-x)-x( l-x)+ 1 = -χ 3 +2χ2-χ+ 1 Π '(χ)= -3χ2+4 χ-1 Π '(χ)=Ο<=:>-3χ2+4 χ- 1 =0 <=> 3χ2-4 χ+ 1 =0, Δ= 1 6-1 2=4 ---οο

χ

Π '(Χ)

\\\\\\\

Π(Χ)

\\\\\\\

Οποτε '

και χ 1 ,

ι

-

I

!

3. Έστω Α,Β ενδεχόμενα ενός δειγματικού χώ­ ρου Ω και Ρ(Α), Ρ(Β), P(AnB), P(AuB) πα­ ρατηρήσεις μιας μεταβλητής Χ με διάμεσο δ=-1 . Δίνεται επίσης ότι η μέση τιμή των 4 Ρ(Α), Ρ(Β), P(AnB), Ρ(Β-Α), Ρ(Α-Β), P(AuB) είναι χ = ..!_5 . Υπολογίστε τις πιθανότητες P(AnB), P(AuB).

2 1 3

ο

ο

i

+οο

1 +

\\\\\\\\\\\

ο

I

είναι

και άρα Ρ(ΑυΒ)= Ρ(Β)= � 5 Ρ(Β-Α)= Ρ(Β)-Ρ(ΑΓΙΒ) = Ρ(Β)-Ρ(Α)= 3 9 6 5 25 25

ενός πειράματος τύχης. Θέτω p(A)=x οπότε 0::Ξ:χ::Ξ: 1 και επίσης είναι Ρ(Ω)= 1 επομένως

--------­

\\\\\\\\\\\\

1 3 1 1 Π ιnin =- ( - ) +2( - ) 2 - - + 1 = 3 3 3 1 2 1 23 =- - + - - - + 1 = 27 9 3 27

:::; Ρ(Α) :::; Ρ(Β) ::Ξ:Ρ(ΑυΒ) P(AnB) ::Ξ:Ρ(Β) ::Ξ: Ρ(Α) ::Ξ: Ρ(ΑυΒ) οπότε Είναι P(AnB)

δ=

2. Αν Ας Β και A:;t:0 όπου Α, Β ενδεχόμενα ενός δειγματικού χώρου Ω και Ρ(Α), Ρ(Β) είναι ρίζες της εξίσωσης: χ3+λ(λ+1) χ 2+λ3 χ=Ο , λ ε R ί) να βρεθούν συναρτήσει του λ οι πιθανότη­ τες Ρ(Α), Ρ(Β) ίί) αν 5λ2-13λ+6=0 να υπολογίσετε τις πιθα­ νότητες P(AuB), Ρ(Β-Α).

Ρ ( Α) + Ρ(Β) Ρ ( Α) + Ρ(Β) . Άρα = _!_ και ε2 2 4

πομένως

Ρ(Α)+Ρ(Β)= _!_

(1)

2 - - �� +Ρ(Β) +�Α nτη +�Β-�+ �Α-τη+�Αυτη χ6 3Ρ(Α) + 3Ρ(Β) - 2 Ρ (Α n Β) χ 6 χ = 3Ρ(Α) + 3Ρ(Β) - 2 Ρ(Α n Β) οπότε 6 3Ρ(Α) + 3Ρ(Β) - 2Ρ(Α n Β) l 5 6 _

i ) χ 3+λ(λ+ l )χ2 +λ3 χ=Ο <=>χ[χ 2+λ(λ+ Ι )χ+λ3 ]=Ο <:::>χ=Ο (μη δεκτή αφού A:;t:0) ή χ 2 +λ(λ+ I )χ+λ3 =Ο είναι Δ=λ2 (λ+ 1 ) 2--4λ3 =(λ2-λ)2 οπότε 2 _ λ ( λ + Ι ) ± (λ - λ) ΧΙ ?2 2 . και τελικά χ=λ ή χ=λ Αφού λε (Ο, I ) επο­ μένως λ2 <λ ακόμη έχουμε Ας Β άρα Ρ(Α)=λ2 ,Ρ(Β)=λ ii) Δίνεται 5λ2-1 3λ+ 6=0 οπότε Δ= Ι 69-1 20=49

_

Άρα

3 [Ρ(Α)+Ρ(Β)]-2 P(AnB)= �

γω της σχέσης

(1)

5

2P(AnB)= _!2 - .!3_ 10

Επίσης

3 20

και λ = 1_ , λ2 =2 που απορρίπτεται . 5 1

Β'

7 20

75 τ.3/58

lO

και λό-

από τη σχέση αυτή προκύ-

6 1 πτει: 3 · - -2 P(AnB)= 2 5

·-

ΕΥΚΛΕΙΔΗΣ

ή

ή

δηλαδή

P(AnB) = ]__ 20

Ρ(Αυ Β)=[Ρ(Α)+Ρ(Β )]-Ρ(ΑnΒ)= _!_ 2


------ Μαθηματικά για τη ν Γ Λυκείου

Ολοκλήρωση

Παραγώγιση

Σωτήρης Ε. Λουρίδας ο

iii) Άκρα του πεδίου ορισμού της συνάρτησης. Θα πρέπει επίσης να έχουμε κατά νου ότι � 11 � ''�� της γραφικής παράστασης της f : Α � IR. (Α διάστημα) σε θέση χ = χ 0 ,όταν: i) Το χ 0 είναι εσωτερικό σημείο του πεδίου ορι­ σμού της f με ( ( χ 0 ) = Ο. Προσοχή: Η ( ( χ0 ) = Ο δεν είναι ικανή συνθήκη ώστε στο χ 0 να έχουμε σημείο καμπής της γραφι­ κής παράστασης της f) ii) Το χ 0 είναι εσωτερικό σημείο του πεδίου ορισμού της f και Γ τ · · η τιμή ( (χ0 )

( ΞJ χ ε Α )( Ρ (χ) ) σημαίνει: Υπάρχει ένα τουλάχι­ στον χ ε Α, ώστε να ισχύει Ρ( χ), ('ν' χ ε Α) ( Ρ (χ) ) σημαίνει: Για κάθε χ Ε Α, ισχύει Ρ( χ ). Όπως ακριβώς αποδίδονται στο βιβλίο:

, '"

μαθήματα κλασσικής αναλύσεως: Λάμπρου Ντό­ κα, Καθηγητού Πανεπιστημίου Πατρών και στα διεθνούς κύρους Μαθηματικά βιβλία: 1 . Duden, Rechnen und Mathemat ik, 2. Schauιη ' s Outline of Theory and Problems of

Discrete Mathematics .

;[

- 1

...

_.

:

'

Θεωρούμε τις συναρτήσεις f : [α, β] � IR., g : [α, β] � :JR. που είναι συνεχείς στο [α, βJ και παραγωγίσιμες στο (α, β). Να α­ ποδειχθεί ότι: ( 3 χ0 ε ( α, β} ) {g ( β} -g ( α} ] r' ( χ0 ) = = [ f ( β } - f ( α ) J g ' ( Χ 0 )) ( ) *

:: • Γνωρίζουμε ότι θα πρέπει να θεωρήσουμε συνάρτηση ,έστω h : [ α, β ] � IR. συνεχή στο διάστη­ μα [α, β] και παραγωγίσιμη στο διάστημα (α, β) που αν είναι δυνατόν να είναι γραμμικός συνδυασμός των f +c, g + m με c,m πραγματικές σταθερές ( αφού ( f + c ) ' = (και (g + m )' = g ' και με βάση την σχέση (*)), με σκοπό να εφαρμόσουμε για αυτήν το θεώρη­ μα του Rolle (θυμόμαστε δηλαδή την ιδέα απόδειξης του θεωρήματος της Μέσης τιμής του Διαφορικού Λογισμού). Έστω συνάρτηση ,

(•)

h : [α,β] � ΙR., h (χ) = [g (β) - g(α)} [f(x) + c ] - [f(β) - f(α)J [g (x) + m ] ( * * ) με h (α) = Ο και h (β) = Ο. Από εδώ προκύπτει: c = -f (α) , m = -g (α) . Όντως για τις τιμές αυτές

-οο

.1 ,

ΕΥΚΛΕΙΔΗΣ

J 1ί

' '� "

1.1

Θεωρούμε γνωστό τον ορισμό της συνεχούς συ­ νάρτησης και τις βασικές προτάσεις που αναφέρο­ νται στην έννοια της συνέχειας. Υπενθυμίζουμε ότι απαραίτητη προϋπόθεση για να είναι μία συ­ νάρτηση f : Α � IR. με Α ς IR., συνεχής στην θέση χ 0 είναι το χ 0 να ανήκει στο πεδίο ορισμού της Α. Θεωρούμε επίσης γνωστό ότι η ΜΗ ύπαρξη της συνέχειας της f : Α � IR. με Α ς IR. , σε σημείο χ 0 έχει σαν αποτέλεσμα την ΜΗ ύπαρξη της πα­ ραγώγου στην θέση χ 0 .Ο μ ω ς θα πρέπει να θυ­ μόμαστε ότι η συνέχεια της f στην θέση χ 0 δεν σημαίνει πάντα την ύπαρξη της παραγώγου ( ( χ 0 ) στην θέση αυτή. Θα πρέπει επίσης να θυ­ μόμαστε ότι η f : Α � ΙR. με Α ς IR., δεν έχει παράγωγο σε σημείο χ 0 Ε Α όταν οι πλευρικές παράγω­ γοι στο σημείο χ 0 Ε Α δεν είναι ίσες ή όταν υπάρχει σημείο P (x 0 , f(x 0 ) ) της γραφικής της παρά­ στασης που η κλίση της τέμνουσας ΡΤ τείνει στο +οο, αριστερά του χ 0 και στο δεξιά ή στο -οο αριστερά του χ 0 και στο +οο, δεξιά του. Είναι χρήσιμο να έχουμε κατά νου ότι i - της γραφικής παράστασης της f : Α � IR. (Α διάστημα), στις εξής περιπτώσεις: i) Στα Εσωτερικά σημεία του Α που μηδενίζουν την πρώτη παράγωγο (Προσοχή : Η ( ( χ 0 ) = Ο δεν είναι ικανή συνθήκη ώστε να υπάρχει ακρότατο της f στην θέση χ 0 ). ii) Εσωτερικά σημεία όπου η πρώτη παράγωγος υπάρχει. .;; i ι

: :ic

από την (**) παίρνουμε εύκολα h ( α) = h (β) = Ο. Επομένως έχουμε: ' i' η : Θεωρούμε την συνάρτηση h : [α, β] � IR., h (χ) = [ g (β) - g (α) J [ f (χ) - f (α) J -

-[f (β) - f ( α) J [g ( χ) - g ( α) ] . Η συνάρτηση αυτή

Β'

προφανώς είναι συνεχής στο [α, β], παραγωγίσιμη 75 τ.3/59


------

Μ αθηματικά για την

Γ

Λυκείου

-------­

στο (α, β) .Εύκολα βλέπουμε ότι Αυτό προκύπτει άμεσα από το θεώρημα του h(α)=h(β)=Ο.Επομένως εφαρμόζεται το θεώρημα Cauchy, αν λάβουμε υπ' όψη ότι g' ( χ 0 ) * Ο, γιατί του Rolle από όπου προκύπτει ότι: αν ίσχυε g ' ( χ 0 ) = Ο, θα είχαμε ( χ 0 ) = Ο, που εί( ΞJχ0 ε ( α, β) ) ( [g ( β ) - g ( α) ] f' ( χ0) = [ f (β ) -f ( α) J g' ( Χ0 ) ) ναι άτοπο από την υπόθεση περί μη( κοινής ρίζας. Π α ρ ατη ρή σεις: ίν) Υπάρχει και 3η εκδοχή του θεωρήματος i) Από πλευράς τεχνικής η προηγούμενη απόδειξη Cauchy προερχόμενη από την 2η όταν αντικαταμπορεί επίσης να πραγματοποιηθεί, θεωρώντας στήσουμε τις απαιτήσεις [ g ( α ) * g ( β ) και οι τηνσυνάρτηση : h : [ α, β ] � IR, h ( χ) = ( , g δεν έχουν κοινή ρίζα στο (α, β)], από την [ g ( β ) - g ( α ) J f ( χ ) - [ f ( β ) - f ( α ) J g ( χ ) , μόνο που πρόταση ( Vx ε ( α, β ) ( g' (χ) * Ο ) που οδηγεί στην ) από εδώ παίρνουμε h(α)=h(β) χωρίς κατ' ανάγκη να έχουμε την ισότητα με το μηδέν. Απλά από g ( α ) * g ( β ) (;). πλευράς μεθοδολογίας όταν 'ζητάμε προέλευση n α ρ {tδειγμα : από παραγώγιση ' πρέπει να δίνουμε βάση Έστω f : [ α, β ] � ΙR ,μεα :2: Ο, συνεχής στο διά­ στην ισότητα ( f + c ) = ( ( c σταθερά ). Ας δούμε στημα fα, β] και παραγωγίσιμη στο διάστημα ένα παράδειγμα: 'Έστω συνάρτηση f : IR � IR, (α, β).Να αποδειχθεί ότι υπάρχουν παραγωγίσιμη στο IR , με lim f (χ) = Ο. Αν υπο·

'

θέσουμε ότι το

lim

χ -t +χ

( (χ)

χ ---t +χ·

υπάρχει, τότε να υπο-

λογιστεί. Προφανώς αρκεί να προσδιορίσουμε το lim ( f (χ) + c )' ( c -::1= Ο ) ,όταν έχουμε X -t +:x: lim

( f (χ) + c) = c

lim

( f (χ) + c) =

X -t +X·

-::1=

Ο.

r'

( ) ( Χ ι ) = ( β + α ) (2 Xz = ( β z + βα + α z ) ( ( χξ) . χ2 3χ3

Απόδειξη :

Εφαρμόζοντας τρεις φορές την 2η εκδοχή του θεω­ ρήματος του Cauchy για τα ζεύγη των συναρτήσεων ( f( x ) ,g( χ ) = χ) , ( f { x ) ,g( χ) =xc ) , ( f{ x),g ( χ) =χ3 ) ,

f( β) -f ( α) f' (x1 ) , f( β) -f ( α) f' (x2 ) - 1 β- -α2 ---, β-α 2χ2 Χ f(β) -----7f(α) f ' (x ) ----'---Ξ-'= --,3- . ' Ετσι έχουμε το ζητούμενο. . x {f(x) + c) (�) . {x{f(x) + c))' 3 β - α3 3χ-3 = lιm = . lιm Χ Χ Προφανώς ισχύει: Χ1 , χ 2 , Χ 3 > Ο. = = Δ� ( r (χ) + c + xf' (χ)) c => }i�., χ( (χ) ο 2) Έστω μη σταθερή συνάρτηση f : lR � lR μεf ( χ + y ) = e 'Y f ( χ ) f ( y ) , V x, y ε IR ( ) και lim _!_ = Ο => lim ( (χ) = Ο. Εδώ ας επιση- παραγωγίσιμη στην θέση Ο, με ( ( Ο) = Ο . Ν α χ Χ ---7 +Χ:

lim

Χ ---7 +Χ

Χ ---> +χ

x {f(x) + c) =

c

-::1=

Ο, με

παιρνουμε.. ,

_

--

>

_

Χ -> +'>)

*

χ ---+ + :Ν

Χ ---7 +Χ

μάνουμε ότι το θέμα αυτό θα μπορούσε να αντιμε­ τωπιστεί και με το Θ. Μ. τ. ii) Το θεώρημα αυτό μπορεί να χρησιμοποιηθεί σαν λήμμα (αφού προηγηθεί η απόδειξή του) σε πάρα πολλά προβλήματα. Για παράδειγμα αν g(x)=x αμέσως έχουμε το γνωστό μας θεώρημα της μέσης τιμής του Διαφορικού Λογισμού. Το ί­ διο συμβαίνει και για τις δύο επόμενες εκδοχές. iii) Έχουμε και την εξής 211 εκδοχή του θεωρήμα­ τος Cauchy: Αν θεωρήσουμε συναρτήσεις f : [ α, β ] � IR, g : [ α, β ] � IR, συνεχείς στο διάστημα [α, β] και παραγωγίσιμες στο (α, β) ώστε g ( α ) * g ( β ) και οι ( ,g ' δεν έχουν κοινή ρίζα στο διάστημα (α, β), ισχύει ότι

[

f' (x0 ) f(β) - f(α) ( ΞJ χ ε ( α, ) ) g ' x ) = g ( β ) - g ( α ) J ' ο ( o j

β

βρεθεί ο τύπος της συνάρτησης f.

Λύση :

Παρατηρούμε ότι: (•)

(•)

χ = y = O � f (O) = I ν f(O) = Ο.

Αν f ( Ο ) = O � f( χ) = Ο, Vx ε IR, y=O

πράγμα άτοπο

αφού υποθέσαμε ότι η f δεν είναι σταθερή. Τελικά έχουμε f(0)= 1 .

f(x) - f (O) = 0 � lim f(x) - 1 = 0 . χ-->0 χ χ-ο Για τον τυχόντα χ 0 ε IR* ισχύει : . f (x0 + h ) - f (x0 ) e h'" f ( x0 ) f ( h ) - f (x0 ) lιm = lιm = h--> 0 h--> 0 h h . e1' '" (f ( h) -1) +e1' x" -1 . -1 =lιmd"'f(h) f( Χο) = f( Χ0 ) limι ( ( 0 ) = 0 � lim χ-->0

ιι-4J

h

ΕΥΚΛΕΙΔΗΣ Β' 75 τ.3/60

Ιι--.ο

-""-----'-

h


------ Μαθηματικά για την Γ Λυκείου στο (α, β] και παραγωγίσιμη στο (α, β) .Να απο­ h· u = f ( Χ0 ) iim e 1'· x ., f ( hh) - 1 + e xh - 1 , δειχθεί ότι, υπάρχουν χ 1 , χ 2 ε ( α,β)με χ1 :;e x 2 ώστε . e •' x " - 1 ( � ) . ( eι' χ" - 1 ) . . f(h) - 1 = 0, \ιm r' ( χ . ) r' ( χ 2 ) + g ' ( χ . ) g ' ( χ 2 ) r ( χ . ) r ( χ 2 ) = ο, lιm = lιm h . = με h h για κάθε συνάρτηση g : [α, β ] IR που είναι 1 = χ lim e ' · x u = χ . συνεχής στο (α, β( και παραγωγίσιμη στο (α, β) ώστε ( \i χ ε (α, β) ) ( g (χ) Ο ). h�O

[

)

IHO

IHO

ο

IΙ � Ο

--------­

h�O

ο

χ

Συνεπώς ισχύει: f' (x) = xf (x) ,Vx εlR=>e:f' (x) +

+-f ) f (x) = Ο =; [e-f x )) = f{

Ο, Αρ α

_.:Ξ_ e 2 f(x) = c,V'x ε iR (χ=Ο) => c = l => f(x) = e�2 ,V'x ε iR .

3)

Υποθέτουμε ότι υπάρχε ι συνάρτηση

:;e

·

Απόδειξ η :

Α ρκεί να υπάρχουν Χ 1 , Χ 2 ε (α, β) : f ' (χ 1 ) = -g ' ( Χ 1 ) f ( Χ 1 ) και ( ( Χ 2 ) = = g ' ( χ 2 ) f ( χ 2 ) , δηλαδη ( ( Χ 1 ) + g' ( Χ 1 ) f (x 1 ) = Ο και ( ( x 2 ) - g' (x 2 )f( χ 2 ) = 0 ή e g (xι)( (χ . ) + ( e g (x,) ) ' f ( χ . ) = ο

f : IR � IR με f ( IR ) ς [ - l, l] , πoυ είν αι δ ύ ο φορές παραγωγίσιμη στο IR , μ ε τη ν ιδιότητα ( ( χ 2 )e g (x,) - ( eg (x , ) ) ' f( χ 2 ) =0 και [ f ( ο) τ + [ r' ( ο) τ 2 + ε. μ ε ε θ ετική σταθερή. g , ,=

Να αποδειχθεί ότι (3Χο ε ΙR)( f( χσ) +( ( χ0 ) 0) =

(e (x ) )

.

ή

τελικά

[ )

' f (χ ) z ' g (e (x , ) f ( x )) = Ο και g = Θεωρούμε λοιΑπό το θεώρημα τη� \1έσης Τιμής του διαφορικού e (x,) 0 . έ χο υ μ ε Λογισμού ( 3 ξ 1 ε (-α, Ο ) ) πόν τις συναρτήσεις, h : [α, β] � IR, h ( χ) = eg(xJf ( χ) Ιl i tΊ ο Ι -"f ( -α Jl Ιr(o) l + f ( -α Jl ι Οι συναρτήσεις και k : [α,β] � ΙR, k(χ) = I

Απόδειξη :

[Ι Ψ

5

=

J

5

}

:�,:;.

αυτές είναι συνεχείς στο διάστημα [α, β] και παρα­

(3ξ 2 ε ( Ο , α ) ) ( / f ' (ξ 2 ) 1 � ι ) , για γωγίσιμες στο (α, β) με h(α)=k(α)=Ο και θετική σταθερή α 2. Θεωρούμε την συνάρτηση h(β)=k(β)=Ο.Αυτό σημαίνει ότι μπορούμε να ε­ φαρμόσουμε θεώρημα του Rolle για τις συναρ­ F : IR � IR, F (χ) = [ f (χ) τ + [ ( (χ) τ . Η συνάρτη­ τήσεις αυτές. τοΈτσι για τις συναρτήσεις h, k αντί­ ση αυτή είναι παραγωγίσιμη στο IR. Άρα η F σαν στοιχα έχουμε : συνεχής στο [ ξ1 , ξ2 ] δέχεται Μ έγιστο έστω στη 3χ1 ε( β) : h' ( χ1 ) =Ομεh' (χ) =eg x ( f' (χ) +g' ( x)f{ χ)) => θέση χ 0 ε [ξ 1 , ξ 2 ], το Μ = F(χ 0 ) Παρατηρούμε => ( ( χ 1 ) = -g' ( χ1 )f { χ 1 ) , F (ξ 1 ) � 2, F (ξ 2 ) � 2 , F( O ) = 2 + ε � Μ => 3χ ε(α,β) : k' (χ ) = 0μεk' (χ) = ( {x) - :;�x)f(x ) => ότι: 2 2 e J F (ξ 1 ) < Μ καιF(ξ 2 ) < Μ => χ 0 ε (ξ 1 , ξ 2 ). => ( ( χ2 ) = g' ( χ 2 ) f{ χ 2 ) . Παραγωγίζουμε την F και έχουμε: Από τις προηγούμενες ισότητες παίρνουμε: F. (χ) = 2f' (χ) [ f (χ) + ( (χ) ] => f ' ( χ ) + g ' ( Χ 1 )g ' ( Χ 2 ) f ( Χ 1 ) f ( Χ 2 ) = 0 . Από => 2f' ( Χ0 ) [ f ( Χ0 ) + ( ( Χ0 ) J = 0 => f ( Χ0 ) + ( ( Χ0 ) = 0, τα( (χ ι ) 2 κατανοούμε ότι ( ( χ 1 ) f' ( χ 2 ) :;e Ο. δεδομένα αφού αν f ' (x 0 ) = 0 => F(x 0 ) = [ f(x 0 ) T � I => Παρατηρούμε ότι: χ1 =χ2 =>2f' {x 1 ) =0=>f' {x1 ) =0, => 2 + ε � 1 => Ο < ε � -1, πράγμα άτοπο, επομένως χ 1 χ 2 . που είναι άτοπο, οπότε ισχύει ( ( χ 0 ) :;e Ο. Τελικά Μεθοδολογικό σ,(όλιο: Το προηγούμενο πρόβλημα μας κατασκευής προβλημάτων αρκεί ως δίνει την δυνατότητα έχουμε (3χ 0 ε iR) ( f(x 0 ) + f" (x 0 ) = 0) . f, g να λαμβάνουμε συγκεκριμένες συναρτήσεις. 4) Έστω συνάρτηση την συνάρτηση S)Θεωρούμε f : [α, β] � IR με f ( α ) = f ( β) = Ο f : ( O ,+oo ) � IR,f ( x ) = l + � ' , με α θετική και( V'x ( α, β) ) ( r ( χ) :;e Ο ) , που είναι συνεχής όμ οια πα ί ρνου μ ε ό τι 2':

.

( )

α,

:;e

(

ε

ΕΥΚΛΕΙΔΗΣ Β' 75 τ.3/6 1

)


------

Μαθη ματικά για την Γ Λυκείου

-------­

σταθερή. Να εξετάσετε αν η γραφική παράστα- ραγωγίσιμες στο 1 -θ, θl συναρτήσεις, ση της f έχει οριζόντια aσύμπτωτη. h : [-θ, θ] � JR, k : [-θ, θ] � JR με h (Ο) = Ο, Κατά τα γνωστά θα εξετάσουμε την ύπαρξη του k ( Ο ) = β, βh ( χ ) - αk ( χ ) = α k ( χ ) + βh ( χ ) = Ο, με α, β σταθερές διάφορες του Ο. Είναι δυνατόν 1i m f (χ) . Παρατηρούμε ότι: με βάση τα δεδομένα αυτά να προσδιοριστούν οι συναρτήσεις h, k ; ( ιη + αχ ) α χ f (χ) = e ' με Δ�, χ ln 1 + --;- = Δ�οc ' Παρατηρούμε ότι τα δεδομένα μπορούν να γρα·

( ι �)

x � +cc

( )

x ln l +-

χ

( �J = lim _!_ = ο

με lim ιη 1 + X � +OC

Χ

] . (ι +�) ι�"χ Η . ι +�) και = lιm

(�)

Η + Ύc

lιm

.

α 1+ α χ Άρα η y = eα είναι οριζόντια aσύμπτωτη (Εδώ ε­ φαρμόστηκε ο κανόνας ι 'Hospital).

-

l i m __ = α => l i m f ( x ) = eα .

X --4- +Y.J

φούν ως εξής: h(O) = Ο k (O) = 1 h( x ) = k (x) k ( x ) = - h(x) β ' β α ' β ' α α ' h ( χ) ' ' οι συναρτησεις οποτε f : [ -θ, θ] � JRμεf( χ) = --, α ( g : [-θ, θ] � JRμεg( χ) = k βχ) , έχουν παρόμοια συμπεριφορά προς τις συναρτήσεις ημ, συν αντί­ στοιχα . Είναι, λοιπόν λογικό να επιδιώξουμε να αποδείξουμε ότι f( χ) = ημχ,g( χ) = συνχ . Για αυσκεφτούμε τό, αλγεβρικά αν ( Α,Β Ε 1RμεΑ 2 + Β 2 = 0 => Α = Β = Ο ) , αρκεί να θεωρήσουμε συνάρτηση την λ : [-θ,θ) � JR,λ(χ) = [f(χ) - ημχ τ + +[g(χ) - συνχΤ με λ(Ο)=Ο με 'την ελπίδα' λ(χ)=Ο, για κάθε στοιχείο του συνόλου [-θ, θ] . Παρατηρούμε ότι: λ (χ) = 2( (x)f(x) + 2g' (x)g(x) = Ο => λ(χ ) = με λ ( Ο ) Ο � c Ο, οπότε: λ( χ) = Ο => f (χ) = ημχ και g (χ) = συνχ, Vx Ε [-θ, θ] . Τελικά έχουμε: h(x) = f (χ) =ημχ =>h(χ ) = αημχ, k (x) = β α = g (χ) = συνχ => k (χ) = βσυνχ .

( ] [ ] '

Χ

Χ �+ ?::

·

X --4- +:f.J

Ν α εξετάσετε αν η εξίσωση χ + 1 + Ιη { χ 2 + 1 } = Ο ,έχει πραγματικές ρίζες και πόσες. ι)

.

Θεωρούμε την συνάρτηση f : JR � JR, f (χ) = χ + 1 + Ι η ( χ 2 + 1), αφού (Vx Ε JR ) ( χ 2 + 1 > Ο ) . Η συνάρτηση αυτή εί­ ναι συνεχής και παραγωγίσιμη στο JR . Παρατηρούμε ότι f' (x) =1+ χ;χ+ 1 =>f' (x) ;:::: OVx EJR-{-1} => => f t ( γνησιως) . Παρατηρούμε επίσης ότι { ) Δ� r (χ) = Δ�, ( χ + 1 + ιη ( χ 2 1 ) ) = Έστω οι παραγωγίσιμες στα πεδία ορισμού f : [α, +οο) JR συναρτήσεις τους }lm. + ιn + ι ) + με c

=

'

}

->? + οο

+

=

και g : [β,+οο ) � JR,μεα, β πραγματικές σταθε­ ρές, ώστε: lim f (χ) = λ Ε JR', lim g ( χ ) = k Ε JR' , . 1 + Ιη ( χ 2 + 1) ( :: ) . [ 1 + 1n ( χ 2 + 1) } lιm = lιm = υπαρχουν τα xll� { ( ( χ ) - f ( χ ) g ' ( χ ) } Χ χ ' = Ο => l i m f ( χ) = -οο , ενώ εύκολα και lim g ( χ ) . Να υπολογιστεί το lim ( ( χ ) . = l im � Χ +1 Χ --4ο-Χ

X --4- -0C·

Χ -->-οο

Χ -->-οο

,

χ---++«>

χ --7 +οο

χ --7 +οο

χ --7 +ο:>

προκύπτει lim f (χ) = +οο . Επειδή η f είναι συνε- Γενικά ,αν για μία παραγωγίσιμη συνάρτηση χής (ΞJρ Ε JR ) ( f (ρ) = Ο ), με τον ρ μοναδικό αφού η h : [ξ,+οο) � JR, μεξ Ε JR έχουμε: l i m h (χ) = μ Ε JR',υπαρχει το l i m h ( x ), f είναι γνησίως αύ ξουσα άρα και 1 -1 . ., Έστω θ, θετική σταθερή. Θεωρούμε τις πα- τοτε l i m h' (χ) = Ο και αυτό διότι χ ---+ +οο

'

X ---t +oc

X ---t +:x:;

X --7 +0C

ΕΥΚΛΕΙΔΗΣ Β' 75 τ.3/62


------ Μαθηματικά για τη ν Γ Λυκείου

------

Rolle για την g ' στο διάστημα ( χ 1 , χ 2 ), έχου­ με: ( 3 ξ Ε ( χ 1 , χ 2 ) ) ( g"( ξ)=Ο). Έστω τώρα ότι το ευθύ ραμμου Μ δεν εί ν αι μέσο τμήμα­ σημείο του γ να θεω­σε πτωσηΜαυτή τοςρήσουμε Στην διαιρείμπορούμε ότι τοπερίσημείο τοx -ΚΛ αέστω λόγο λ :;t: -1 'ώστε λ= βΧο--αχ ο = g(g(β)o )- g(g(xo )) => => g( Χοχ 0) -α- g( α ) = g(β)β -- g(Χ ο χ ο ) . Εδώ εφαρμόζουμε εκκούνέουΛοτογισθεώρημα φορι μού: της Μέσης Τιμής του Δια­ ( 3 χ Ε ( α, Χ ο )) [ g' ( Χ ' ) = g( xΧ0 ο) -α- g( α ) ) ' ( - Ε ( Χ0 , β ) ) [ g ' ( Χ .-0 ) - g(β) -- g( x o ) ) , β Χο θεωρούμε συν άρτη ση έτσι παίρνουμε (χ;)= (χ�), δηλαδή g' g' g : [α 1 ,β 1 J � { O ,+co ) με α 1 < β1 , που είν α ι συνεχής στο διάστημα [ α 1 , β1 J ,παραγωγίσ ιμη στο ( 3 ξ Ε (χ ;, χ � ))( g"( ξ)=Ο).Σε κάθε λοιπόν περί­ ( α 1 ,β 1 ) , ώστε ( ( α 1 , β 1 ) )( g ' ( χ ) Ο ) , με τη ν πτωση (3 ξ Ε ( α , β ) ) {g"( ξ)=Ο). Υπολογίζουμε τώ­ g' να είναι πα ραγωγίσιμη στο διάστημα ρα την δεύτερη παράγωγο τηςχ f : χ - χ g ' συνεχή στο πεδίο ορισμού ( (χ) = g' ( χ ) => ( ( χ ) = g" ( ) g ( 2) [ g' ( )τ ;::: Ο, ( α 1 , β 1 ) και με χ) χ) της. g ( g ( ωρ ε α ε α α < α<β< ούμε < β <β<β Θ ,βμε 1 1 α, β μ ιlι 1 με g' ( χ ) :;t: Ο. και { χ 0 ε (α , β ) }{ g " ( χ ο ) g(χ ο ) � [ g' (χ ο ) Τ ) · Αν g" ( Χ0 ) g ( Χ0 ) = [g' ( Χ0 )τ => ( ( Χ0 ) = 0, ενώ αν Αν τα σημεία κ ( α, g ( α )) , M{x 0 ,g(x 0 )), g " ( Χ0 ) g ( Χ0 ) > [ g ' ( Χ0 ) ] 2 , για Λ ( β, g ( β )} , είν αι σημεία της ίδιας ευθείας, να αποδειχθεί ότι η δεύτερη παράγωγος της συ­ χ = ξ => f " ( ξ ) = [ g ' ( ξ) τ < Ο,μεf "( χ 0 ) > Ο.Απο g- ( χ ) νάρτησης f : [ α, β ] � JR,f ( χ ) = ln g( χ ) έχει μία Bolzano για την δεύτερη παράγω­ του τογο θεώρημα τουλάχιστον ρίζα. έχουμε 2 ύπαρξη της ρίζας ρ της παρα­ την f ης στο διάστημα που ορίζουν οι αριθμοί ξ, χ 0 . γώγουΈστω σημείο Μ είναι μέσο ευθ. Έστω ότι το του τμήμα­ συνάρτηση f : JR � JR, δύο φορές πα­ τοςxΚΛ-έχουμε: ραγωγίσιμη JR . Θεωρούμε ότι το σημείο x α g( o ) g( ) = g(β) - g( o ) ( * ). Αν εφαρμόσου- { 7, f 7 )) είναιστο σημείο καμπής της γραφικής { χ 0 -α β - Χ ο παράστασης της f .Θεωρούμε επίσης την συ­ μερικτώρα θεώρημα το της Μέσης τιμής του Διαφο­ Λογισμού για την g στα αντίστοιχα διαστή­ νάρτηση g : JR � JR, g( χ ) = r ( χ 2 + 3 ) . Αποδείξτε ματαούπαίρνουμε: ότι g' (2 ) = 2 g " { 2 ) . α ) ( g( [ (3 χ 1 Ε ( α,χ 0 ) ) g'( x 1 ) = χ:; =: } g'( x ) =2xf' (x 2 +3), g"( x ) =2f'(x 2 +3)+ x ) g( g(β) ( ) o ( 3 χ 2 Ε ( χ ο , β ) ) g (χ 2 ) = β -χ ο ' +4χ 2 ( (χ 2 + 3) => g" ( 2 ) = 2( ( 7 ) . . . . δύο κάθετες ημιευθείες Ο χ , Oy Επί άρα g' ( χ 1 ) = g' ( χ 2 ) οπότε από το θεώρημα του της Έστω ημιευθείας Oy παίρνουμε τα σημεία μ = lim h ( χ ) = lim e' he'( x ) (�)= lim ( e'h( e')( x, ) )' = = Διr, ( h. ( χ ) + h ( χ ) ), Επομένως Δ�( h ( χ ) + h. ( χ))= μ => Διr, h. ( χ ) =0. Αν σκεφτούμε λίγο θα δούμε ότι ισχύει: ' l ι. m [ fe�(χ) )' = => lι. m -,fe-(�,(χ-)) = eλ = Ε JR* . Άρα lι. m f' ( x ) -feg((xx) )g' ( x ) = Ο με lι. m e- =ek => Δ� ( ( ( χ ) -f ( x )g' ( χ) ) = Ο με Διrχ g . (χ) = αφού lim g( x ) = k EJR* . lim f ( x )g' ( x ) =ek ·0=0::::> lim f(x) =O. χ ->+"'

Χ

X ---t +x

χ --> +οο

k

e

X ---t +7')

, __, _

Χ � +'Χ

Χ ---t +'Χ:

·) --;;(Χ

ΚΛ.

ο

e( x ) ·

ο,

χ --; .μχ:

X ---t +:YJ

\ ---t - :ι:

Ι

·

Ι

::J • ::J X 0

τη ν

V'x Ε

:;t

τη ν

3

.

ο

" Qι )

·

J

;

ΕΥΚΛΕΙΔΗΣ Β' 75 τ.3/63


------

Μαθηματικά για την Γ Λυκείου

--------­

Β, Α : Ο < β = ΟΒ < ΟΑ = α. Θεωρούμε ότι επί δική ρίζα την Ρ = I η -2 στην οποια' γινεται ' αλλαγη' α της Ο χ κινείται σημείο Μ . α Να προσδιοριστεί η θέση του Μ για την οποία του πρόσημου της 2ης παραγώγου ,οπότε το σημείο η �ΑΜΒ να είναι η μέγιστη. ' Λ \1 ση :

Θεωρούμε ω = �ΑΜΒ,φ = �ΒΜΟ, χ = ΟΜ. Παρατηρούμε ότι: α-β εφφ φ) + εφ(ω = εφω = 1 + εφ(ω + φ)εφφ = χαβ - (αχ 2-+β)χ αβ f(x). l+ χΑρκεί, λοιπόν να προσδιορίσουμε το Μ έγιστο (maximum) της f(x), που όπως παρατηρούμε είναι ορισμένη στο σύνολο των πραγματικών αριθμών και είναι παραγωγίσιμη ως πηλίκο παραγωγίσιμων (πολυωνυμικών) συναρτήσεων. Έτσι παίρνουμε: (α - β) ( χ 2 + α β ) - 2 (α - β) χ 2 f . (χ) = ( χ 2 + αβ) ( α - β) ( αβ - χ 2 ) ( χ 2 + αβ ) 2 με την πρώτη παράγωγο να μηδενίζεται όταν χ = -jc;β η χ = jc;β :::::> χ = jc;β, αφου χ > Ο. Προφανώς η γωνία ω γίνεται μέγιστη όταν χ = jc;β, δηλαδη χ 2 = α β. Σz6 λ ω : Ο προσδιορισμός του Μ ,ώστε χ 2 = αβ <:::> ΟΜ 2 = ΟΑ · ΟΒ( * ), είναι απόλυτα α­ κριβής, δηλαδή με κανόνα και διαβήτη και αυτό επειδή η σχέση (*) οδηγεί στην κατασκευή κύκλου που να διέρχεται από τα σημεία Α, Β και να εφά­ πτεται της Οχ σε σημείο που προφανώς είναι η θέ­ ση του Μ την οποία ζητάμε (δύναμη σημείου ως προς κύκλο με την ΟΜ εφαπτομένη του). Αυτή εί­ ναι μία Ιστορικής σημασίας κατασκευή καθότι εί­ ναι μία από τις κατασκευές του Απολλώνι (έζησε στην Πέργη της Παμφυλίας περί το 262-1 90 π.χ. ). J

7

( � , 2 - ( �) J = ( ρ" , 2 - ρ"' ) είναι ln

ln

σημείο καμπής. Διαπιστώνουμε τώρα εύκολα ότι τα ση­ μεία αυτά ανήκουν στην παραβολή y = -χ 2 + 2.

συνάρτηση Θεωρούμε την f : [ο, π ] � JR,f ( x ) = �ημ χ + �ημ χ + �ημ χ + .. . , με άπειρα ριζικά. Να υπολογιστεί η παράγωγος της. 1 3)

Λ \1 ση :

f(χ) = �ημχ + �ημχ + �ημχ + ... :::::> f(x) = = Jημχ + f( χ) :::::> f 2 (χ) = ημχ + f( χ), Άρα έχουμε: ( (χ) -f( χ) =ημχ :::::> 2f( x)f' (χ) -f' (χ) = ' ορισμου, της = συνχ :::::> f. ( χ ) = 2fσυνχ , με πε ιο δ (χ) - 1 ( επίσης το [0, π], αφού 2f( χ) * I, για κάθε χ ε [Ο, π]. Αυτό προκύπτει από τον εξής συλλογισμό: 1 θα είχαμε Αν υπήρχε φ ε [Ο, π] με f ( φ) = -, 2 1- - -1 = ημφ --Ι = ημφ ημφ < Ο που είναι ά4 4 2 τοπο αφού: φ ε [Ο, π] :::::> ημφ ::::=: Ο. :::::>

:::::>

1 4) Α) Να αποδειχθεί ότι:

Β) Έστω η

e ' :?: χ + l, x ε JR .

συνάρτηση: + '+ f : JR � JR , f ( χ) = ln 2 "' 1 (e ' t + 2συν 2 '· , +Ι ( ημ ιν_,+ι χ + χ 2 '·, + 1 ) ) , ν ι ε Ν. α ) Να προσδιοριστεί το πεδίο ορισμού της . β) Να προσδιοριστεί η εξίσωση της εφαπτομέ­ νης της γραφικής παράστασης της f στο σημείο 1 2 ) Θεωρούμε την οικογένεια των συναρτήσεων της {Ο, f (Ο) ) . fu : JR � JR , fα (χ ) = αe ' - χ 2 ,α > ο. Λ\Jση Ν α αποδειχθεί ότι για κάθε συνάρτηση από αυ­ Α) Άσκηση 8, σελίδα 292 του βιβλίου των Μαθη­ τές υπάρχει ένα ακριβώς σημείο καμπής και ότι ματικών της κατεύθυνσης. ' όλα αυτά τα σημεία καμπής είναι σημεία συ­ Β) α) Από την σχέση: e > χ + Ι, χ ε JR : προκύ­ γκεκριμένης γραμμής. πτουν τα ακόλουθα:

Απ6ιiΕιξη : χ' +Ι ν0+ Ι ( ν3 + Ι ν +Ι Παρατηρούμε ότι η παράγωγος της fα ,για συγκε­ e '> Χ 2 + 2 > -2συν 2 ημ 2 Χ + Χ 2 , ) :::::> κριμένο α, ορίζεται στο σύνολο των πραγματικών :::::> e χ +Ι + συν 2 ν, + Ι ( ημ 2 ν; +Ι χ + χ 2 ν, + Ι ) > Ο . έχουμε Επομένως το πεδίο ορισμού της f είναι το JR, με οπότε αριθμών χ f� JR JR , f� (χ) = αe - 2χ. 'Ο μοια έχουμε την την f να είναι παραγωγίσιμη στο JR ύπαρ ξη της ( : JR � JR , ( (χ) = αe - 2, με μονα- σαν σύνθεση παραγωγίσιμων στο JR συναρτήσεων και πράξεων μεταξύ αυτών. :

'

ΕΥΚΛΕΙΔΗΣ Β' 75 τ.3/64


------μόνο μία παράγωγος ( ( χ ) .Αντίθετα σε μία πα­ ράγωγο ( ( χ ) aντιστοιχίζονται άπειρες αρχικές συναρτήσεις f(x) οι οποίες διαφέρουν μεταξύ τους κατά σταθερά. Το να μας ζητήσουν να προσ­ διορίσουμε την αρχική μίας συνάρτησης ή την παράγουσα ή το αόριστο ολοκλήρωμά της ,το πρόβλημα αυτό γίνεται κάτω από το σύμβολο f. Μ ετά το σύμβολο f ακολουθεί το διαφορικό της ζητούμενης συνάρτησης που με τύπους και ιδιότητες θα aποκρυπτογραφηθεί για να μας την προσδιορίσει. Άμεση συνέπεια είναι η: Jf' ( χ ) · x = f ( χ ) + c. Εδώ υπενθυμίζουμε ότι αν η συνάρτηση f είναι ορι­

------- Μαθηματικά για την Γ Λυκείου

( O ) = Ιη 2 ν , + Ι ( e + 2) .

Θα β) Παρατηρούμε ότι: f προσδιορίσουμε τώρα την παράγωγο f' ( Ο) , που όπως είπαμε υπάρχει. Εδώ μπορούμε να διαπιστώ­ σουμε ότι: ( Vx IR ){ f ( -χ) = f ( χ )) ( αρτια ) . 'Ετσι παίρνουμε: f ( h + O) - f (O) f ( h ) - f ( O) = ' f ( O ) = lim = l im h h -u ) - f ( O ) f f ( u ) - f ( O) ( = lim = - l im = -f ' ( O ) , Ε

h ->0 '

u ->0-

h ->0 '

u ->0-

-u

u

από όπου άμεσα προκύπτει ότι ( (Ο) = Ο. Άρα η ε­ ξίσωση είναι y = \η 2 ν , + ι ( e + 2) . Π ροη:ινό μηα θέματ α :

I ) Θεωρούμε την παραγωγίσιμη στο IR συνάρ­ τηση f : IR � IR για την οπ ο ία ισχύει ( '11 x E IR )(r 2 ( x ) + x 2 ( r ' ( x ) ) 2 + x ( f 2 (x ))' � ι). Να αποδειχθεί ότι η f είναι φ ρ αγ μ ένη . \ π6δ::: ιξη :

d

σμένη και συνεχής σε διάστημα Δ ,τότε κάθε χ συνάρτηση F ( χ = f t ) dt, χ ε Δ, με α στα-

) J( α

θερά που ανήκει στο Δ ,είναι συνεχής στο Δ και είναι αρχική συν ά ρτηση ή παρ άγουσα ή αόριστο ο λοκλή ρωμα της f(χ).Δηλαδή

σχέση που είνα ι σε ισχύ δίνει ' - Ι � ( xf ( χ )) � Ι, στην συνέχεια χρησιμοποιώντας την σχέση μεταξύ του πρόσημου της παραγώγου ισχύει: F ( χ ) f ( t) dt f ( χ ) . με την μονοτονία συνάρτησης έχουμε: Υπενθυμίζουμε επίσης ότι το ο ρ ι σ μ έ ν ο oλo­ l f ( 0)1 :::; \και ( Ε �· )(l f ( χ )I :::; Ι ) � ( Vx E IR)(I f ( χ )I :::; Ι ) . κλήρωμα Jf ( χ ( στα&ερα ακρα ) , είναι σταΗ

Ο ) )dx θερή οπότε [fr(x)dx) � ο . Θα ήθελα ακόμα να αναφέρω ότι για να προσδιορίσουμε το πεδίο ορι­ σμού της συνάρτησης ) f f ( t )dt , κάνουμε �

'lix

fJ

2 η Ν α μελετηθεί η συνάρτηση f ( χ) = � . Ιη χ

Υπι'ιδt:ιξη :

Θα πρέπει να ασχοληθούμε με τα εξής: α) εύρεση του πεδίου ορισμού, β) περιοδικότητα, γ) συμμετρίες (άρτια ή περιττή), δ) συνέχεια, ε) aσύμπτωτες , στ) μέγιστα - ελάχιστα - σημεία καμπής ,ζ) μονοτονία ,η) κυρτότητα ,θ) σημεία το­ μής με τους άξονες ι) πίνακας καταγραφής και γραφική παράσταση. Β - Ο ι\ Ο Κ Λ Η Ρ Ω Μ Λ 'Ι' Λ Θεωρούμε γνωστό το πρόβλημα της Π α ρ α γ ώ γ ι σ η ς μιας συνάρτησης f(x) . Υπενθυμίζουμε επίσης ότι το δ ι α φ ο ρ ι κ ό μίας συνάρτησης f(x) συμβολίζεται df(x) και ισούται με την παράγωγο της συνάρτη­ σης επί dx δηλ. f ( χ ) = ( ( χ ) dx, με dx aπειροστή μεταβολή του χ.Τώρα μας ενδιαφέ­ ρει το α ν τ ί σ τ ρ ο φ ο πρόβλημα, δηλαδή δίνεται η παράγωγος ( ( χ ) συνάρτησης f(x) και ζητάμε την f(χ).Τότε η συνάρτηση f(x) ονομάζεται αρχική ή παράγουσα (αφού παράγει την παράγωγο) ή αόριστο ολοκλήρωμα. Εδώ πρέπει να επισημά­ νουμε ότι σε μία συνάρτηση f(x) aντιστοιχίζεται

d

α

F(χ =

g( χ ) Ιι ( χ

)

τις εξής κινήσεις : l η) βρίσκουμε τα πεδία ορισμού A Γ , A h , Ag των συναρτήσεων f,h,g αντίστοιχα, 2η) βρίσκουμε την τομή Α = Α 1, Ag και προσδιορίζουμε εκείνα τα χ Α ώστε τα άκρα h(x) και g(x) να ανήκουν στο πεδίο ορισμού ΑΓ της f , 3η) Το πεδίο ορισμού A F της F(x) αποτελείται από εκείνα τα στοιχεία που προσδιορίσαμε στην 2η κί­ νηση και για τα οποία η f είναι συνεχής στο κλει­ στό διάστημα που ορίζουν τα άκρα h(x), g(x). Ε

Π uρ ά δ r. ιγμu:

dt

n

Για το πεδίο ορισμού της συνάρτησης F ( χ) = f �· έχουμε: Το πεδίο ορισμού της νt Γχ+ι χ'-ι

ΕΥΚΛΕΙΔΗΣ Β ' 75 τ.3/65

1


------ Μαθηματ ικά για την Γ ' Λυκείου

Ι, έχουμε: Ιίm(χ - Ι) = καβ � κ = -αβ Ιίm(χ - Ι) = λα(α - β) � λ = α (αα--Ιβ ) ' Ιίm(χ -Ι) =μαβ � μ β β-α Εύκολα διαπιστώνε) ται ότι επαληθεύουν την (*) . Επομένως έχουμε { Ι + α- Ι + β - 1 Jl- αβχ α(α - β)(χ -α) β(β - α)(χ -β) χ = β - 1 J�χ = χ + - αβΙ J-;1 ctχ + α(αα --1β) J� χ - α β(β - α) χ - β 1 Ιn l xl + α - Ι Ιn lx - α l + -β β - α ) Ιn lx - b l + c. αβ α(α - β)

Ι ,ειναι το A r = (Ι,+οο ), της f( t) = �

νt - Ι h (χ) = χ 2 - 1 το Ah = IR. και της g( χ) = Γχ+ι τοΑ g = [ -Ι,+οο ) . Επομένως Α = Ah A g = [-Ι, +οο) .Για να ορίζεται η F πρέ­ πει και αρκεί 2 J2 Γχ+ι > Ι και χ - Ι > Ι � χ > Α. Συνεπώς το πεδίο ορισμού της F είναι το Α = ( J2 , +οο).

χ �ο

r- I

χ�α

n

·�

ε

'

}

Ι=

F

" Να υπολογιστεί το ολοκλήρωμα: r '' ( x ) f 2 ( x ) x . I= f ( χ) ( r' ( x))3

s[� -

------

}

r-\

_, ;

Να αποδειχθεί ότι 2 ν J d x ) = χ ) + (2 ν - ι ) J d x ) (χ + 1 (χ + 1 (χ + 1 για κάθε φυσικό αριθμό ν. 2

ν+ Ι

2

ν

2

ν '

Παρατηρούμε ότι το ολοκλήρωμα του 1 μέλους με το ολοκλήρωμα του 2"" μέλους αναφέρονται σε συναρτήσεις που η μορφή τους μας επιτρέπει να συμβολίσουμε f dx οποτε Ι ν+1 = dx Αρκει. λοιΙν = J( χ 2 + 1Γ ( χ 2 + 1) ' πόν να αποδείξουμε ότι χ + ( 2ν - Ι ) Ι ν . 2ν l ν + Ι = ν ( χ2 + 1 ) Σε τέτοιες περιπτώσεις ,συνήθως χρησιμοποιούμε την μέθοδο της ολοκλήρωσης κατά μέρη (ή κατά παράγοντες),οπότε παίρνουμε l ν = fx' χ = χ - Jx *) . d ν ( χ- +I ) dχ ( ( χ2 +Ι γ ( χ 2 + Ι γ ου

1 •

Θεωρούμε ότι το ολοκλήρωμα αυτό είναι σημα­ ντικό από την άποψη ότι μας δίνει δυνατότητα κα­ τασκευής προβλημάτων στα ολοκληρώματα, το­ ποθετώντας στην θέση της f(x) μία συγκεκριμένη συνάρτηση.

I

Να

[I] ,

'

υπολογιστεί το ολοκλήρωμα: 1 - - 2νχ I = J χ 3 α +χ β- 1χ 2 + αβχ d χ ,οταν α, β ε !R.* μεα -::;:. β Ό μως έχουμε: ... Ι (πα -( ) ( χ + 1) + ( χ 2 + Ι )'

[ ]

2

ν

ράγωγος πηλίκου ).Από την σχέση (*) παίρνουμε: ότι Παρατηρούμε 2 χ 3 - (α + β) χ 2 + αβχ = χ (χ - α) (χ - β), επομένως I ν = 2 χ ' + 2ν J 2 χ • dx = (χ + ΙΓ (χ + 1) αριθμούς πραγματικούς ζητούμε μ . λ + --η -Ι χ + 2ν J χ 2 + I - I dx = -κ + -= κ ' λ' μ ' χ(χ -χα)(χ - β) χ χ - α χ - β ( χ 2 + Ι Γι ( χ 2 + 1) ' χ - Ι = κ (χ - α)( χ - β) + λχ (χ - β) + μχ (χ - α)( * ) , Ι dx = χ + 2ν J 1 dx - 2ν J για κάθε χ � {Ο, α, β} , οπότε έχουμε: ( χ 2 + ΙΓ • ( χ 2 + Ι )' ( χ 2 + 1) ' ΕΥΚΛΕΙΔΗΣ

Β'

75 τ.3/66


----χ (χ 2 +1 ) + 2νlν -2νl + .

--ν

ν

Μαθηματικά για την Γ Λυκείου

επίσης ότι ισχύει: ( 'ν' χ Ε JR ) ( f " ( Χ ) = ln ( Χ + 1 ) ) . Αν υποθέσουμε ότι η γραφική παράσταση της f περνά από το σημείο (0,1) και στο σημείο αυτό εφάπτεται γραφικής παράστασης της 2 :g JR � JR , g(της ) = χ χ α, όπου α πραγματική σταθερή, να υπολογιστεί το ολοκλήρωμα:

ι

Από εδώ προκύπτει εύκολα:

ι

I = J(r( x)-g (x))dx .

Ι! ;

e'

-------­

ο

παρουσία του σαν παράγοντα σε γινόμενο μας κάνει να σκεφτούμε το πλεονέκτημα που έχει, ανά πάσα στιγμή να συμπεριφέρεται και σαν πα­ Θα προσδιορίσουμε τους τύπους των και στην ράγωγος του εαυτού του καθότι ισχύει συνέχεια το Ι. Από τα δεδομένα παρατηρούμε ότι: . Παρατηρούοπότε έχουμε την δυνατότητα να θέ­ (Ο) 1 α οποτε τον τουμε σε ισχύ ισότητες του τύπου: με επίσης ότι: προσδιορισμό του τύπου της έχουμε: (i), Η

f, g

= -1 g ( χ ) = χ 2 + 1 g'( χ ) =2χ � g' ( 0 ) =0 ( 1 ) . Για f η χ ) = J(( x )dx = JΙη ( χ+ 1 ) dx = J( χ+ ι )Ίη ( χ+ Ι ) dχ = ( χ + 1 ) Ιη ( χ + 1 ) - Jdx = = ( χ + 1 ) Ιη ( χ + 1 ) -χ + f'(O) = g' ( 0 ) (=0ι J � c ι = Ο � ( ( χ ) = ( χ+ 1 ) !η ( χ+ Ι ) -χ . Συνεχίζοντας έχουμε: f ( x ) = Jf' ( x ) dx = J( χ+Ι ) Ιη ( χ+Ι)dχ- Jxdx = = J�[( x+1/]ιη ( x+l ) dx- �x 2 = και έτσι μπορούμε να προσανατολιζόμαστε σε = _!_ ( χ+ 1 ) 2 Ιη ( χ+ Ι ) _!_ J( χ+ 1 ) 2 �χ _!_ χ 2 = 2 2 2 χ+1 μορφές αρχικών συναρτήσεων Υπολογίστε το ολοκλήρωμα: _!_2 ( χ + 1 )2 Ιη ( χ + 1 ) - _!_2 J( χ + I )dx 1 J ( 1) 3χ 2 χ ' με --χ21 - =-21 ( x+l )2 Ιη ( x+l ) ----+c 4 2 Λαμβάνοντας υπ' όψη το μεθοδολογικό σχόλιο 1, f (Ο) = 1 � c = I ( αφού η γραφική παράσταση της f έχουμε: περνά από το σημείο (0, 1) ) . Τελικά παίρνουμε: 2 χ I e' ( χ 2 + χ")+ 1) = e' (χ 2 + 2χ +') 1 -χ) = 3χ 2 χ+ 1 χ =χ+ 1 Ι η ----+ f ( ) ( ) ) ( 2 4 2 2 ( χ+1 )( χ+1 )I 7χ χ 2 � χ =χ χ+ Ι Ι η χ + 1) - - - -. f ( ) ( ) g ( ) ( e' [ + I ) 2 -χ] e' ( χ+ 1 )( χ+ 1 ) -xe' (χ 2 4 2 - ( χ+1 ) 2 - ( χ+1 )2 ι 1 = 1=-21 0J(x +lγ lη ( x +1 ) dx --47 0Jx -? dx --Jxdx 20 (= e'x),( x+1 ) - ( e'x) ( x+1 ), = [ xe' )' � x+l ( χ+1 ) 2 = _!_6 �LfΓ( χ+Ι γ ] Ιη ( χ+Ι ) dχ - 2[ χ 3Τ _!_4 [χ 2 Τ = 12 f(-xe' )' dx =--+c. xe' =-61 [( χ+ I )3 !η ( χ + 1 )J ι --61 j ( χ+ I γ dx- -6 = χ+1 x+l Θεωρούμε συνάρτηση f : JR+ JR που είναι = 4 1η 2 _!_ [ ( χ+1 )3 ] ι - � = . . = 12Ιη 2-Ι1 _ 6 3 18 δύο φορές παραγωγίσιμη στο JR+ . Θεωρούμε ( e' )' = e', e'f ( x ) + e' ( ( χ ) = ( e'f( χ))' (" " ) e χ f( χ ),-(( χ )(χ) = ι�J· r(χ) f' (x) -f ( x ) = e'f'( x ) -e' f( x ) = [ f (x) )' ( iii ) ' e·' e2 ' e' x ) - f (x)g'( x ) = e' ( f( x ) + f' ( x ))g( g" (x ) 'f( x )g' ( x ) - ( e'f ( x ) )' ( . ) = (e'f( x )), g(x)-e ( g χ) g(χ)

g =

11 ,

r-

Cι .

Από τα δεδομένα του προβλή ματος κατα-

νοούμε ότι:

ιν ,

2

-� ,

e

χ

χ

2

+χ+

χ+

z

d

χ

_

.

_

ο

1

_

ι

?

ι

,

ι

ο

ι

ο

=

r

?

5

ο

_

ο

ΕΥΚΛΕΙΔΗΣ Β ' 75 τ.3/67

ο

_

9


----6)«Υπάρχει συνάρτηση f : [ α, β ] � JR,

Μαθηματικά για την Γ Λυκείου

με α < β,συνεχης στο διαστημα [ α, β ] και { V' χ ε [ α, β]) { f ( χ) > Ο ) , με

Jr ( x ) d x = κ 2 , Jxr ( x ) d x = κλ, Jx 2 f ( x ) d x = λ2 , όταν κ, λ μη μηδενικές σταθερές Να χαρακτηρίσετε τον παραπάνω συλλογισμό χρησιμοποιώντας έναν από τους χαρακτηρι­ σμούς ΣΩΣΤΟ ή ΛΑΘΟΣ, αιτιολογώντας τα ε­ πιχειρήματα σας. β

β

β

α

α

α

».

Λ ίJ ση :

Έστω ότι υπάρχει μία τέτοια συνάρτηση f. Παρα­ τηρούμε ότι: κ 2 Jx 2 f( χ )dx + λ2 jf( χ )dx - 2κλ Jxf( χ )dx = κ 2 λ2 + κ 2 λ2 - 2κ 2 λ2 = Ο => J( κ2 χ2 + A_l -2κλχ )f( x }Jx =0=> j( -λ) 2 f( x }Jx =0. Θετουμε g( χ) = ( -λ) 2 f( χ ) . Έχουμε : g( χ) � Ο, 'i:i x ε [α, β]. Τότε προκύπτει ό­ τι: Jg( χ )dx = Ο, με g( χ) � Ο, με την g(x) συνεχή στο διάστημα [α, β].Από γνωστό θεώρημα αυτό συμβαίνει μόνο όταν g (χ) = Ο, για κάθε χ ε [α, β], πράγμα άτοπο αφού η εξίσωση g(x)=O έχει ως μοναδική ρίζα την χ = .?:.κ . Τελικά ο συλλογισμός χαρακτηρίζεται ως ΛΑΘΟΣ. β

β

β

α

α

α

β

β

α

α

κχ

κχ

β

α

------

β

β

α

α

jf( χ )dx > Jg( χ )dχ(αυστηρα μεγαλυτερο).

7)

I

Να αποδειχθεί ότι 3 J 1 3 d x ν χ - 3χ + 4 <2. ο

Απόδειξη : Η

συνάρτηση h (χ) = χ3 - 3χ + 4, είναι συνεχής στο διάστημα [0, 1 ], με h(0)=4>0, h( 1 )=2>0 οπότε O<h( l )< h(O) και h ' (x) = 3(x - l )(x + 1) < 0, οταν χ ε (Ο, 1), οπότε η h είναι γνησίως φθίνουσα στο [0, 1 ] που σημαίνει ,τελικά ότι h(x)>O , για κάθε στοιχείο χ του διαστήματος [0, 1].Παρατηρούμε ό( vx ε [0,1]) ( 4 - 3χ + χ3 � 4 - 3χ > ο ) => τι: � => ( vx ε [0,1]) 1 1 ν 4 - 3χ + χ3 ν 4 - 3χ με την ύπαρξη χ 0 ε (Ο,1)(π.χ. χ 0 = .!_2 ), με4 - 3χ 0 + χ� > 4 - 3χ 0 ( αυστηραμεγαλυτερο ). Μ ε ολοκλήρωση λοιπόν παίρνουμε: 1 dx 1 dx ι dx Jo .Jx3 - 3x + 4 < 0J�4 - 3χ ' με 0J�4 - 3χ = = - 3.3 [�4 - 3χ ] ι = 3.3 => Jι dx < 3.3 ° o .Jx3 - 3x + 4 'Η 3 J dx < 2. o .Jx3 - 3x + 4

{

(

�J

-;=====

8)

e·' Να αποδειχθεί ότι 3 J d x < 2eJ . .π.

e .

ln χ

Απόδειξ :

Το γνωστό θεώρημα είναι το εξής: Έστω g συ­ Θέλουμεηνα αποδείξουμε ότι: νεχής συνάρτηση σε διάστημα � 2 3 = -2e 3 . d x < 2 e 3 η, e3J d x < -e (α, β]. Αν {'tχ ε [α,β])(g(χ ) � ο) και η συνάρ­ 3 f.π. Ιη χ 3 Ιη e3 .π. Ιη χ e e τηση g δεν είναι παντού μηδέν στο διάστημα β Εδώ διαπιστώνουμε ότι το e 3 εμφανίζεται σε τρία αυτό ,τότε Jg( χ ) d x > Ο. σημεία της προς απόδειξη ανισότητας, οπότε το ·

·

Λ ναφομά

α

σε ανισ6τ η τες και ολο κλη ριJψατα :

α ) Υπενθυμίζουμε ότι η ολοκλήρωση (κατά Rie­ mann σε διάστημα [α, β]) των μελών μίας ανισό­ τητας δεν αλλάζει την διάταξη της ανισότητας. β) Το θεώρημα που αναφέραμε στο προηγούμενο πρόβλημα οδηγεί στην εξής διαπίστωση: Αν έχου­ με τις συναρτήσεις f : [α, β ] � JR,g : [α, β] � JR(α < β) : ('i:ix ε [ α, β]) ( f(x) � g(x)) και ( ΞJ χ 0 ε [α,β]) ( f(χ 0 ) * g(χ 0 )) , τότε στην περίπτωση αυτή ισχύει :

γεγονός αυτό σε συνδυασμό με την μορφή της μας οδηγεί στο να θεωρήσουμε την συνάρτηση: ln x - 2 . ' = => f : JR : � JR,f(x) = ln2χx ji.J � f (x) ιη-, χ ln t Άρα: χ > eJ5 > e 2 ( 5 > 4 => J5 > 2 => eJ5 > e 2 ) => ln χ - 2 > Ι e 2 - 2 = Ο, που σημαίνει ότι η f θα είναι γνήσια αύξουσα στο διάστημα [ eJ5 ,+oo ) , με f ( eFs ) = 2Js > 0 =>

ΕΥΚΛΕΙΔΗΣ Β' 75 τ.3/68

_

e .

η

J5

eJ\ <e1


-------

Μαθηματικά για την Γ Λυκείου

------

dt 2e => Από τις (Ι),(2) => 0 < Jr4 (x)dx s; (α - β)μM . 2e - J dt O < f (e ) < f (e ) = 3 .;; ln t => J Ιη t < 3 ι 0 ) Έστω οι θετικοί πραγματικοί αριθμοί x ,y με dx 3 J ln χ < 2e3. y<x . Να αποδειχθεί ότι: e:y > e; J5

3

3

c

c'

c

3

β

J5

α

( ) ( )Υ x

J5 e .

'

x Θεωρούμε την μη σταθερή συνάρτηση Απόδηξη : f : [ α, β] � IR ( α < β), με f ([ α, β]) ς ( -1, 1 ) που αρκεί είναι συνεχής στο Ι α, βl και για την οποία ισχύει Αρκεί να αποδείξουμε ότι �χ e 2 > e 2 Lή Χ) Υ β Jr ( x )dx = O ( * ) . Aν η f έχει σαν ελάχιστη τιμή -χ > e - ( Υ) η' αρκει' ln-χ > 2-χ-y, Υ x+y την μ και σαν μέγιστη τιμή την Μ , να αποδει­ Υ Ρ ή τελικά ,αν θέσουμε t = � <:::::> χ = ty ( t > Ι ) , αρ­ χθεί ότι: Ο < Jf4 ( χ ) d x s; ( α - β) μΜ. Υ 9)

χ

( )x +y

α

ν

Y

? χ-

α

t-I ότι: ln t 2 -- ή αρκεί κεί να αποδείξουμε Αφού η συνάρτηση f είναι συνεχής στο διάστημα t+I [α, β] θα είναι συνεχής συνάρτηση και η 1 f" ,με ν ε Ν = {2,3, . . } , στο ίδιο διάστημα. Γνωρί­ 2 l t l t +2 1 ή αρκεί -2Ι [Ιη u Τ - 2+ Ι ή ζουμε επίσης ότι η f δέχεται ελάχιστο (minimum) , 1 1Jdu 2 1J du ( η, τελικά αρκεί να μ και μέγιστο (maximum) Μ που δεν ταυτίζονται αρκεικαθότι η f δεν είναι σταθερή. 2 (u + 1) 2 ) Αν μ Οτοτεf( χ) Ο, μ Ο => f( χ) Ο, οπότε από ' -1 ;::; 2 ' υ με οτι: γνωστό θεώρημα και την σχέση (*) παίρνου ­ αποδειξο 2 u ( u + 1) 2 με: {Vχ ε [ α,β]){f(χ) = Ο ) , πράγμα άτοπο αφού η f είναι μη σταθερή συνάρτηση, άρα μ<Ο. Με ίδιο που είναι αληθής πρόταση όταν u [ 1, t] , με t 1 . σκεπτικό καταλήγουμε στο συ μπέρασμα Η σχέση (*) ισχύει καθότι για το τυχόν [ β] Μ >Ο. Ά ραυπάρχουν 2 ( 1 , t) Ι 2χ 0 ( χ 0 + 1) 2 • ( \f χ ε [α, β]) ( - 1 < μ = f (χ ) ::::; f (χ) ::::; f (χ 2 ) = Μ < 1 ) Έτσι έχου με: Μ εΟοδολογ ι κό σχόλιο2 : Θα πρέπει να γνωρίζου­ f( ( χ) -μ)( f( χ) -Μ) :::; O => f2 ( χ) - ( μ+Μ) f ( χ) + μΜ:::; Ο με το λήμμα που ακολουθεί: => f2 ( χ) - ( μ+Μ) f ( χ) :::; -μΜ => Ι σχύει ο τύπος: Jr( χ )dx = Jr( α + β - χ )dx. Η ισότητα αυ τή αποδεικνύεται εύκολα, αν θεωρή­ Jr2 (χ )dx - (μ + Μ ) Jr( χ )dx ::::; - JμMdx => μετασχηματισμό σουμε τον u = α + β - χ => dx = -du και ξεκινήσουμε από το => Jr 2 (x)dx s; (α - β)μM. 2° μέλος κάνοντας ταυτόχρονα αλλαγή των άκρων ολοκλήρωσης . . . . . . Όμως f{[ α, β] ) (-Ι, Ι) => f 2 (χ) - Ι s; Ο => ι ι) Να υπολογιστεί το ολοκλήρωμα ) => f 2 (χ) ( f 2 ( χ ) - 1 ::::; ο => f4 (χ) s; f 2 (χ) => I = J f( x ) d . f(α - x ) + f( x ) x Jf4 (x)dx :::; Jf 2 (χ)dχ :::; (α - β)μΜ ( Ι). Ι σχύει επί- Υπόδειξη : Να χρησιμοποιηθεί άμεσα το προηγού­ μενο μεθοδολογικό σχόλιο . σης ότι: ( ΞJ χ 0 ε [α,β]){f(χ 0 ) :;t: Ο ) , καθότι η f δεν Απάντηση : Ι = � 2 είναι σταθερή. Οπότε από γνωστό θεώρημα έχου με 1 2 ) Θεωρούμε την συνεχή στο IR συνάρτηση την σχέση Ο < Jf4 (χ )dx ( 2). f : 1R JR. >

Απόδειξη :

.

2

-

2

2

n

>

I

2

I

- -

U

-

>

>

α,

, ω στε

Χ0 Ε

1

β

α

β

α

β

β

α

α

β

α

α

]ι I

=>

-

β

β

α

α

>

>

.

ς

β

U

--

*

I

Ε

χ1 , χ2 Ε

[

0

β

α

Αν

υποθέσουμε

ΕΥΚΛΕΙΔΗΣ Β' 75 τ.3/69

ότι

η

συνάρτηση


------t_ ) , παρουσιάζει τοπικό g ( χ) = τ f (2 χ + 1 dt

Μαθηματικά για την Γ Λυκείου

χ'

1

κρότατο στη θ έση

1

()

I

Jr 2 ( x ) dx � f -21 - -21 . 1

α-

να αποδειχθ εί ότι

Γνωρίζουμε ότι η g είναι παραγωγίσιμη στο JR συ­ νάρτηση. Από το θεώρημα του Fenηat έχουμε την ισότητα g. (1) = Ο. Θεωρούμε ότι: t = u => t = ( χ 2 + I ) u => dt = ( χ 2 + 1 ) du. χ2 + 1 t = χ 2 + 1 => u = 1 . Ά ρα: Για t = I => u = χ 1-, - +I ι 2 + 1) x = g(x) ( J f(u)du = - ( x 2 + 1) J f(u)du => ι ι ?

x ::? + l

( --- )

1 . f g ' (χ) = -2χ 'Ί' f( u )du + � Χ + I χ- + 1 Από την σχέση g ' ( I) = Ο παίρνουμε ι ι 2 2 -1 f -I = Jf(u)du = Jf(x)dx. Από 2 2 ι ι ι ι ι J( f(x) + 1) 2 dx � 0 => Jf 2 (x)dx + 2 Jf(x)dx + ι ι ι 2 2 2 ?

1

()

-

-

2 2 Jf 2 (x)dx � f -21 - -.2Ι ι

()

1

Δίνεται η συνεχής στο σύνολο JR συνάρτηση : JR h � JR και οι αριθ μοί κ, λ Ε JR με lκl * l λ l . Θεωρούμε την ύπαρξη συνάρτησης f : JR � JR για την οποία ισχύει: ( "d χ Ε JR) ( κf ( χ - α) + λf ( β - χ ) = h ( χ)) ( ) να αποδειχθ εί ότι: Η f είναι συνεχής στο σύνολο των πραγματι­ κών αριθ μών. *

1 Jh ( χ )d x. :; ' Jf ( χ - α )d x = κ+λ α β

ο

,

β

Υπο λογίστε το 2 .Jx 2 - 3χ + 2 - .Jx 2 + χ dx. 3,_/-χ2 + χ + 2

r� '

J1

u

Α) i) Αν στην (*) όπου χ θέσουμε α+ β -χ παίρ­ νουμε την σχέση: κf ( β - χ ) + λf (χ - α ) = h ( α + β - χ ) , "d χ JR ( * * ) . Λύνοντας το σύστημα των (*), (**) εύκολα έχουμε: f (χ - α ) = κ 2 κ- λ2 h (χ ) - κ 2 λ- λ2 h ( α + β χ) . Από την τελευταία αυτή σχέση προκύπτει ότι η f είναι συνεχής (;) . ii) Επίσης από την τελευταία σχέση παίρνουμε ότι: Jf(x-α) dx = � κ-lt Jh(x)dx - κ2 λ- /t Jh(α+β-x)dx, οπότε λαμβάνοντας υπ' όψη το μεθοδολογικό σχόλιο2, τελικά έχουμε: ff (χ - α )dx = κ +I λ fh (χ )dx. Β) Έστω η συνεχής συνάρτηση στο διάστημα [0, 1 ] ( ; ), r , [ ο, ι ] --> ΙR, r ( χ) = z.Jx' �3' +2 2 - � "" 3 -χ + χ + 2 => f(x) = 3_3 11 +- χx _ }_3 2 χ . -χ Θεωρώντας τώρα την συνάρτηση h : [0, 1] JR, h ( χ) = έχουμε: f(x) = f(x - 0) = 2 2 2_ 1 h (x) - 2 " 1_ 1 h (0 + 1 - x). ι I ι )dx =-1 f �dx, Επομένως ισχύει: Jf(x)dx =-Jh(x 3 0 1+χ 30 0 οπότε αρκεί να υπολογίσουμε το ολοκλήρωμα του δεύτερου μέλους . Αφού έχουμε Ο ::;; χ ::;; I, μπορού με να θέσουμε χ = ημt => dx = ( συνt )dt => 1 - ημt ( συνt)dt = Χ = J -f -dx 0 1+Χ 2 1 + ημt 2 1 - ημt t)dt = " ημt)dt = ... = -π - 1 . J(I J-- (συν 2 0 συνt 0 ι Τελικά παίρνουμε: ff (χ) dx = 2:6 - }_3 . ε

-

β

β

β

α

α

α

ολοκλήρωμα:

--

β

α

Ρ

α

J§,

'{§-

l

� 3\

-------­

-

J§ -

χ � Ο => ι �ο 2 π

χ � Ι => ι � .": 0

π

π

Έστω συνάρτηση f : JR � JR συνεχής στο JR , ώστε r ( x-l) = ' r χ2 +tx + l ) dt - 'J[( 2t + 3) Jr( θ)dθ]dι Ν α υπολογιστεί το εμβαδό του χωρίου που πε­ ρικλείεται μεταξύ της γραφικής παράστασης της συνάρτησης h : JR � JR,μεh( χ ) = ( χ 2 + χ+ ι ) r ( χ - l ) καιh ( ο) = ι, ο

1 : !

ΕΥΚΛΕΙΔΗΣ Β' 75 τ.3/70

'J' ( ο

ο

ο


------ Μαθηματικά για την Γ Λυκείου 3

------

των ευθειών χ = α, χ = β, με Ο < α < β και του άμε li m �3 = +οο � li-->+moc χ0Jeι' dt = +οο. ξονα Οχ. χ ->+ >:> x στην επίλυση που ακολουΣυνεπώς καταλήγουμε

Παρατηρούμε ότι :

3

θεί: l ι. m - Ι � u = χ - 1, οπότε έ-

Για t = Ο � u = Ο, t = χ χουμε την ισχύ της ισότητας που ακολουθεί: ' ' f( x - I ) = J ( x2 + x + I)f (u) ctu - J ( 2t +3) Jf ( θ)dθ � ο

Ί

ο

f ( χ - Ι ) = ( χ2 + χ + Ι ) f ( u )du ο

[

'

} J [ J θ θ} ο

( 2t + 3) f ( )d t

ο

ο

Χ -> +>:

J

ex

x--> +z

J

e

(

e

-' τ: χ '

,

:

j

f (χ

-

ι)

( )

χ---+lim+Χ> xs·e1' dt ο

:Ι φ ο .

=ο�

{ 3 ξ ε ( α, β } )

\:

ι

= e.

f{ξ) g(ξ) = 1 β Jr(x)dx Jg( x ) dx

ξ

α

χ·' + χ :! h ( x ) = c ( χ 2 + χ + I ) e3 2 -- χ , με h ( Ο ) = Ι � c

h { ψ ( χ' + χ + l ) e 'i + '

J

_ _

(�)

θεωρούμε τις συναρτήσεις ") f : [α, β] � IR, g : [α, β] � JR, συνεχείς στο διάστημα Ια, β] και {'v'χ ε [α,β])(r ( χ),g ( χ) > Ο) . Να αποδειχθεί ότι:

Έτσι έχουμε:

=>

(�)

ο

{ (χ - Ι) = (χ2 + χ + Ι ) f ( χ - Ι) � f' ( x - I ) - ( x 2 + x + I ) f ( x - I ) = O �

( }

X - HOC

ο

ο

'i-f·γ ( χ - ι ) - 'i f '

ο

_ ___

+ ( x2 + x + I ) f ( x - I ) - ( 2x + I ) χ J f ( θ )d θ �

e

(ψ }

ι ' dι •'. dt ( ; ) . = lιm , χ( χ2 )

2_xe_x_'-= I im = Iim 2 χ eι' dt x--> +oc 2 eι' dt + 2xex'

ο

� f' ( x - I ) = (2x + I ) χ J f ( u )du +

ιη

+'i+')

ξ

σ ng( )

'Εστω: F ( α, β) -> IR, F ( χ ) = e- ' f ( t)dt t flt => = Ι � � F ( α ) = F (β) = Ο. Εδώ για την F εφαρμόζεται το θεώρημα Rolle . . .

" ΆΈστω η συνάρτηση Άρα το ζητούμενο εμβαδό είναι f : [α, β] � IR,α < β, με r {[ α, β]) = lf ( α ),f(β) J , που είναι γνησίως αύξουσα και παραγωγίσιμη β 3 β' α3 α' β� χ 3 χ ' β α \ 2 2 3 3 +P Ε = fh ( x )dx = !l e 3 + 2 + x ) dx = e + - e + + . στο διάστημα Ι α,β Ι1. Λαμβάνοντας υπ' όψη ότι η αντίστροφή της Γ : [ f ( α ),f(β) J � [α,β], είναι J

Να υπολογιστεί το όριο: ,Il�

παραγωγίσιμη στο διάστημα [ r ( α ),f(β) ] , να αποδειχθεί ότι: f{ β ) β Jr ( x ) dx + J Γ 1 ( χ )d χ = βf ( β) - αf ( α) .

(χfeι'dt)�

α r(α ) Αποδεικνύεται ότι: ( Vx ε IR )( ex ;::: χ + Ι ) (Άσκη­ ση 8 , σελίδα 292 του βιβλίου των Μ αθηματικών της κατεύθυνσης). Έτσι έχουμε: t = f ( χ ) � χ = f--1 ( t) � dx = [ Γ 1 ( t ) } dt. .... και x ε !R�e· :::: χ + Ι �/ :::: t2 + l > t2 � f/dt > Jt2 dt = � , Jf ( χ ) dx = Jf ( u ) du =. . . (βουβή μεταβλητή) 3 χ >Ο χ

ο

χ

ο

3

β

β

α

α

ΕΥΚΛΕΙΔΗΣ Β ' 75 τ.3171


Επιμέλεια: Γιάννης Στρατής -

Βαγγέλης Ευσταθίου

Η

διδασκαλία της συνεπαγωγής, στους μαθητές Λυκείου Γ.Σ. Τασσόπουλος Καθηγητής Μαθηματικών Βαρβακείου Λυκείου Αν θεωρή σου με δυ ο προτάσεις p , q , τότε εύ­ πάρχει άνθρωπος που κατοικεί στην Καλαμάτα κολα μπορούμε να δώσου με στους μαθητές να κα­ χωρίς να κατοικεί στην Πελοπόννη σο. Αφού λοιπόν θεωρή σαμε λογικό να δεχτούμε ταλάβουν την έννοια της σύζευξης ( p q), καθώς και την έννοια της διάζευξης (p q), αυτών των ότι η μόνη περίπτω ση που είναι ψευδής η πρόταση προτάσεων. Ως γνω στόν η σύζευξη αλη θεύει μό­ ( p => q) είναι όταν ( η p αλη θής) και (η q ψευδής), νον όταν και οι δυο προτάσεις p , q αλη θεύουν, ενώ οφείλουμε να δεχτούμε ότι σε όλες τις άλλες περι­ η διάζευξη αλη θεύει μόνον όταν αλη θεύει μια πτώσεις θ α είναι αλη θής. Δεχόμαστε' λοιπόν ότι η πρόταση (p => q), θ α είναι αλη θής, μόνο στις πε­ τουλάχιστον από τις p , q. Επίσης εύκολα μπορεί να οριστεί -η άρνηση ριπτώσεις : I ) · η p αλη θής και η q αλη θής μιας πρότασης p που συμβολίζεται με p και είναι 2 ) · η p ψευδής και η q αλη θής ( Ι ) , αλη θής μόνον όταν η p είναι - ψευδής. -Είναι δε 3) · η p ψευδής και η q ψευδής προφανές ότι η άρνη ση της p , δηλαδή η p είναι η ίδια η p . (Αρνούμαι ότι δεν ήμουν παρών, σημαί­ αφού στην περίπτω ση που είναι : (η p αλη θής) και νει ότι ήμουν παρών). (η q ψευδής) δεχτήκαμε ότι η πρόταση ( ρ => q) εί­ Η έννοια όμως που είναι αρκετά δύ σκολο να ναι ψευδής. Παρατηρείστε ότι η παραδοχή (Ι) γίνει κατανοητή από του ς μαθητές, και όχι μόνο, γράφεται συντομότερα ως διάζευξη : (η p ψευδής) είναι η έννοια της συνεπαγωγής με υπόθεση p και ή (η q αλη θής). Συ μβολικά : (Ρ q ) . συμπέρασμα q, που συμβολίζεται με (p => q) και Σύμφωνα αυτή την παραδοχή (ορισμό), η πρόταση: αποδίδεται με την έκφραση " Αν p τότε q " . <<Αν ο Α κατοικεί στην Καλαμάτα, τότε ο Α κατοικεί στην Η προ σπάθ ειά μας λοιπόν αφορά στο πώς θ α Πελοπόννησο», αληθεύει για κάθε άνθρωπο Α, είτε αυτός μπορέσουμε όχι μόνο να καταστή σουμε κατανοη­ κατοικεί Καλαμάτα είτε όχι, δηλαδή είτε είναι αληθής η τό τον ορισμό αυτής της έννοιας αλλά και στο πώς υπόθεση στην είτε όχι, όπως ακριβώς η συνεπαγωγή: <<Αν κόψω θ α γίνει ο ορισ μός λογικά αποδεκτός, θ α συ μφωνεί το χέρι μου τότε θα ματώσει» είναι προφανώς αληθής ανε­ δηλαδή με την κοινή λογική , αν και αυτό δεν είναι ξαρτήτως του αν έκοψα το χέρι μου ή μάτωσε άJJ..o τρόπο. πάντοτε απαραίτητο στα Μαθηματικά. Χαρακτηριστικό δε παράδειγμα συνεπαγωγής ψευδή υπό­ Ισχυρίζεται κάποιος ότι για οποιονδήποτε άνθρωπο θεση αποτελεί η απλή λαϊκή έκφραση ν εσύ είσαι Δεσπό­ τότε εγώ είμαι Πρωθυπουργός», που χρησιμοποιείται Α ισχύει το εξής: «Α ν ο Α κατοικΕί στην Πελοπόννη­ όταν είναι γνωστό ο πρώτος δεν είναι Δεσπότης (ψευδής τότε ο Α κατοικεί Καλαμάτα». σο, στην υπόθεση). Φαίνεται ότι ο απλός λαός έχει κατανοήσει πλήρως Έχου με τότε μια συνεπαγωγή (p => q), όπου p , την έννοια της συνεπαγωγής πρίν οι "σοφοί" δάσκαλοι q είναι αντιστοίχως οι προτάσεις : χεφήσουν να του την διδάξουν. p : ο Α κατοικεί στην Πελοπόννη σο Δ ιπλt1 συνεπαγωγή t1 ισοδυναμία q: ο Α κατοικεί στην Καλαμάτα (p <=> q), εννοούμε προφανώς Για να ανατρέψουμε έναν τέτοιο ισχυρισμό, πλέονΜετητοσύσζεύμβολο η [(p => q) (q=>p)], των δυο συ­ : υξ δηλαδή να αποδείξουμε ότι αποτελεί ψευδή πρό ­ νεπαγωγών (p => q), (q=>p ). Επομένως η πρόταση ταση, είναι λογικά αποδεκτό , πως αυτό επιτυγχά­ που αποδίδεται με το μβολο (p <=>q) οφείλου με σύ νεται μόνον αν διαπιστώσου με ότι υπάρχει περί­ να δεχτούμε πως είναι αλη ής μόνον όταν αλη θεύ­ , θ πτωση να είναι (η p αλη θής) και (η q ψευδής). υν και οι δυο συνεπαγωγές (p => q), (q=>p) , δηλαδή Στο παράδειγμά μας πράγματι ο ισχυρισμός αυ­ ομόνο στις περιπτώσεις που είναι και οι δυο προτά­ τός αποτελεί μια ψευδή πρόταση, αφού υπάρχει πε­ εις , q αλη θείς ή και οι δυο ψευδείς, αφού στις υ­ σ p ρίπτωση ο Α να κατοικεί στην Πελοπόννησο , χωρίς πόλοιπες περιπτώσεις θα είναι προφανώς ψευδής. να κατοικεί στην Καλαμάτα, π.χ. να κατοικεί στην Πράγματι : Αν (η p είναι αλη θής και η q είναι Τρίπολη. Κάποιος άλλος ισχυ ρίζεται ότι : τότε δεν αλη θεύει η συνεπαγωγή ψευδής), «Α ν ο Α κατοικεί στην Καλαμάτα, τότε ο Α (p => q), ενώ αν (η q αλη θής και η p ψευδής), τότε κατο ικεί στην Π Ελοπόννη σο». δεν αλη θεύει η συνεπαγωγή ( q=>p ). Η πρόταση Έχουμε τότε μια αλη θή πρόταση , διότι δεν υ ν

Λ

}

ν

με

με

της,

«Α

με

ότι

επι­

Λ

ΕΥΚΛΕΙΔΗΣ Β' 75 τ.3/72


Το Βήμα του Ευκλείδη

(p q), λέγεται ισοδυναμία ή διπλή συνεπαγωγή των p, q και αποδίδεται με την έκφραση (p αν <::::>

και μόνον αν q ).

Για να γίνουν κατανοητά τα παραπάνω, παραθέτουμε μερικά παραδείγματα συνεπαγωγών (απλών ή διπλών), των οποίων η αλήθεια ή το ψεύδος θα πρέπει να αιτιολο­ γηθεί αναλυτικά στους μαθητές από τον διδάσκοντα. Π α μ ό.δηγμα I

(α) 3 . 5 <3 �4>2 (α) (ψ) 4. 5<3 �4<2 (α) 2 (α) 3 . 5<3<::::>4<2 (α) (ψ) 4. 5>3 <::::>4<2 (ψ) 1. Για κάθε χ Ε IR. ισχύει: χ>3 �χ2>9(α) 2. Για κάθε χ Ε IR. ισχύει: χ2<9 �χ<3 (α) 3. Για κάθε χ Ε IR. ισχύει: χ2>9 �χ>3 (ψ) Π α ρ (ι.δειγμΗ 4 1 . Για κάθε χ Ε IR. ισχύει: Jx J<5<::::> - 5<x<5 (α) 2 . Για κάθε χ IR. ισχύει: Jx J<-5<::::> 5 <x<-5 (α) 3 . Για κάθε χ Ε IR. ισχύει: Jx J>-5<::::> x <5 ή χ>-5(α) Π u. μ ύ.δi:ιγμα 5 Να χαρακτηρίσετε ως αληθή ή ψ ευδή κάθε μια από τις παρακάτω προτάσεις: 1 . Υπάρχει χ Ε 1R. , τέτοιο ώστε να ισχύει: ( 1 - χ )4 � - ( χ 2 + χ ) 2 2 . Υπάρχει y Ε IR , τέτοιο ώστε, για κάθε χ<Ο να ισχύει: x JY = .J;!Y 3 . Για άπειρες τιμές του χ Ε IR. , ισχύει η συνεπα' γωγη: χ 2 + χ <_ 2 � χ + χ -? < ο και για απειρες δεν ισχύει. 4. Για κάθε χ Ε IR. , ισχύει η συνεπαγωγή: χ2 + [χ[ � Ο � χ4 + χ2 > Ο 5 . Υπάρχει χ Ε IR. , τέτοιο ώστε να ισχύει η ισο­ δυναμία: l x 2 - χ Ι � 2 <=> -2 � χ � 1 6. Υπάρχει χ Ε JR. , τέτοιο ώστε να ισχύει η συνε­ παγωγή: l x 2 - x l :2:: 2 � -2 � χ � 1 Σε περίπτωση που η πρόταση χαρακτηριστεί αληθής, να βρείτε τις τιμές των x,y που καθι­ στούν την αντίστοιχη ισότητα, ανισότητα, συ­ νεπαγωγή ή ισοδυναμία αληθή. l: z 6λ ι α : (Για την περίπτωση που θα έχουμε ανταπόκριση από τους μαθητές). Σαν μια επιπλέον προσπάθεια εμβάθυνσης στα όσα προαναφέραμε δίνουμε μέσα από έναν απλό σχολιασμό μερικές ακόμη πληροφορίες και παρατηρήσεις . Γνωρίζουμε ότι για κάθε α, β Ε 1R. ισχύουν οι ισοδυναμίες: α: + β� = Ο <=> α = Ο και β = Ο α� + β: Ο <=> α Ο ή β Ο 1 . 5>3 �4>2 2 . 5>3 �4<2 Π α ριiδειγμα 1 . 5>3<::::>4>2 2 . 5<3 <::::>4 >2 Π α ρό.δηγ μ Η 3

Ε

4

:;t:

;t.

;t.

: Η άρνηση δηλαδή της σύζευξης (α=Ο και β=Ο ) είναι η διάζευξη ( α ;t. Ο ή β ;t. Ο )

• •

Ρ q ) <=> ( Ρ q ) , (I) Συμβολικα, : ( Επίσης: α . β = Ο <=> α = Ο ή β = Ο α β Ο <=> α Ο και β Ο . : Η άρνηση δηλαδή της διάζευξης ( α=Ο ή β=Ο) είναι η σύζευξη ( α Ο και β Ο ) Συμβολικά : ( p v q) <=> (Ρ g) , (Π) Οι ισοδυναμίες ( 1), (ΙΙ) που ισχύουν για οποιο­ δήποτε ζεύγος προτάσεων p, q αποτελούν τους νό­ μους του De' Morgan στη Λογική. : Παρατηρείστε επι πλέον ότι: Αφού δεχτήκαμε ότι η συνεπαγωγή (ρ � q) είναι ψευδής μό­ νον όταν (p αληθής και q ψευδής), συμβολικά ( Ρ � q) <=> ( Ρ g) , θα έχουμε και: Λ

·

ν

;t.

;t.

• •

;t.

;t.

;t.

Λ

• •

Λ

(ρ � q) <=> ( q) <=> (ρ v q ) <=> (Ρ v q) ' ( 1 ) Εντελώς όμοια: (q�p)��vp)�qvp), Ρ

Λ

(2)

Από τις ( 1 ), (2) βλέπουμε ότι:

(p � q) <=> (g � p) ,

σχέση (3) ονομάζεται νόμος της αντιθετοαντι­ στροφής και θα μας φανεί αρκετές φορές χρήσι­

: Η

• •

(3)

μος, π.χ. για να δείξουμε ότι για κάθε τρίγωνο ΑΒΓ ισχύει: Β Γ � β γ μπορούμε να δείξουμε ότι: β = γ � Β = Γ, που είναι πολύ εύκολο. Τέλος παρατηρούμε ότι η σύζευξη ( q g ) εί­ ναι πάντοτε ψευδής, για αυτό λέγεται άτοπο (πα­ ράλογο). Αν λοιπόν είναι αληθής μια συνεπαγωγή της μορφής [Ρ � ( q g) J , της οποίας το συμπέρα­ σμα, όπως προαναφέραμε, είναι ψ ευδές, -συμπεραίνουμε ότι θα είναι και η υπόθεση αυτής p ψευδής, άρα η πρόταση p αληθής . : Στην παρατήρηση αυτή βασίζεται η μέθοδος της εις άτοπον απαγωγής, η οποία εφαρμόζε­ ται όταν δεν μπορούμε να αποδείξουμε απευ­ θείας ότι η πρόταση p είναι αληθής, οπότε αρκεί να αποδείξουμε ότι η p είναι ψευδής . ;t.

;t.

mo

Λ

Λ

• •

I.

2. 3. 4. 5. 6.

Β ι βλιογρ α φ ί α

Bittinger Μ . (Logic and proof) Copi I . (symbolic Logic) Exner-Rosskopf (Logic i n Elementary Mathematics) Kilmi ster C . (Language Logic and Mathematics) Novikov Ρ . (Instruction a la logique mathematique) Α . Κ. Κυριακόπουλος (Μαθηματική Λογική)

ΕΥΚΛΕΙΔΗΣ Β' 75 τ.3173


Χάρης Καλλ ι γάς 1 946 - 2009

Χαράλαμπος (Χάρης) Καλλιγάς γεννήθηκε το 1 946 στην Αθήνα. Ο πατέρας του με κα­ ταγωγή από την Κεφαλλονιά και η μητέρα του με καταγωγή από την Τραπεζούντα, γεν­ νή θηκαν και οι δύο στην Ρωσία και κατέληξαν στην Ελλάδα προσφυγες μετά την Μι­ κρασιατική καταστροφή . Τέλειωσε το Ε ' Γυμνάσιο Αθήνας και στη συνέχεια έλαβε πτυχίο από το Μαθημα­ τικό Τμήμα του Πανεπιστη μίου Αθηνών. Παρακολούθησε διετές μεταπτυχιακό πρόγραμμα με θέμα «πρότυπα δημιουργίας εκπαι δευτικού λογι σμικού» στο Τμή μα Πλη ροφορικής του Πανεπι­ στη μίου Αθηνών. Επιπλέον παρακολούθηκε μεγάλο αριθμό σεμινάριων σε θέματα Ψυχολογίας της Μάθησης, Επικοινωνίας, Παιδαγωγικής, Μαθη ματικών και Πλη ροφορικής. Υπήρξε υποψή­ φιος διδάκτωρ στο Τμή μα Ψυχολογίας του Παντείου Πανεπιστημίου με ερευνητικό θέμα: <<Δυ­ ναμι κή κοινωνικών αναπαραστάσεων, προϋποθετικότητα και θεσμική πολιτική: Η διαχείριση αξι ­ ολογικών διεργασιών της εκπαι δευτικής δι αδικασίας ως προς τη χρήση των νέων τεχνολογιών» .

Ουσιαστικά η διατριβή του αποτελούσε τη σύνοψη των επί πολλά έτη ερευνητικών ενδιαφερό­ ντων του που κυρίως ήταν η κατάλληλη διδακτικά και παιδαγωγικά χρήση των Νέων Τεχνολο­ γιών στο χώρο της εκπαίδευσης. Δυστυχώς ο πρόωρος θάνατός του δεν του επέτρεψε την ολο­ κλήρωση της διατριβής, η οποία είχε στόχο να αποτελέσει σημείο αναφοράς και ορόση μο στον ευαίσθητο τομέα της σωστής χρήσης της Πλη ροφορικής στα εκπαιδευτικά δρόμενα. Παντρεύτηκε το 1 97 1 και χώρισε 1 Ο χρόνια μετά. Είναι πατέρας δύο αγοριών του Διονύση και του Μανου, των οποίων λίγο μετά το χωρισμό ανέλαβε την επιμέλεια, πράγμα αρκετά σπά­ νιο για τα ελληνικά δεδομένα. Εργάστηκε, ως ιδιοκτήτης Φροντιστηρίων Μέσης Εκπαίδευση ς και Εισαγωγικών Εξετάσε­ ων, ως καθηγητής στα Μεταλυκειακά Φροντιστή ρια, σε Ιδιωτικό και Δη μόσιο Γυμνάσιο και σε Φροντιστήρια Εισαγωγικών Εξετάσεων για την Τριτοβάθμια Εκπαίδευση . Διετέλεσε υπάλληλος στο ΥΠΕΧΩΔΕ ( 1 Ο έτη Προϊστάμενος Τμήματος Πληροφορικής) από το 1 98 1 μέχρι την η μέρα του θανάτου του, εξαιτίας του καρκίνου, στις 3 1 / 1 0/ 2009 . Παρότι για περίπου 30 χρόνια εργάστηκε στο ΥΠΕΧΩΔΕ, μεγάλες αγάπες του Χάρη και κεντρική δραστη­ ριότητα υπήρξε πάντα αφενός η Παιδεία και αφετέρου η κοινωνική και συνδικαλιστική δράση και προσφορά με στόχο ένα κόσμο δημοκρατικό και δίκαιο. Ο λόγος που δεν διορίστηκε στην Εκπαίδευση , αλλά παρέμεινε στο ΥΠΕΧΩΔΕ είναι ότι του ήταν αδύνατο να μετακινηθεί μόνος και χωρίς βοήθεια με δύο μικρά παιδιά στην Επαρχία. Ο Χάρης ήταν από τους πρωτεργάτες, μαζί με πολλους άλλους συναδέλφους, των αλλαγών στη διοίκηση, τη δομή , τη λειτουργία και κυρίως το πνεύμα της ΕΜΕ, μετά την επταετή δικτα­ τορία. Αποφασιστικός ήταν ο ρόλος του στην οργάνωση της διακίνησης και διάθεσης του έντυ­ που υλικού (Ευκλείδης) της ΕΜΕ. Με χρήση του δικού του αυτοκινήτου γύριζε τα σχολεία της Αττικής φορτωμένος με περιοδικά της ΕΜΕ και διέδιδε τον Ευκλείδη στηριζόμενος κυρίως στην προσωπική επαφή με τους συναδέλφους. Συμμετείχε και άφησε τη σφραγίδα του σε πολλές από τις δραστηριότητες της ΕΜΕ. Ενδει-

ΕΥΚΛΕΙΔΗΣ Β ' 75 τ.3/74


κτικά αναφέρουμε τη συμμετοχή του σε Επιτροπές Παιδείας και Αναλυτικών Προγραμμάτων και Συντακτικές Επιτροπές περιοδικών της ΕΜΕ. Η κοινωνική και συνδικαλιστική του δράση στο ΥΠΕΧΩΔΕ αποτυπωθηκαν κυρίως στα ε­ ξής: Αιρετό μέλος του Υπηρεσιακού Συμβουλίου, ταμίας και υπεύθυνος σύνταξης της εφη μερί­ δας της Πανελλήνιας Ομοσπονδίας Υπαλλήλων και Αντιπρόεδρος του Συλλόγου Υπαλλήλων. Πολύ ση μαντικό υπήρξε και το επιστημονικό του έργου, κεντρικός άξονας του οποίου ήταν η σύνδεση της Πληροφορικής με τη Διδακτική και τα Παιδαγωγικά. Διετέλεσε εξωτερικός επι­ στη μονικός και τεχνικός συνεργάτης της Ομάδας Διοίκησης του Π.Ι, Υπεύθυνος της Ομάδας Εργασίας του κόμβου του ΥπΕΠΘ για το έργο και Συντάκτης του Τεχνικού Δελτίου Έργου & Υποέργων της πράξης της ΚτΠ: «Προετοιμασ ία του Δασκάλου της Κοινωνίας της Πληροφορίας/ Αρχική Επιμόρφωση όλων των Εκπαιδευτικών στις ΤΠΕ» ( 1 999-2004) . Συμμετείχε ως επιμορ­ φωτής ΤΠΕ για τα Μαθη ματικά, στο πλαίσιο του προγράμματος «ΟΔ ΥΣΣΕΑΣ», που αφορά στην αξιοποίηση των Δικτυακών και Υπολογιστικών Τεχνολογιών στην Εκπαίδευση . Επίσης συ μετείχε ως επιμορφωτής στο 3 ° ΠΕΚ Αθήνας στο αντικείμενο : «Προϋποθέσεις χρήσης των ΤΠΕ στη διδακτιΚή πράξη» .

Συμμετείχε στη συγγραφή μεγάλου αριθμού άρθρων σε περιοδικά και εισηγήσεων σε Διε­ θνή και Ελληνικά Συνέδρια. Υπή ρξε μέλος της συγγραφικής ομάδας που ανέλαβε, κατόπιν δια­ γωνισμού, τη συγγραφή των σχολικών βιβλίων Μαθηματικά Α ' Γυμνασ ίου - Βιβλίο μαθητή και Μαθηματικά Α ' Γυμνασ ίου - Βιβλίο Καθηγητή.

Χαρακτη ριστικό της υπευθυνότητας, της δύναμης του χαρακτή ρα, και της ανάγκη ς για άρ­ τια προσφορά του Χάρη είναι το εξής: Παρότι ήταν αρκετά κοντά στην τελική φάση της αρρώ­ στειας του, διέθετε τέτοιο κουράγιο και ψυχική δύναμη ώστε να αναλάβει μόνος του το δύσκο­ λο έργο της νέας μορφοποίησης του βιβλίου των Μαθη ματικών της Α' Γυμνασίου, προκειμένου να ενσωματωθούν σε νεώτερη έκδοση όσες από τις αλλαγές και βελτιώσεις που προτάθηκαν από τους συναδέλφους καθηγητές Μαθη ματικών κρίθηκαν θετικές. Μάλιστα αξίζει να τονισθεί ότι ο ίδιος είχε την πρωτοβουλία για τη σύσταση ομάδας εμπείρων μάχιμων μαθη ματικών και Σχολι­ κών Συμβούλων η οποία, σε συνεργασία με τα στελέχη των Μαθη ματικών του Π. Ι . , ανέλαβε την αξιολόγηση των προτάσεων για αλλαγές και βελτιώσεις που κατατέθηκαν από τους συνα­ δέλφους από όλη των Ελλάδα. Η αγωνιστικότητα και η συνέπεια που έδειξε στο παραπάνω θέμα ήταν ένα από τα χαρακτη ριστικά του Χάρη που ποτέ δεν τα εγκατέλειψε, ακόμη και στη δύσκο­ λη τελευταία διετία της ζωής του που σημαδεύτηκε από την αρρώστεια. Ευχόμαστε, για λογαριασμό του Χάρη αλλά και όσων έχουν ανάλογες πεποιθήσεις, να πραγματοποιηθεί κάποτε το βασικό του όραμα που ήταν ένας κόσμος δη μοκρατικός, με ανθρω­ πιά, δίκαιος, ενωτικός, με έμφαση στις πνευματικές αξίες και με ευαισθησία στις ανάγκες των άλλων. Η δυσκολία πραγμάτωσης ενός τέτοιου οράματος καθώς και η «απολυτότητα» στα πι­ στεύω του στάθηκαν και η αιτία που ο Χάρης συνάντησε στη ζωή του αρκετές ματαιώσεις και aπογοητεύσεις. Κλείνοντας, παραθέτουμε αυτούσια τα λόγια ενός μαθητή του φροντιστηρίου που άνοιξε ο Χάρης στην περιοχή του Παπάγου,στα χρονια της δικτατορίας. Είμαι σ ίγουρος, ότι για όλους εμάς που ζήσαμε αυτό το πρωτόγνωρο φροντιστήριο, η εποχή εκείνη μας σημάδεψε για όλη μας τη ζωή. Πρέπει να ήταν μι α από τις ελάχιστες εφαρμογές της εκ­ παίδευσης τύπου

SummerHίll.

Να τον θυμόμαστε πάντα με αγάπη .

ΕVΚΛΕΙΔΗΣ

Β ' 75 τ.3/75


Ε u κλε i δ n ς

π ρ ο τ ε i vε ιι

Επιμέλεια: Γ . Κ . Τ Ρ Ι ΑΝΤΟΣ - Ν . Θ . ΑΝΤΩΝ Ο Π Ο Υ ΛΟΣ

ΑΣΚΗΣΗ 1 54 ( ΤΕΥΧΟΥΣ 7 1 ) Αν α, β, γ ::?: Ο , α + β + γ = 1 , να δειχθεί ότι

συνάρτηση g έχει μέγιστη τιμή για γ =

, II - - 2αβ γ ::; α - + β 2 + γ 2 :5: 1 - 2 αβγ . 27 (Προτείνεται από τον συνάδελφο Γ Ε Ω Ρ Γ Ι Ο N I K H T A K H - Σητεία Κρήτης ) 3 I 1 = -(γ - 1)[ (γ - Ι) 2 + γ] = - (γ - 1) - γ(γ - Ι ) = g(γ) 4 4

1 γ .!. β = - = , α = 1 - (β + γ) = .!. . 2 3 3

, I I 3 ειναι: B ,nax = - 4 (3 - 1)

Λύσεις έστειλαν Μ ο υ ρο ύ κος

, 3 I , με g (γ) = - - (γ - I)- - 2 γ + I = - - (3 γ - + 2 γ - I) 4 4 ,

και g '(γ ) = 0 <=> γ = - 1 ν γ =

Ά ρτ ας ,

.!. , g'(γ) > Ο <=> 3

.!..!. ::; α 2 + β2 + γ 2 + 2 αβγ ::; Ι . 27 Ε�αι : α2 + β 2 + γ 2 + 2 αβγ = (α + β + γ f -2(αβ + βγ + γα) + 2 αβγ = I - 2 (αβ + βγ + γα) + 2 αβγ = I - 2(αβ + βγ + γα - αβγ) = I - 2[α(β + γ) +

βγ(Ι - α)] = 1 - 2 [ α(β + γ ) + βγ(β + γ)] =

I - 2( β + γ)( α + βγ) = 1 - 2(1 - α)( α + βγ) ::; 1

αφού είναι (1 - α)( α + β γ ) ::?: Ο με την ισότητα να ισχύει όταν δύο από τους τρεις αριθμούς είναι ίσοι με Ο και ο τρίτος είναι ίσος με την μονάδα. Αποδεικνύουμε στην 11 , , , συνεχεια οτι Α ::?: - , οπου 27 Α = α 2 + β2 + γ2 + 2 αβγ . Σύμφωνα με τα προηγούμενα οπότε αρκεί να

' ' ' I I ' η το αυτο δ ειχθ ει οτι : I - 2 Β ::?: 27 Β ::;

επίσης

8

. Δηλ. Β ::;

27 οι συνάδελφοι:

Μ ή τ σιο ς

Γιώ ργος

Κ ατσάνος

-

Π ρ έβεζα ,

! 55

( ΤΕΥΧΟΥΣ 7 1 )

Σε κυρτό τετράπλευρο ΑΒΓΔ είναι:

ΔΕΛΗ ΣΤΑΘ Η - Κάτω Πατήσια)

Α = 1 - 2 ( α β + βγ + γα - αβγ)

1 1

8 27

.

Β αγγέλη ς - Ρ ά μ ια Ρ οδόλφο ς

Μ πό ρ η ς - Δ άφνη .

Αρκεί να δειχθεί ότι

είναι

μέγιστη τιμή του Β

- 3 (3 - I ) =

Α γρ ίν ιο ,

Ο ρ έστη ς

ΑΣΚΗΣΗ

Λ ΥΣΗ (από τον συνάδελφο Γ Ι Ω Ρ ΓΟ

-

Η

.!. . Τότε 3

Α = I 20°

.J3 Β = Δ = 90° ' ΓΔ2 - ΑΒ2 = 2 .J3 και AB < . ΓΔ 2 Ι) Να υπολογισθεί το εμβ αδόν του ΑΒΓΔ. 2 11) Να δειχθεί ότι: ΑΓ > ΒΔ και ΑΓ = 0 ΒΔ 2 - εφ1 5 (Προτείνεται από τον συνάδελφο Γ Ι Ω Ρ Γ Ο Α Π ΟΣΤΟΛΟ Π Ο Υ Λ Ο - Μεσολόγγι ) Λ ΥΣΗ (από τον συνάδελφο ΒΑ ΓΓΕ Λ Η ΜΟΥΡΟΥΚ Ο - Αγ ρίνιο ) I) Έστω Ε το ση μείο τομής των ευθειών ΑΔ και ΓΒ. Από την ομοιότητα των ορθογωνίων τριγώνων Γ ΔΕ και ΓΔ2 (ΓΔΕ) , . = ΑΒΕ εχουμε: ( ΑΒΕ) ΑΒ2 ( ΑΒ ΓΔ) (ΑΒΕ)

=

(ΓΔΕ) - (ΑΒΕ) (ΑΒΕ)

ΓΔ2 ΓΔ2 - ΑΒ2 --- - 1 = --....,-ΑΒ2 ΑΒ2

=

Ά

ρα,

(ΓΔΕ) (ΑΒΕ)

-\

=

2 .)3 ΑΒ2

_!_ , όπου Β = αβ + βγ + γα - αβγ = 27

= (α + βγ)(β + γ) = (I - β - γ + βγ)(β + γ) = β + γ - β2 _γ 2 _ 2 βγ + β2 γ + βγ 2 = (γ _ Ι )β2 + (γ2 _ 2 γ + ! )β -

γ( γ - I ) = (γ - 1)β2 + (γ - 1)2 β - γ(γ - I ) � Β = (γ - Ι)β2 + (γ - 1) 2 β - γ(γ - I) .

Αν

γ = Ι ( οπότε α = β = Ο) , είναι Β = Ο . Αν γ :;t Ι , είναι γ - I < Ο και το τριώνυμο ως προς β δέχεται μέγιστη

τιμή Β ιnax =

για

β=-

(γ - ! )2 2( γ - I)

=

Ι -γ

ίση

2

-4(γ - 1)2 γ - (γ - 1) 4

-4(γ - Ι)γ - (γ - Ι )3

4( γ _ J)

4

I Ο<γ<-, 3

Συνεπώς,

με

η

Γ .)3 1 Ι h Ί ' Επισης, ( ΑΒ Ε) = - ΑΒ · ΒΕ = - ΑΒ(ν 3ΑΒ) = - ΑΒ 2 2 2 Ά ( ΑΒ ΓΔ) 2 .J3 ρα, r;; = � (ΑΒΓΔ) = 3 τ.μ . ΑΒ 2 , ν3 - ΑΒ 2 Il) Από το θεώρημα των συνημιτόνων στα τρίγωνα ΑΒΔ και Β ΓΔ έχουμε:

ΕΥΚΛΕΙΔΗΣ Β' 75 τ.3176


-------

ΒΔ 2 = ΑΒ 2 + ΑΔ 2 - 2ΑΒ · ΑΔ · συν \ 20° (\) = ΑΒ 2 + ΑΔ 2 + ΑΒ · ΑΔ ΒΔ 2 = ΒΓ 2 + ΓΔ 2 - 2Β Γ - ΓΔ · συν 60° = ΒΓ 2 + ΓΔ 2 - ΒΓ · ΓΔ

Ο Ε υκλείδης προτείνει . . .

ι => _ ΨΥz � _ ι - ;::: J3 => 3 ΓΙ ;::: J3 ΨΥz J3 νφ

(2)

Μ ε πρόσθεση κατά μέλη των σχέσεων ( I ) ,(2)

�+t+�

:

3

2ΒΔ 2 = (ΑΒ 2 + ΒΓ 2 ) + (ΑΔ 2 + ΓΔ 2 ) + ΑΒ · ΑΔ - ΒΓ · ΓΔ = 2ΑΓ 2 + ΑΓη μΓ 2 · ΑΓ η μΓ 1 -ΑΓ συν Γ 2 · ΑΓ συν Γ 1 = 2ΑΓ 2 - ΑΓ 2 συν (Γ 1 + Γ 2 )

Η

2

2 r:; j \f 3

·

xyz

-

(31 I χ

Ι r:; ;::: 3 ν 3 z

=> - + - + -

I

y

1

χ=y=z=

J3

τ

(2)

(4) και

(4) δίνει:

με την ισότητα να ισχύει

. ελ ικα, ειναι ·

·

= 2 νr;;3 = f ( ι

, ι , ι ) . J3 J3 J3

· r (x, y, z) 2Η:

( από τον συνάδελφο ΚΟΥ ΚΑΚΗ - Κάτω Απόστολοι Κιλκίς ) Παρατηρούμε ότι η f γράφεται:

ΛΥΣΗ

ΔΗΜΗΤΡΙΟ

2 ΑΓ 2 = - => - = ------::J3 ΒΔ 2 - εφl 5 °

2

2 - (2 - J3 )

Μπό ρης Φίλιππος - Χ ανιά,

χ, y, z είναι θετικοί αριθμοί με χ 2 + y 2 + z 2 = Ι ,

τότε να βρεθεί η ελάχιστη τιμή της συνάρτησης :

χ 2 χ 2 y 2 / 22 22 f(x, y, z) = - + - + - + - + - + - . Υ z χ z Υ χ

--

--

;,

g " ( t ) = .!. ( h + ) > Ο , για κάθε t ε (O, J) . 4 ν' t5 ν' t 3

Από την ανισότητα του JENSEN έχουμε ότι

χ 2 + Υ2 + z2 g(x 2 ) + g( / ) + g(z 2 ) ������ ;::: g( ) 3

(Προτείνεται από τον συνάδελφο Γ Ι Ω Ρ Γ Ο ΤΣΑΠΑΚΙΔΗ - Κυψέλη Αιτ/νίας ) . Λ Υ Σ Η ι Η (από τον συνάδελφο ΑΘΑΝΑΣIΟ ΚΑΛΑΚΟ - Κ.Πατήσια)

ι

3

� f(x, y, z) ;::: g(.!.) = Ι - 3 = 2 J3 => 3

χ 2 χ 2 y 2 y 2 22 22 f(x, y, z) = - + - + - + - + - + - = Υ z χ z Υ χ 22 22 2 /+ + χ χ 2 + y2 1 _ χ 2 1 _ / + + = -- + -χ Υ z χ Υ 2 Ι-z ι ι + -- = -(χ + y + z) + (- + -ι + -) (I) χ y z z

ι

3

f(x, y, z) ;::: 2 νr:;j = f( ι

3

J3

, ι , ι ) J3 J3 J3

Λύσεις έστειλαν επίσης οι συνάδελφοι:

Γ ίαννης Ηλιόπουλο ς Ρο δ όλφος Μπό ρης - Δ άφνη, Βαγγέλη ς Γ ιάννα ρ ος - Πύ ργος, Μ ουρούκος - Αγρ ίνιο , Διονύ m1 ς Δη μή τρης Μ αντζα β ίνος - Λ α μ ία , Αντώνι1 ς Ι ω ανν ίδ ης Λ άρισα, ο Πολιτικό ς Μ ηχανικός Ανδ ριi ς Ι ωάννη ς - Α θιΊνα.

---

-

Από την ανισότητα του SCHWARTZ έχουμε: 2 2 2 2 ( 1 : + 1 2 + 1 )(χ + / + z ) ;::: (1 - x + l · y + · z)

::::> 3 ;::: ( x + y + z) 2 ::::> x + y + z � J3

χ 2 χ 2 y 2 y 2 22 22 f(x, y, z) = - + - + - + - + - + - = Υ z χ z Υ χ 22 22 2 2 y + + χ χ 2 + / I _ χ 2 I _ y 2 I _ 22 + + = -- + -- + -- (\) Υ Υ χ z χ z Θεωρούμε την συνάρτηση g : (0, I) --t R με τύπο t g( t) = lft . Παρατηρούμε ότι η ( I ) γράφεται: I - χ 2 I - y2 I - z 2 f ( x , y, z) = -- + -- + -- = Υ χ z 2 2 2 = g(x ) + g(y ) + g(z ) (2) 2 2 για κάθε x, y, z > O : x + y + z2 = 1 . Η g είναι δύο φορές παραγωγίσιμη στο (0, 1) με --

Λύσεις έστειλαν επίσης οι συνάδελφοι: Ροδόλφος 2:ταματογιάννη ς - Δροσιά, Δάω,η. Γιά\'\η.:: Σερέφογί.ου - \Ιεί.ίm α. Κω,· νο; Τζαγκαράκης Γ ιιίψγος Μιiτσιος - Ράμια Αρτα.:: . ΑΣΚΗΣΗ 1 56 ( ΤΕΥΧΟΥΣ 72)

--

·

mιn

2

2

--

3

( I ), λόγω των σχέσεων

μονον οταν

> I => ΑΓ > ΒΔ

J3 c J3 - ι ) J3(J3 + 1)

Αν

f ( x , y, z) ;::: - J3 + 3 J3 = 2 J3

= ΑΓ 2 (2 - συν 60° ) = ΑΓ\2 - .!_) = � ΑΓ 2

ΑΓ 2 => = ΒΔ 2

- ;:::

---=---

(3)

Καλαμάτα,

ΑΣΚΗΣΗ 1 5 7 ( ΤΕΥΧΟΥΣ 72 )

l

� = 2χ 2 - 1 + 2χ� ( ! ), τότε να δειχθεί ότι

Αν

(2)

Από την ω1σότητα αριθμητικού - γεωμετρικού μέσου για ' ' ' 1 1 1 ' τους θ ετικους αριθ μους χ 2 , y- , z- και τους - - -

χ

Υ

z

ισχυει: •

χ2 =

5-

J5 . ( Π ροτεινεται ' απο' τον συνα' δ ελ φο

--

8

Γ. Ν Ι ΚΗΤ ΑΚΗ - Σητεία Κρήτης ) ΛΥΣΗ (από τον συνάδελφο Δ. Γ Ι Ά Ν Ν Α Ρ Ο - Πύργος) Επειδή η ( \ ) ορίζεται για χ Ε R : lxl � ' θέτουμε

ι

ΕΥΚΛΕΙΔΗΣ Β' 75 τ.3/77


Ο Ευκλείδης προτείνει . . .

------

χ = συνω , ω Ε [0, π ] . Η ( I ) γράφεται

3x3 - 7 x2 - 6xy - 2 ι x - 1 8y + 88 = 0 ( ι ) . (Προτείνεται από τον Χημικό Δ Η Μ Η Τ Ρ Ι Ο ΚΑ Ρ Β Ε ΛΑ - Πεύκη ) ΛΥΣΗ ( από τον συνάδελφο Γ Ι Ω Ρ ΓΟ ΤΣ Α Π Α Κ Ι Δ Η - Κυψέλη Αιτ/νίας ) Η εξίσωση ( 1 ) γράφεται ισοδυνάμως: (7 χ + 6y)(x + 3) = 3χ3 + 88 (2) . Αν ήταν χ = -3 η (2)

J

.Jι - συνω = 2συν2ω - 1 + 2συνω η μ2 ω <=> <=>

.J1 - συν ω = συν2ω + 2συνωη μω .J1 - συν ω = συν2ω + η μ2ω

(2)

Από την (2), με ύψωση στο τετράγωνο, έχουμε 1 - συνω = 1 + η μ4ω <=> η μ4ω = - συνω <=>

<=>

3π η μ4ω = η μ(- + ω) 2 3π 3ω = 2kπ + 2 ν ,k

3π 4ω = 2kπ ± (- + ω) , k 2

<=>

2kπ π ω = -- + 2 3 ν

Ε

θα έδινε 0=7, άτοπο, άρα είναι χ ΖΟ

(2)

,k E Z

Β αγγέλης Μ ο ιφοl> κος ι Ί6νναρος Λ ά ρ ισα,

Η λεκτρολόγος

-

Κ.

Μ πό ρ η ς

Δ<Ίφνη ,

-

"ω ι Ι Ι<ιχ.:

­

E . :vi . Π

και ο

Β α σιλέας

Π ολιτικός Μ η χανι κ ό ς

( Προτείνεται από τον συνάδελφο Γ Ι Λ Ν Ν Η Η Λ Ι Ο Π Ο Υ ΛΟ - Καλαμάτα) ΛΥΣΗ ( από τον συνάδελφο Β Α Γ Γ Ε Λ Η \1 0 Υ ΡΟ Υ ΚΟ - Αγρίνιο) Παρατηρούμε ότι: (x - l)P(x) = x 7 - 1 ( \ )

' και τελικα

Το υπόλοιπο της διαίρεσης P( l ) = 7 .

Ρ( χ) : ( χ - I) είναι ίσο με

Επομένως, υπάρχει μοναδικό πολυώνυμο

Q( χ) τέτοιο, ώστε να ισχύει P(x) = (x - l)Q(x) + 7 (2). Στην ταυτότητα (2) θέτουμε

8

Π ατΙΊ σ ι α.

Π έτ ρ ο ς

.

Να βρεθεί το υπόλοιπο της διαίρεσης: Ρ(χ 7 ) : Ρ(χ )

Δη μι1 φ η ς

όπου χ το χ 7 και έχουμε την ταυτότητα (3). Β άσει της ( I ) η (3) P(x 7 ) = (x 7 - \) Q(x 7 ) + 7

γράφεται: Ρ( χ 7 ) = (χ - \)P(x)Q( x 7 ) + 7 ή το αυτό

Γ ιάννη ς

Σ καλτσάς

Δ η μ Ι1 τ ρ ι ο ς

-

P(x 7 ) = P(x)[(x - l )Q(x 7 )] + 7 = P(x)R (x) + 7 (4)

Μ αντζα β ίνος -

Μ ου ρ ο ί>κος - Α γ ρ ίν ι ο .

Κ αλαμ<Ίτα,

' \ ιηα.: .

1 59 ( Τ Ε ΥΧΟΥΣ 72 ) Έστω Ρ( χ) = χ 6 + χ 5 + χ 4 + χ3 + χ 2 + χ + 1

Λ ά ρ ισ α . Γ ι ιίJ ργος Δεληστάθης - Κ . Γl αηΊ σ ι α , Γ ι ιίφγος

Κ αλάκος

Μ η χαν ικό�

Ε . Μ. Π Α ν δ ρ Ι 1 ς ι ω ιΊνν η ς - Α θ Ι1να.

Ο ρέστη ς Κ ατσάνος - Π ρέβεζα, Α ντιίΝ η ς ι ωανν ί δ η ς -

Β αγγ{:λη ς

Ρ rΊ. μ ι ιι.

ΑΣΚΗΣΗ

Λύσεις έστειλαν επίσης οι συνάδελφοι:

Λαμία,

Μ ι1 τσιος -

Κ α ρ δ α μ ίτσης - Κ ο ρ ω π ί

5 - .JS

Ρ ά μ ι α Ά ρτας,

Γ ι rί) ργος

Αγρίνιο,

Σ κ αλτσιΊς

Λ α μ ί α . Α\'Ι<:)\·ηc Ι ωωνίδης

Χ υ η1 ρ η ς - Κ έ ρ κ υ ρ α , Δ η μ Ι1 τ ρ ι ο ς Κ ο ι ι κύ. κ η � - Κ ύ. τ ω Απόστολο ι Κ ιλκίς, Ν ι κόλαο ς Π α uί.οιι Χ αλ κ ί δ α , Ο

--

Μ ΙΊ τ σ ι ο ς -

Πύργο�. Γl έφος

-

Δ η μ Ι1 τ ρ η ς Μ αντζα β ί\'0�

.JS - 1 π π π => 4η μ 2 - + 2η μ - - 1 = 0 => η μ - = 4 10 10 10

.J5 - ι 1 - -(2) => χ2 = --4'-----2

Α γ ρ ί ν ι ο , Γ ιάνν η ς Η λΗ)πουλος ­

Καλαμάτα, Κ ων/νοc Τζαγκαρ<Ίκης - Χ αν ι <Ί , Δ ι ονl> σ η ς

π π ::::> 4(1 - η μ 2 -) - 3 = 2η μ 10 10

4

7 = 3χ2 - 9χ + 27 + -- Επειδ χ+3

Λύσεις έστειλαν επίσης οι συνάδελφοι:

3π 2π π π συν - = η μ - ::::> 4συν 3 - - 3συν - = 10 10 10 10 π π π ' π 2η μ - συν - => 4συv- - - 3 = 2η μ 10 10 10 10

.J5 - 1

χ +3

οπότε θα είναι: χ = -4 ν χ = -2 ν χ = - ι ο ν χ = 4 Από την (2) για κάθε μία από τις παραπάνω τιμές του χ, παίρνουμε τις αντίστοιχες τιμές του y που είναι y = 22 ν y = 1 3 ν y = 8 ι ν y = 2 .

' 2π ' ια τον υπο λ ογισμο του συν - , εχουμε 5

--

3χ3 + 88

χ + 3 να διαιρεί τον αριθμό 7 . Δηλ. χ + 3 = - Ι ν χ + 3 = ι ν χ + 3 = - 7 ν χ + 3 = 7

3π ' ' ' σε κα' θ ε περιπτωση εχουμε χ 2 = συν- ω = συν-' - . 10 2π 3π 1 + συν - 1 - συν 3π 5 5 Ά ρα χ -' = συν 2 (3) 2 2 10

' 2π π Συνεπως, συν - = η μ - = 5 10

Ίχ + 6y =

-3 , οπότε

όλες τις τιμές των ακεραίων χ, y , πρέπει ο ακέραιος

3π 2kπ 3π 5 ω = 2kπ - ω = -- - 2 5 10 ι λύσεις της (2) στο διάστημα [Ο,π] είναι : 7π ' 3π ω = - ν ω = - . Ε πειδη ει,ναι 1 0 10 3π 3π 7π συν - = συν( π - -) = -συν 10 10 10 '

Γ

<=>

*

ή οι αριθμοί 7 x + 6 y , 3x2 - 9χ + 27 είναι ακέραιοι για

-

Ε Ζ ::::>

------

Η λιόπουλος

Α γ ρ ίν ι ο ,

Κουκ<Ίκης

Όπου R(x) = (χ - I )Q(x) .

Α θ ανάσιος -

Με αποτέλεσμα το ζητούμενο υπόλοιπο να είναι υ = 7 Λύσεις έστειλαν επίσης οι συνάδελφοι: Δ η μ ι] τ ρ ι ο ς .

Κ άτω

Α π όστολο ι Κ ιλκίς, Ο Η λεκτρολόγος Μ η χανικός Ε . Μ . Π Β α σιλέας Κ α ρ δ α μ ίτσης - Κ ο ρ ω π ί

και ο

Μ αντζαβ ίνος - Λ α μ ία , Π έτρος

Π ολιτικός

Α θ αν6σιος

Μ η χανικός ΑνδρΙΊς ι ω <Ίννη ς - ΑθΙΊνα.

Α Σ Κ Η Σ Η 1 58 ( ΤΕΥΧΟΥΣ 72 ) Να βρεθούν οι ακέραιες λύσεις της εξίσωσης:

ΕΥΚΛΕΙΔΗΣ

(4) αποτελεί την ταυτότητα

της Ευκλείδιας Διαίρεσης Ρ( χ 7 ) : Ρ( χ)

ΡοΜ)λφος -

Η

Κ αλάκος

Κ.

Σ καλτσάς - Α γ ρ ί ν ι ο , Π αηΊ σ ι α ,

Γ ι ιίJ ργος

Τσαπακίδης - Κ υ ψ έλη Α ιτ/νίας, Αντιί)\ι η ς ι ω ανν ί δ η ς Λ άρισα,

Β'

Γ ι ιi) ργος

75 τ.3/78

Μ 1]τσιος - Ρ ά μ ι α Ά ρ τας,

Κ ων/νος


Ο Ευκλείδης προτείνει . . .

------

Χ αv ι rΊ . Δ ι ονί>σιος

Τζαγκα ρ rΊ. κ η -;

Δ η μ ιi τ ρ ι ο ς Ν ι κόλαος Δ rΊφνη ,

Κ ουκrΊκης

Π αύλου - Χαλκίδα, και

ο

Προτείνεται από τον Χημικό κ. Δ Η Μ Ή Τ Ρ Ι Ο Κ Λ Ρ Β ΕΛ Α - Πεύκη . 1 7 8 . Αν α, β , γ είναι θετικοί πραγματικοί αριθμοί, τότε

Γ ιάvv α ρ ο ς - Π ίφγ ο ς .

Απόστολο ι

Κάτω

Ρ ο δ ό λφος

Η λεκτρολόγος

Κ ιλκίς,

Μπόρης

Μ η χα ν ι κ 6 ς

-

να δειχθεί ότι ισχύει η ανισότητα: z z / α+β+γ α β , + ----,- + , :2: 2 , r::;;:: β - γ- αΖ, αβγ

Ε.Μ.Π

Β ασ ιλέας Κ α ρ δ α μ ίτση ς - Κ ο ρ ω π ί .

1 60

ο

( ΤΕΥΧΟΥΣ 72 ) Αν α,β,γ είναι θετικοί αριθμοί να δειχθεί ΑΣΚΗΣΗ

Προτείνεται από τον συνάδελφο

ι

4 , 3 3 α β 1_ + - + + - (α' + β3 + γ 3 ) :2: - (α4 + β4 + γ4 ) . 3 β γ α 3

1 7 9.

+ = φ (όπου φ = J5 1 ο λόγος της 2 β χρυσής τομής ) Έστω Ε , Ζ τα μέσα των ΑΔ και Β Γ αντιστοίχως. Στην προέκταση της ΓΔ θεωρούμε ση μείο ΑΒ=α , ΒΓ= β και

(Προτείνεται από τον συνάδελφο Γ Ι Ω Ρ Γ Ο Α Π Ο Σ Τ Ο Λ Ο Π ΟΥ Λ Ο -

Μεσολόγγι)

Λ ΥΣΗ ( από τον συνάδελφο Γ Ι Α !\ Ν Η

Η Λ Ι Ο Π ΟΥΛΟ

- Καλαμάτα ) Από την ανισότητα μέσου αριθμητικού γεωμετρικού γ ια τους θετικούς αριθμούς α β γ ι . β , Υ , , 3 (α> + β>. + γ >. ) παίρνουμε : α

Η τέτοιο, ώστε

ι

'

'

'

= 4 - (α> + β> + / ) = ( 3

=>

β + l + -ι ( �+-

3 3 .!_ α� + β� + γ 4 , ) � :2: -----'---'--3

α3 + β 3 + γ'

4

,

α � + β ' + / ) :2: - ( α :; + β4 + γ4 ) ,

3

3 β γ α 3 Όπου έγ ινε zρήση της ανισότητας του μέσου αριθμητικού των δυνάμεων. Αποδεικνύουμε την

Για

την

_!_

7

ι οο

συνάρτηση f(x) = χ 4 , χ > Ο έχουμε:

-

7

3 3 -4 . . η f, (χ ) = -ι χ :;· , f "( χ ) = - - χ < 0 , που σημαινει οτι 16 4 συνάρτηση f είναι μη κυρτή και συνεπώς, από την ανισότητα του JENSEN παίρνουμε f(x) + f( y) + f(z) x+ y+z => f( ) :2: 3 3 (

χ+Υ+z 3

) :2:

χ

_!_

Ε Ν Η Μ Ε ΡΩΣ Ε Ι Σ - Π Α Ρ Α τ ι � Ρ Η Σ c Ι Σ - Σ Χ Ο Λ Ι Α

1 . Παρακαλούμε όλους τους συναδέλφους που μας

στέλνουν τις εργασίες τους να γράφουν πάνω σε αυτές την ημερομηνία αποστολής τους στην ΕΜΕ καθώς και τα τηλέφωνά τους ( κινητό - σταθερό) 2 . Με μεγάλη Ταχυδρομική καθυστέρηση λά β αμε από τον συνάδελφο Γ Η ;ψγο Λ ποστολι'> τιουλο \11 ε σολόγγ ι , λύση της άσκησης 1 45 του τεύχους 69 (που προτάθηκε από τον συνάδελφο I Ί r ;φγο Τ ρ ι cΊ η ο ) η λύση της οποίας δημοσιεύθηκε ήδη στο τεύχος 73 . 3 . Η στήλη καλωσορίζει την συμμετοχή των τριών φίλων και επί πτυχίω φοιτητών του Α.Π.Θ. Δη μιi η> ιο Κουκr'1 κη (Κιλκίς) - Ν ικ 6λ α ο Π α ύλου ( Χαλκίδα) και Ζελεβ6ρη Άννα (Θεσσαλονίκη) Τους ευχαριστεί θερμά, για τα καλά τους λόγια και τη θετική ανταπόκριση στις ασκήσεις του περιοδικού, τους θεωρεί πλέον πολύτιμους συνεργάτες της στήλης και περιμένει νέες εργασίες τους .

ι

4 + y4 + z4 3

και για χ = α 3 , y = β 3 , z = γ 3 προκύπτει η αποδεικτέα.

-

Λύσεις έστειλαν επίσης οι συνάδελφοι: Δ η μ ιi τ ρ ι ο ς Μ αντ ζα β ίνο-; - Λ α μ ί α .

Α ΟανrΊσιος

Μ ο υ ρο\ ηω ς

Κ αλrΊκος

Ρ ο δόλφο ς Μ π6 ρ η ς - Δ 6φvη , Κ.

Π ατ ιi σ ι α ,

Β αγγέλη ς

Α γ ρ ί ν ι ο . Δ ιο ν ύ σ ι ο ς Γ ι cΊνναρος - Π ι'ι ρ γο ς .

Α ν τι\> νη ς Ι ωανν ί δ η ς - Λ r'φ ισα , Δ η μ ιi τ ρ ι ο ς Κ ο υ κ r'ι κη ς Κ cΊτω Λ πί>στολο ι Κ ιλκίς. Ζελε β cΊ ρ η Άννα - Θ ες/ν ί κ η . και

ο

Η λεκτρ ολ(>γος

Κ α ρ δ α μ ί τση ς - Κ ο ρ ω π ί .

Μ η zα\' ι κ (κ

Γ.Μ.Π

Β α σ ιλέας

Π Ρ ΟΤ Ε Ι :\ 0 \ιJ Ε :\ Α Θ Ε ." Λ Τ Λ

1 77.

Ν α β ρεθούν όλες οι ακέραιες λύσεις της εξίσωσης: 73 ι χ + 779y = 720408 (1 ) .

ΕΥΚΛΕΙΔΗΣ

.fi

< (ΚΛΜ ) < - . 20

Προτείνεται από τον συνάδελφο Γ Ι Ω Ρ Γ Ο Μ Η ΤΣ Ι Ο - Ράμια Άρτας .

-

.!_

_1_ = φ

Προτείνεται από τον συνάδελφο Γ Ι Ω Ρ Γ Ο Μ Η ΤΣ Ι Ο - Ράμια Άρτας . 1 8 0 . Δίνεται οξυγώνιο τρίγωνο ΑΒΓ με A B = l και μέτρα γωνιών Α <Β<Γ που αποτελούν διαδοχικούς όρους αριθμητικής προόδου . Φέρουμε το ύψος ΒΔ. Αν ΑΔ=ρ, όπου ρ (θετική) ρίζα της εξίσωσης 8χ 3 - 6χ + Ι = Ο ( Ι ), τότε να υπολογισθεί το εμβαδόν του τριγώνου ΑΒΓ συναρτήσει του ρ. Αν - επιπλέον- Κ ,Λ, Μ είναι ση μεία των πλευρών Β Γ , Γ Α , ΑΒ αντιστοίχως τέτοια, ώστε οι ευθείες ΑΚ , ΒΛ , ΓΜ να συντρέχουν σε ση μείο Ο και να ικανοποιούν τη ΟΑ ΟΒ ΟΓ . σχεση - + - + -- = 6 , Τ οτε ισχυει: . να δ ειχθ ει. οτι . . ΟΚ ΟΑ ΟΜ

_ . - .

3

α

και ονομάζουμε Θ το σημείο ΗΕ τομής της Η Ε με την διαγώνιο ΑΓ. Να υπολογισθεί ο (ΑΒ ΓΔ) ' . των εμ βαδ ων : λ = λ ογος . (ΘΕΖ)

μέσου

. . α β γ ι . - + - + - + - (α> + β> + / ) . . α β ν ι( . β γ α 3 _,______,_ _!_ . - α·' + β ' + γ ) = :2: 4 4 β γ α 3 _ _ _ _ _ _ _

Μεσολόγγι. Δίνεται ορθογώνιο παραλληλόγραμμο ΑΒΓΔ με

Γ Ι Ω Ρ ΓΟ Α Π ΟΣΗΜ.Ο ΓΙ Ο Υ Λ Ο -

Β'

75 τ.3/79


Τα Μαθη ματικά μας διασκεδάζουν Παναγιώτης Π. Χριστόπουλος Γνωρίζετε ότι : :

• •

Ο Ήρωνας το 2° π.Χ. αιώνα κατάφερε να δημιουρ­ γήσει πολυμέσα (multimedia); Κατάφερε με κίνηση

νερού και ατμό να κάνει αυτόματο θέατρο. Σε ομοίωμα του ναού του Διονύσου, κινεί φιγούρες, ανάβει φωτιές, στεφανώνει το επιστύλιο με λουλούδια, ενώ ταυτόχρο­ να ακούγονται ήχοι από τύμπανα και κύμβαλα. :

• •

:

• •

Ο πρώτος Η/Υ έγινε το 1939 με λυχνίες τις οποίες κα­ τασκεύασε ο Lee de Forest το 1906; Το 1948 έγινε η ε­ φεύρεση του τρανζίστορ, έτσι από το 1956 τα τρανζίστορς αντικαθιστούν τις λυχνίες στους ΗΝ ενώ εμφανίστηκε και η πρώτη γλώσσα υψηλού επιπέδου FOR­ TRAN . Το 1964 έχουμε το ολοκληρωμένο κύκλωμα ( chip) και την τρίτη γενιά ΗΝ. Από το 1 97 1 με τον

μικροεπεξεργαστή (microprocessor) πήγαμε στην τέταρτη γενιά. Σήμερα είμαστε σε μια γενιά «έξυπνων» υπολογιστών, με ιστούς, με νευρωνικά δίκτυα που τα χειριζόμαστε όχι μόνο με τα δάκτυλα αλλά και με τη φωνή, ή τα μάτια, ή την κίνηση στους μυς. Το 1997 ο υπερυπολιστής B ig Blue της ΙΒΜ νίκησε τον Κασπάροβ στο σκάκι. Η Εμι Νέδερ ονομάστηκε μητέρα της σύγχρονης άλγεβρας; Το 1 9° αιώνα και αρχές του 20ου δια­ πρέπουν στα Ευρωπαϊκά Πανεπιστήμια μόνο άνδρες. Έτσι γινόταν ανέκαθεν. Τα Πανεπιστήμια ήταν μόνο για αγόρια. Στη Γερμανία είναι ο Γκάους, ο Ντέντεκιντ, ο Ντιρικλέ, ο Φίσερ, ο Χίλμπερτ, ο Βά­ ιερστρας, ο Κλάιν, ο Αϊνστάιν και ανάμεσά τους, μόνο τρείς γυναίκες που κατά εξαίρεση γίνονται δεκτές στο Πανεπιστήμιο του Γκέτινγκεν αλλά χωρίς μισθό. Μια από αυτές η Εμι Νέδερ (18821935). Όταν ζήτησε τη θέση στο Πανεπιστήμιο υπήρχαν αντιρρήσεις, ήταν γυναίκα με εβραϊκή κα­ ταγωγή και ειρηνόφιλη. Τότε ο Χίλμπερτ είπε «Κύριοι μιλάμε για μια θέση σε πανεπιστήμιο, όχι σε δημόσια λουτρά. Το φύλο δεν πρέπει να αποτελεί κριτήριο». Ο Αϊνστάιν πρόσθεσε «Χθες συζητούσα με τη δεσποινίδα δόκτορα Νέδερ. Η μαθηματική απόδειξη της θεωρίας μου, ακόμα και για εμένα, εί­ ναι μεγάλη αγγαρεία. Οι επαναστατικές τεχνικές της Νέδερ στην αφηρημένη άλγεβρα οδηγούν στην τέλεια μαθηματική διατύπωση της σχετικότητας». Η Νέδερ προσέδωσε στην άλγεβρα την αξιωματι­ κή δομή που επιζητούσε ο Χίλμπερτ στο 2° Παγκόσμιο Μαθηματικό Συνέδριο στο Παρίσι 8-8-1 900 με τα 23 προβλήματα που έθεσε και από τα οποία 2 ακόμη μένουν άλυτα. Η Έμι κατόρθωσε να δημι­ ουργήσει τη μαθηματική διατύπωση της σχετικότητας. Ονομάστηκε μητέρα της σύγχρονης άλγε­ βρας. Γνωρίστε την. Διαβάστε το βιβλίο που κυκλοφόρησε με τίτλο «Εμι Νέδερ η κυρία της άλγε­

βρας».

Κάποτε στην αρχαία Ελλάδα ο σοφός Επιμενίδης από την Κρήτη είπε: «οι Κρήτες είναι πάντα ψεύτες». Το 19° αιώνα διατυπώθηκε ως λογικό παράδοξο του Επιμενίδη. Δηλαδή αφού ο Εmμενί­ δης λέει ότι οι Κρήτες είναι πάντα ψεύτες, ο Εmμενίδης είναι Κρητικός, άρα ο Επιμενίδης λέει ψέματα, άρα οι Κρήτες λένε την αλήθεια, άρα και ο Επιμενίδης λέει την αλήθεια, άρα οι Κρήτες είναι ψεύτες κ.ο.κ. Ο Φαί)λος κί)κλος:

κύκλος των συλλογισμών που οδηγεί από τις αρχικές σου σκέψεις σε δεύτερες και έπειτα σε επαναφέ­ ρει στις πρώτες λέγεται φαύλος κύκλος. Οι πίθ η κο ι : Δύο πίθηκοι μπήκαν 3 φορές σε οπωρώνα και συγκέντρωσαν ισάριθμες ποσότητες από

μπανάνες και καρύδες, ετοιμάζονται για το φαγοπότι αλλά βλέπουν τον ιδιοκτήτη να τους πλησιάζει με το μπαστούνι. Υπολογίζουν ότι για να τους φτάσει θα περάσουν 2 .!._ λεπτά. Τρώνε με λαιμαργία, ο

4

ΕΥΚΛΕΙΔΗΣ Α . 75 τ.3/80


-------

Τα Μαθηματικά μας διασκεδάζουν

--------­

πρώτος που τρώει 1 Ο καρύδες στο λεπτό, τις τρώει όλες στα 2/3 αυτού του χρόνου και βοηθά το φίλο του να φάει τις μπανάνες, προλαβαίνουν στο παρά πέντε. Ο πρώτος καταναλώνει τις μπανάνες 2 φο­ ρές πιο γρήγορα από ότι τις καρύδες. Με πόση ταχύτητα ο δεύτερος τρώει τις μπανάνες; το 1

(

ει' ναι ισο με τ ο

'

� = � ) έχουμε 'J"b ν b

νε λάθος ;

-

1 ; 'Ενας

' ' μαθητης εγραψε 1 -

=-

-1 ' 1 αρα - = -

.

Ι =τ

= � αλλά Γ-Ϊ = i δηλαδή i .i = 1 . 1 άρα ί J7f ν1 ν-1

Γ-ϊ = νΓι =τ και επειδη' ντ-1-

' αρα

2 =

1 1 .

ή

l

-

1

=

I

1 ! ! ! . Πού έκα-

Π ότε γεννή θη κε ; Ο καθηγητής των μαθηματικών την 1 η Απρίλη 20 1 Ο είχε τα γενέθλιά του αλλά πό­ τε γεννήθηκε; Είπε όμως στους μαθητές του ότι θα είναι Χ χρόνων το έτος Χ2 • Τα ψ ά ρ ια : Τρείς φίλοι συμφώνησαν να πάνε την Κυριακή για ψάρεμα και το βράδυ να φάνε μαζί με τις συζύγους τα ψάρια που θα φέρουν. Ο Θωμάς έφερε ένα ψάρι 3 κιλά, ο Σταύρος ένα ψάρι 5 κιλά και ο Χρίστος ένα ψάρι 4 0 κιλά το οποίο όμως πούλησε προς 30€ το κιλό. Το βράδυ αφού έφαγαν τα άλλα 2 ψάρια ο Χρίστος έδωσε στους φίλους του να μοιραστούν 80€. Πώς πρέπει να τα μοιράσουν; Είναι δίκαιο το ποσό που έδωσε στους φίλους του; Ο Χυ μός: Ένας πελάτης ζήτησε χυμό πορτοκάλι. Ο σερβιτόρος έβαλε το χυμό από ένα πορτοκάλι σε ένα κωνικό ποτήρι και έφτασε στο μέσο του ύψους του ποτηριού το συμπλήρωσε με πάγο και το σέρβιρε. Ο επόμενος πελάτης ζήτησε χυμό αλλά χωρίς πάγο, με το χυμό πόσων πορτοκαλιών θα γείσει το ποτ ' ι; Ευχαριστώ τους συναδέλφους μαθη ματικό τη φιλόλογο και τον καθη­ γητή πληροφορικής για τη στήριξη που πρόσφεραν στη στήλη .

Πέτρο Χριστόπουλο,

Οι

πίθηκοι

Σκλάβου Ελένη,

Πρέκα Μαρία

Απαντήσεις στα μαθηματικά μας διασκεδάζουν. 2 3

Ο πρώτος πίθηκος στα - του χρόνου που είχε στη διάθεσή του δηλαδή

2 3

-

2 1 35 90 3 60 60

.2 14= - . - = - = 1 ' 5

λεπτό έφαγε 15 καρύδες αφού τις τρώει με ταχύτητα 1 0 στο λεπτό . Ύστερα βοήθησε το φίλο του να φάνε τις μπα­ νάνες. Άρα από τον υπόλοιπο χρόνο που είχε στη διάθεσή του των 45 δευτερολέπτων έφαγε μπανάνες για 40 δεύτε-

40 , ' ' Δη λαδη' εφαγε -. ρα αφου εφυγαν στο παρα' πεντε. '

40 1 20 = - = 1 3 3 3 60

' ' επει δη' ομως οι καρυ' δ ες και οι μπανανες που

1 30 2 1 ' ' θ μες 1 5 - 1 3 ' ' ' ' ' συγκέντρωσαν ηταν ισαρι = 1 - μπανανες εφαγε ο δ ευτερος πι' θηκος σε - λ επτα' αρα η ταχυτητα που τις έτρωγε ήταν Το Ι

ε ίναι

ίσο

με

60

3 3 2 1 30 5 1 3 5 6 1 ο ' 1 - : - = - : - = - . - = - μπανανες το λεπτο.' 3 60 3 6 3 1 3 1 3

το - 1 ;

�a 'Jbb --Jfνb

Δ εν ισχύει η ιδιότητα

0

Πότε γεννήθηκε; Τέλειο τετράγωνο είναι μόνο το έτος 2025 = 45 2 δηλαδή ο καθηγητής το 2025 θα είναι 45 ετών

άρα γεννήθηκε το 1 980 και σήμερα είναι 30 ετών. Τα ψάρια Οι φίλοι έφαγαν μαζί με τις γυναίκες τους 5+3=8 κιλά ψάρια. Κάθε ζεύγος έφαγε 8/3 κιλά. Ο Χρίστος τα υπολόγισε με 30 € το κιλό που πούλησε τα δικά του έτσι τους έδωσε να μοιραστούν

� .30=80€. Άρα ο Θωμάς θα 3

8 1 ' ' (5- -8 ) . 3 0= -7 . 3 0=70€ και ο Σταυρος παρει (3- - ) . 3 0= - . 3 0= 1 0€. 3

3

3

3

Ο Χυ μ ός Η κορυφή του κώνου είναι στη βάση του ποτηριού και η βάση του είναι τα χείλη του δηλαδή κύκλος αR κτίνας R. Όταν το γεμίσουμε μέχρι τη μέση η επιφάνεια της πορτοκαλάδας είναι κυκλικός δίσκος ακτίνας . Ο -

2

συνολικός όγκος του ποτηριού είναι V= _!_ πR2 . h Με το χυμό από ένα πορτοκάλι φτάνει στη μέση άρα ο όγκος του

, ,

πορτοκαλιου ειναι

3 1 R 2 h 1 1 2h 1 , Υπ= - π( - ) . - = - . - πR . = - V. Το ποτη ρι γεμι' ζ ει με χυμο, 8 πορτοκαλιών. 3 2 2 8 3 8 ΕΥΚΛΕΙΔΗΣ Α ' 75 τ.3/81


Issuu converts static files into: digital portfolios, online yearbooks, online catalogs, digital photo albums and more. Sign up and create your flipbook.